Neuro 4 - Answers

Download as pdf or txt
Download as pdf or txt
You are on page 1of 107

Exam Section: Item 1 of 50 National Board of Medical Examiners

■ Mark Clinical Neurology Self-Assessment

1. A 27-year-old man comes to the physician because of a 2-year history of severe muscle cramping and weakness immediately following calisthenics
or after jogging a short distance. The crampi ng and weakness resolve withi n 24 hours. He also has dark urine during these episodes. Neurolog ic
exam ination, performed while he is asymptomatic, shows no abnormalities. Which of the following is the most likely diagnosis?

A) Alcohol ic rhabdomyolysis
B) Hypokalemic period ic paralysis
C) Muscle phosphorylase defi ciency (McArd le disease)
D) Myasthenic (Lambert-Eaton) syndrome
E) Polymyositis
Correct Answer: C.

Muscle phosphorylase deficiency (McArdle disease) is a glycogen storage disease that affects the muscles. It is characterized by the
inability of myocytes to break down glycogen as a result of the absence of muscular glycogen phosphorylase, which causes exercise
intolerance with fatigue, cramping, and muscle weakness. Muscle cramping may resolve with rest or be permanent. Occasionally, a
"second wind" phenomenon may be seen when blood flow to the muscles increases after approximately 10 minutes of exercise,
delivering fatty acids as fuel and relieving the need for glycogen breakdown. Patients may also have episodes of rhabdomyolysis,
accompanied by red or brown urine and urinalysis positive for blood resulting from the presence of myoglobinuria without microscopic
evidence of red blood cells. A complication of McArdle disease is acute kidney injury and acute tubular necrosis secondary to the
release of nephrotoxic myoglobin and heme pigments in rhabdomyolysis. Treatment involves avoiding low-carbohydrate diets and
improving cardiovascular fitness with regular, mild aerobic exercise.

Incorrect Answers: A, B, D, and E.

Alcoholic rhabdomyolysis (Choice A) refers to muscle breakdown induced by excessive alcohol consumption. Rhabdomyolysis is
characterized by a triad of muscle pain, weakness, and dark urine. It may also be brought on by episodes of strenuous exercise, though
jogging a short distance would not be expected to cause it.

Hypokalemic periodic paralysis (Choice B) is characterized by episodes of painless muscle weakness brought on by heavy exercise.
Episodes occur in association with low serum potassium concentration. Exercise intolerance with muscle cramping and myoglobinuria
is more suggestive of a metabolic myopathy, like McArdle disease.

Myasthenic (Lambert-Eaton) syndrome (Choice D) is a paraneoplastic syndrome often caused by small cell carcinoma of the lung.
Presynaptic calcium channel antibody production causes the proximal extremity weakness. A characteristic physical examination
feature is the augmentation of strength with repetitive action, which is caused by progressive increases in calcium concentration. ....

~, https://t.me/USMLENBME2CK
~ ~ r,
Next Score Report Lab Values Calculator Help Pause
Exam Section: Item 1 of 50 National Board of Medical Examiners
■ Mark Clinical Neurology Self-Assessment

E) Polymyositis ...
Correct Answer: C.

Muscle phosphorylase deficiency (McArdle disease) is a glycogen storage disease that affects the muscles. It is characterized by the
inability of myocytes to break down glycogen as a result of the absence of muscular glycogen phosphorylase, which causes exercise
intolerance with fatigue, cramping, and muscle weakness. Muscle cramping may resolve with rest or be permanent. Occasionally, a
"second wind" phenomenon may be seen when blood flow to the muscles increases after approximately 10 minutes of exercise,
delivering fatty acids as fuel and relieving the need for glycogen breakdown. Patients may also have episodes of rhabdomyolysis,
accompanied by red or brown urine and urinalysis positive for blood resulting from the presence of myoglobinuria without microscopic
evidence of red blood cells. A complication of McArdle disease is acute kidney injury and acute tubular necrosis secondary to the
release of nephrotoxic myoglobin and heme pigments in rhabdomyolysis. Treatment involves avoiding low-carbohydrate diets and
improving cardiovascular fitness with regular, mild aerobic exercise.

Incorrect Answers: A, B, D, and E.

Alcoholic rhabdomyolysis (Choice A) refers to muscle breakdown induced by excessive alcohol consumption. Rhabdomyolysis is
characterized by a triad of muscle pain, weakness, and dark urine. It may also be brought on by episodes of strenuous exercise, though
jogging a short distance would not be expected to cause it.

Hypokalemic periodic paralysis (Choice B) is characterized by episodes of painless muscle weakness brought on by heavy exercise.
Episodes occur in association with low serum potassium concentration. Exercise intolerance with muscle cramping and myoglobinuria
is more suggestive of a metabolic myopathy, like McArdle disease.

Myasthenic (Lambert-Eaton) syndrome (Choice D) is a paraneoplastic syndrome often caused by small cell carcinoma of the lung.
Presynaptic calcium channel antibody production causes the proximal extremity weakness. A characteristic physical examination
feature is the augmentation of strength with repetitive action, which is caused by progressive increases in calcium concentration.

Polymyositis (Choice E) is an autoimmune condition affecting the proximal muscles. It is closely related to dermatomyositis, however,
unlike dermatomyositis, it does not present with cutaneous findings. The muscular inflammation is characterized by an endomysial
CD8+ T-lymphocyte inflammatory infiltrate causing damage to the myocytes. The weakness is not related to exercise and does not
resolve with rest.

Educational Objective: Muscle phosphorylase deficiency (McArdle disease) is a glycogen storage disease in which myocytes are
unable to break down glycogen because of the absence of muscular glycogen phosphorylase. Fatigue, cramping, and muscle
weakness are seen with exercise. It may be complicated by rhabdomyolysis and acute kidney injury.

.,.

~, https://t.me/USMLENBME2CK
~ ~ r,
Next Score Report Lab Values Calculator Help Pause
Exam Section: Item 2 of 50 National Board of Medical Examiners
■ Mark Clinical Neurology Self-Assessment

2. A 77-year-old woman with mild dementia, Alzheimer type, is brought to the emergency department by her daughter 3 hours after the sudden onset
of fearfulness and combativeness. The patient lives with her daughter and son-in-law. She attempted to smash their television screen with her
walker and struck her son-in-law when he tried to stop her. She has hypertension, gastroesophageal reflux disease, and diverticulosis. Her
medications are metoprolol, omeprazole, triamterene-hydrochlorothiazide, and donepezil. On arrival, she is hostile and accuses the physician of
trying to steal her money. Her temperature is 37.5°C (99.5°F), pulse is 74/min, respirations are 16/min, and blood pressure is 144/82 mm Hg.
Physical examination shows restless movements. Mental status examination initially shows hypervigilance; the patient later appears drowsy. She
mutters under her breath as if responding to internal stimuli and picks at the air. Her Mini-Mental State Examination score is 10/30. Results of which
of the following diagnostic studies are most likely to be abnormal in th is patient?

A) Liver fu nction tests


B) Measurement of serum vitamin B 12 (cobalamin) concentration
C) Serum toxicology screening
D) Thyroid fu nction tests
E) Urinalysis
Correct Answer: E.

Urinalysis is likely to be abnormal in this patient with delirium, which is defined by an acute-onset alteration of consciousness and
cognition, with symptoms that wax and wane throughout the day. In elderly patients, especially those with underlying dementia,
systemic infection can rapidly lead to the development of delirium. Other common causes of delirium include intoxication and adverse
effects from medications such as anticholinergics, benzodiazepines, or opiates. Al l patients with delirium should be evaluated for
evidence of systemic infection with urinalysis and culture, chest x-ray, and occasional ly with blood cultures, if indicated. Treatment of
delirium should focus on treatment of the underlying infection if one is detected, frequent reorientation, maintenance of normal sleep-
wake cycles, and placement of the patient in a calm and soothing environment.

Incorrect Answers: A, B, C, and D.

Liver function tests (Choice A) may be abnormal in patients with acute liver failure or patients with cirrhosis who present with altered
mental status. This patient has no other evidence of either acute or chronic liver failure on examination; evaluation for an underlying
infection is more appropriate.

Measurement of serum vitamin B 12 (cobalamin) concentration (Choice B) is unlikely to be helpful. Decreased concentrations of vitamin
B 12 (cobalamin) can result in peripheral neuropathy and impaired proprioception from subacute combined degeneration of the dorsal
and lateral columns of the spinal cord. Patients also frequently demonstrate a macrocytic anemia.
....
- -- - -- -- -- --- -- -- -- - -- -- -- - - -- - - - -- - - - -- - - - -- -- - - ---- - - - -- - - - - - - - - - - - -
r ~, ~ ~ r,
Previous Next Score Report
https://t.me/USMLENBME2CK Lab Values Calculator Help Pause
Exam Section: Item 2 of 50 National Board of Medical Examiners
■ Mark Clinical Neurology Self-Assessment
- - ...
D) Thyroid function tests
E) Urinalysis
Correct Answer: E.

Urinalysis is likely to be abnormal in this patient with delirium, which is defined by an acute-onset alteration of consciousness and
cognition, with symptoms that wax and wane throughout the day. In elderly patients, especially those with underlying dementia,
systemic infection can rapidly lead to the development of delirium. Other common causes of delirium include intoxication and adverse
effects from medications such as anticholinergics, benzodiazepines, or opiates. Al l patients with delirium should be evaluated for
evidence of systemic infection with urinalysis and culture, chest x-ray, and occasional ly with blood cultures, if indicated. Treatment of
delirium should focus on treatment of the underlying infection if one is detected, frequent reorientation, maintenance of normal sleep-
wake cycles, and placement of the patient in a calm and soothing environment.

Incorrect Answers: A, B, C, and D.

Liver function tests (Choice A) may be abnormal in patients with acute liver failure or patients with cirrhosis who present with altered
mental status. This patient has no other evidence of either acute or chronic liver failure on examination; evaluation for an underlying
infection is more appropriate.

Measurement of serum vitamin B 12 (cobalamin) concentration (Choice B) is unlikely to be helpful. Decreased concentrations of vitamin
B 12 (cobalamin) can result in peripheral neuropathy and impaired proprioception from subacute combined degeneration of the dorsal
and lateral columns of the spinal cord. Patients also frequently demonstrate a macrocytic anemia.

Serum toxicology screening (Choice C) is frequently performed in the evaluation of delirium when a good history cannot be obtained or
when there are other signs of intoxication. This patient's caregivers did not report a recent history of substance ingestion, so a
toxicology screen is unlikely to be helpful.

Thyroid function tests (Choice D) are unlikely to be helpful. Patients with altered mental status from hyperthyroidism often have
concomitant findings of heat intolerance, sweating, tachycardia, and hypertension, whi le patients with severe hypothyroidism present
with reduced levels of consciousness, not hyperactive delirium.

Educational Objective: Delirium is a common finding in elderly patients with underlying dementia who have an underlying systemic
infection. Delirium is characterized by an acute-onset alteration in cognition and consciousness that fluctuates over time. Evaluation
should focus on identifying the presence of any underlying infections as well as asking caregivers about recent ingestions or medication
changes.

.,.

r ~, ~ ~ r,
Previous Next Score Report
https://t.me/USMLENBME2CK Lab Values Calculator Help Pause
Exam Section: Item 3 of 50 National Board of Medical Examiners
■ Mark Clinical Neurology Self-Assessment

3. A 77-year-old woman is brought to the physician by her son because she recently has become unable to take care of herself or walk independently.
Her son says that her ability to function has declined during the past 4 months, but she has not had any specific complaints. She has hypertension
treated with atenolol and enalapril. Her only other medication is 81-mg aspirin. Six years ago, she had a myocardial infarction. On examination, her
speech is confused. Her pulse is 72/min, and blood pressure is 145/86 mm Hg. Exam ination shows diffuse muscle weakness. Gait testi ng shows an
inability to ambulate. She can recall one of three objects after 2 minutes. Results of laboratory studies are within the reference ranges. An MRI of
the brain shows multiple small infarctions throughout both cortices and a few in the thalamus. In addition to adjustments to her antihypertensive
medications, administration of which of the following is the most appropriate management?

A) Enoxaparin
B) Hepari n followed by warfarin
C) Vitamin E
D) Warfarin only
E) No additional medication is indicated
Correct Answer: E.

Multi-infarct (vascular) dementia refers to cognitive impairment that results from acute cerebral infarctions secondary to underlying
cerebrovascular or thromboembolic disease. Patients with vascular dementia typically possess a history of clinical strokes with
consequent focal neurologic deficits and a stepwise decline in function. Some patients alternatively demonstrate si lent cerebrovascular
disease, characterized by progressive neurologic decline and mu ltiple small infarcts on neuroimaging, as in th is patient. This patient
possesses risk factors for cerebrovascular disease such as hypertension and atherosclerotic disease (as evidenced by a prior
myocardial infarction). Diagnosis requires evidence of cerebrovascular disease (eg, clinical history of stroke or neuroimaging evidence
of infarct) and cognitive impairment. Management requ ires secondary prevention of stroke via blood pressure control. In al l patients
with a stroke history and many patients with si lent cerebrovascular disease, antiplatelet therapy with aspirin is required. However, th is
patient is already on aspirin for the secondary prevention of myocardial infarction, thus no additional medication is indicated.

Incorrect Answers: A, B, C, and D.

Long-term antithrombotic therapy in the form of enoxaparin (Choice A), heparin fol lowed by warfarin (Choice B), and warfarin on ly
(Choice D) would be appropriate after acute ischemic stroke related to atrial fibril lation, left ventricular thrombus, or valve replacement.
This patient does not possess a history of acute ischemic stroke (eg, an abrupt neurologic change) but instead presents with silent
cerebrovascular disease, and she does not demonstrate evidence of cardiac indications for chronic anticoagulation.

Supplementation of vitamin E (Choice C), an antioxidant, has not proven effective for the primary or secondary prevention of
cardiovascular or cerebrovascular disease. ....

r ~, ~ ~ r,
Previous Next Score Report
https://t.me/USMLENBME2CK Lab Values Calculator Help Pause
Exam Section: Item 3 of 50 National Board of Medical Examiners
■ Mark Clinical Neurology Self-Assessment
-- . - - .. . .. . -- - -- - . . - --- - - . .. - ,. -- . -
the brain shows multiple smal l infarctions throughout both cortices and a few in the thalamus. In addition to adj ustments to her antihypertensive
...
med ications, admin istration of wh ich of the following is the most appropriate management?

A) Enoxaparin
B) Hepari n followed by warfarin
C) Vitam in E
D) Warfarin only
E) No additional medication is indicated
Correct Answer: E.

Multi-infarct (vascular) dementia refers to cognitive impairment that results from acute cerebral infarctions secondary to underlying
cerebrovascular or thromboembolic disease. Patients with vascular dementia typically possess a history of clinical strokes with
consequent focal neurologic deficits and a stepwise decline in function. Some patients alternatively demonstrate si lent cerebrovascular
disease, characterized by progressive neurologic decline and mu ltiple small infarcts on neuroimaging, as in this patient. This patient
possesses risk factors for cerebrovascular disease such as hypertension and atherosclerotic disease (as evidenced by a prior
myocardial infarction). Diagnosis requires evidence of cerebrovascular disease (eg, clinical history of stroke or neuroimaging evidence
of infarct) and cognitive impairment. Management requires secondary prevention of stroke via blood pressure control. In al l patients
with a stroke history and many patients with si lent cerebrovascular disease, antiplatelet therapy with aspirin is required. However, this
patient is already on aspirin for the secondary prevention of myocardial infarction, thus no additional medication is indicated.

Incorrect Answers: A, B, C, and D.

Long-term antithrombotic therapy in the form of enoxaparin (Choice A), heparin fol lowed by warfarin (Choice B), and warfarin on ly
(Choice D) would be appropriate after acute ischemic stroke related to atrial fibril lation, left ventricular thrombus, or valve replacement.
This patient does not possess a history of acute ischemic stroke (eg, an abrupt neurologic change) but instead presents with silent
cerebrovascular disease, and she does not demonstrate evidence of cardiac indications for chronic anticoagulation.

Supplementation of vitamin E (Choice C), an antioxidant, has not proven effective for the primary or secondary prevention of
cardiovascular or cerebrovascular disease.

Educational Objective: Vascular dementia refers to cognitive impairment that results from a history of stroke or silent cerebrovascu lar
disease (small infarcts on neuroimaging without a history of acute neurologic symptoms). Patients typically present with stepwise
cognitive decline and focal neurologic deficits, though patients with silent cerebrovascular disease may instead present with
progressive neurologic decline. Management requires secondary prevention with antihypertensive and antiplatelet therapy.

.,.

r ~, ~ ~ r,
Previous Next Score Report
https://t.me/USMLENBME2CK Lab Values Calculator Help Pause
Exam Section: Item 4 of 50 National Board of Medical Examiners
■ Mark Clinical Neurology Self-Assessment

The response options for the next 2 items are the same. Select one answer for each item in the set.

For each patient with a visual disturbance, select the most likely site of the lesion.

A) Cerebellum
B) Cranial nerve 11
C) Cranial nerve 11 1
D) Cranial nerve IV
E) Cranial nerve VI
F) Extraocular muscles
G) Medial long itudinal fascicu lus
H) Neuromuscular junction
I) Retina

4. A 62-year-old woman is brought to the emergency department 3 hours after the onset of double vision and left eye pain that she first noted when
she awoke. She describes a horizontal separation of images, which is worse for distant as opposed to near vision. She has an 18-year history of
type 2 diabetes mellitus. Examination of the left eye shows mild adduction on primary gaze with severe weakness of abduction with conjugate gaze
to the left. The remainder of the neurologic examination shows no abnormalities.
Correct Answer: E.

Microvascular damage resulting from diabetes mel litus can produce neuropathy, such as distal polyneuropathy, radiculopathy or, as in this case,
cranial nerve palsy. Common ly affected nerves include the abducens nerve (cranial nerve VI), which presents with esotropia, unilateral abduction
deficit, and diplopia in horizontal gaze, and the oculomotor nerve (cranial nerve Ill), which presents with unilateral ptosis, exotropia, and hypotropia.
Patients may experience associated ocular or periocular pain. It is important to exclude other possible causes of cranial nerve palsy, such as stroke,
increased intracranial pressure, vascu litis, or optic neuropathy. Most patients with microvascular abducens nerve palsy experience resolution of
symptoms over several months fol lowing treatment of the underlying cause.

Incorrect Answers: A, B, C, D, F, G, H, and I.

The cerebellum (Choice A) contains a variety of structures, including the vermis, floccu lus, and paraflocculus, which are important for the
coordination of ocular saccades, pursuit movements, and gaze holding. Cerebellar lesions do not produce esotropia. ....

r ~, ~ ~ r,
Previous Next Score Report
https://t.me/USMLENBME2CK Lab Values Calculator Help Pause
Exam Section: Item 4 of 50 National Board of Medical Examiners
■ Mark Clinical Neurology Self-Assessment Please Walt
...
Microvascular damage resulting from diabetes mel litus can produce neuropathy, such as distal polyneuropathy, radiculopathy or, as in this case,
cranial nerve palsy. Common ly affected nerves include the abducens nerve (cranial nerve VI), which presents with esotropia, unilateral abduction
deficit, and diplopia in horizontal gaze, and the oculomotor nerve (cranial nerve Ill), which presents with unilateral ptosis, exotropia, and hypotropia.
Patients may experience associated ocular or periocular pain. It is important to exclude other possible causes of cranial nerve palsy, such as stroke,
increased intracranial pressure, vascu litis, or optic neuropathy. Most patients with microvascular abducens nerve palsy experience resolution of
symptoms over several months fol lowing treatment of the underlying cause.

Incorrect Answers: A, B, C, D, F, G, H, and I.

The cerebellum (Choice A) contains a variety of structures, including the vermis, floccu lus, and paraflocculus, which are important for the
coordination of ocular saccades, pursuit movements, and gaze holding. Cerebellar lesions do not produce esotropia.

Cranial nerve II (Choice B), the optic nerve, carries visual sensory information from the retina to the primary visual cortex via the lateral geniculate
nucleus. Optic nerve defects can be caused by demyelination or trauma and present with monocular vision loss.

Cranial nerve Ill (Choice C), the oculomotor nerve, is responsible for motor innervation of the levator palpebrae superioris, the superior, medial, and
inferior rectus muscles, as well as the inferior oblique muscle. Deficits of the oculomotor nerve present with ptosis, exotropia, and hypotropia.

Cranial nerve IV (Choice D), the troch lear nerve, is responsible for motor innervation of the superior oblique muscle. Deficits of the troch lear nerve
present with diplopia, hypertropia, and compensatory head ti lt.

Extraocular muscles (Choice F) may cause diplopia in thyroid ophthalmopathy, orbital mass lesions, or entrapment within a fractured orbital bone.
This patient's history of diabetes mellitus suggests microvascular ischemia as a more likely cause of her diplopia.

Lesions of the medial longitudinal fascicu lus (MLF) (Choice G) cause internuclear ophthalmoplegia (INO), which presents with an ipsilateral
adduction deficit, characterized by an inability of the affected eye to cross mid line, and an associated contralateral abduction saccade (as the
contralateral eye undergoes more rapid nerve firing to attempt to pu ll the ipsilateral eye across the midline).

Defects of the neuromuscular junction (Choice H) occur in myasthenia gravis, which presents with diplopia and ptosis that worsens towards the end
of the day.

Defects of the retina (Choice I) may produce visual disturbances but do not cause acute onset diplopia or deficits of extraocular movements.

Educational Objective: Diabetes mellitus causes microvascular damage to a variety of tissues, including nerves, and can cause cranial nerve palsy.
Microvascular abducens nerve palsy presents with esotropia, unilateral abduction deficit, and diplopia in horizontal gaze. Most patients experience
spontaneous recovery over several months following treatment of the underlying cause.
....

r ~, ~ ~ r,
Previous Next Score Report
https://t.me/USMLENBME2CK Lab Values Calculator Help Pause
Exam Section: Item 5 of 50 National Board of Medical Examiners
■ Mark Clinical Neurology Self-Assessment

For each patient with a visual disturbance, select the most likely site of the lesion.

A) Cerebellum
B) Cranial nerve 11
C) Cranial nerve 11 1
D) Cranial nerve IV
E) Cranial nerve VI
F) Extraocu lar muscles
G) Medial longitudinal fa sciculus
H) Neuromuscular junction
I) Reti na

5. A 35-year-old man comes to the physician 1 day after the sudden onset of double vision. He describes seeing two objects side by side that are
farthest apart when he looks to the ri ght. Exam ination shows the eyes move normally w hen looking to the left. When looking to the ri ght, the left eye
does not fully adduct, and there is nystagmus in the abducting eye. T he remainder of the neurologic examination shows no abnormalities.
Correct Answer: G.

The medial longitudinal fascicu lus (MLF) contains interneurons that al low coordination of the oculomotor nucleus with the contralateral paramedian
pontine reticular formation and abducens nucleus. During horizontal eye movements, abduction requires close coordination between the abducens
and oculomotor nuclei. For example, during left gaze, impulses from cortical centers travel through the left paramedian pontine reticular formation to
the left abducens nucleus, resu lting in abduction of the left eye. Conjugate gaze requires symmetric adduction of the right eye via the right
ocu lomotor nucleus. Lesions of the MLF cause internuclear ophthalmoplegia (INO), which presents with an ipsilateral adduction deficit,
characterized by an inability of the affected eye to cross midline, and an associated contralateral abduction saccade (as the contralateral eye
undergoes more rapid neural stimu lation in an attempt to pull the ipsilateral eye across the mid line). INO may be unilateral or bilateral. In younger
patients, such as in this patient, demyelination associated with multiple sclerosis is the most common cause, whereas ischemic stroke is the most
common cause in older patients.

Incorrect Answers: A, B, C, D, E, F, H, and I.

The cerebellum (Choice A) contains a variety of structures, including the vermis, floccu lus, and paraflocculus, which are important for the
coordination of ocular saccades, pursuit movements, and gaze holding. Cerebellar lesions do not produce INO.
....

r ~, ~ ~ r,
Previous Next Score Report
https://t.me/USMLENBME2CK Lab Values Calculator Help Pause
Exam Section: Item 5 of 50 National Board of Medical Examiners
■ Mark Clinical Neurology Self-Assessment

Correct Answer: G. ...

The medial longitudinal fasciculus (MLF) contains interneurons that al low coordination of the oculomotor nucleus with the contralateral paramedian
pontine reticular formation and abducens nucleus. During horizontal eye movements, abduction requires close coordination between the abducens
and oculomotor nuclei. For example, during left gaze, impulses from cortical centers travel through the left paramedian pontine reticular formation to
the left abducens nucleus, resulting in abduction of the left eye. Conjugate gaze requires symmetric adduction of the right eye via the right
oculomotor nucleus. Lesions of the MLF cause internuclear ophthalmoplegia (INO), which presents with an ipsilateral adduction deficit,
characterized by an inability of the affected eye to cross midline, and an associated contralateral abduction saccade (as the contralateral eye
undergoes more rapid neural stimu lation in an attempt to pull the ipsilateral eye across the midline). INO may be unilateral or bilateral. In younger
patients, such as in this patient, demyelination associated with multiple sclerosis is the most common cause, whereas ischemic stroke is the most
common cause in older patients.

Incorrect Answers: A, B, C, D, E, F, H, and I.

The cerebellum (Choice A) contains a variety of structures, including the vermis, flocculus, and paraflocculus, which are important for the
coordination of ocular saccades, pursuit movements, and gaze holding. Cerebellar lesions do not produce INO.

Cranial nerve II (Choice B), the optic nerve, carries visual sensory information from the retina to the primary visual cortex via the lateral geniculate
nucleus. Optic nerve defects can be caused by demyelination or trauma and present with monocular vision loss.

Cranial nerve Ill (Choice C), the oculomotor nerve, is responsible for motor innervation of the levator palpebrae superioris, the superior, medial, and
inferior rectus muscles, as well as the inferior oblique muscle. Deficits of the oculomotor nerve present with ptosis, exotropia, and hypotropia.

Cranial nerve IV (Choice D), the trochlear nerve, is responsible for motor innervation of the superior oblique muscle. Deficits of the trochlear nerve
present with diplopia, hypertropia, and compensatory head tilt.

Cranial nerve VI (Choice E), the abducens nerve, is responsible for motor innervation of the lateral rectus muscle. Deficits of the abducens nerve
present with esotropia and diplopia in horizontal gaze. Adduction is not affected by abducens nerve palsy.

Extraocular muscles (Choice F) may cause diplopia in thyroid ophthalmopathy, orbital mass lesions, or entrapment within a fractured orbital bone.
This patient lacks suggestive signs or risk factors for these etiologies, suggesting INO as a more likely cause of his diplopia.

Defects of the neuromuscular junction (Choice H) occur in myasthenia gravis, which presents with diplopia and ptosis that worsens towards the end
of the day.

Defects of the retina (Choice I) may produce visual disturbances but do not cause acute onset diplopia or deficits of extraocular movements.

Educational Objective: The medial longitudinal fasciculus (MLF) contains interneurons that allow coordinated, conjugate, horizontal gaze ....

r ~, ~ ~ r,
Previous Next Score Report
https://t.me/USMLENBME2CK Lab Values Calculator Help Pause
Exam Section: Item 5 of 50 National Board of Medical Examiners
■ Mark Clinical Neurology Self-Assessment
pontine reticular formation and abducens nucleus. During horizontal eye movements, abduction requires close coordination between the abducens ...
and oculomotor nuclei. For example, during left gaze, impulses from cortical centers travel through the left paramedian pontine reticular formation to
the left abducens nucleus, resulting in abduction of the left eye. Conjugate gaze requires symmetric adduction of the right eye via the right
oculomotor nucleus. Lesions of the MLF cause internuclear ophthalmoplegia (INO), which presents with an ipsilateral adduction deficit,
characterized by an inability of the affected eye to cross midline, and an associated contralateral abduction saccade (as the contralateral eye
undergoes more rapid neural stimulation in an attempt to pull the ipsilateral eye across the midline). INO may be unilateral or bilateral. In younger
patients, such as in this patient, demyelination associated with multiple sclerosis is the most common cause, whereas ischemic stroke is the most
common cause in older patients.

Incorrect Answers: A, B, C, D, E, F, H, and I.

The cerebellum (Choice A) contains a variety of structures, including the vermis, flocculus, and paraflocculus, which are important for the
coordination of ocular saccades, pursuit movements, and gaze holding. Cerebellar lesions do not produce INO.

Cranial nerve II (Choice B), the optic nerve, carries visual sensory information from the retina to the primary visual cortex via the lateral geniculate
nucleus. Optic nerve defects can be caused by demyelination or trauma and present with monocular vision loss.

Cranial nerve Ill (Choice C), the oculomotor nerve, is responsible for motor innervation of the levator palpebrae superioris, the superior, medial, and
inferior rectus muscles, as well as the inferior oblique muscle. Deficits of the oculomotor nerve present with ptosis, exotropia, and hypotropia.

Cranial nerve IV (Choice D), the trochlear nerve, is responsible for motor innervation of the superior oblique muscle. Deficits of the trochlear nerve
present with diplopia, hypertropia, and compensatory head tilt.

Cranial nerve VI (Choice E), the abducens nerve, is responsible for motor innervation of the lateral rectus muscle. Deficits of the abducens nerve
present with esotropia and diplopia in horizontal gaze. Adduction is not affected by abducens nerve palsy.

Extraocu lar muscles (Choice F) may cause diplopia in thyroid ophthalmopathy, orbital mass lesions, or entrapment within a fractured orbital bone.
This patient lacks suggestive signs or risk factors for these etiologies, suggesting INO as a more likely cause of his diplopia.

Defects of the neuromuscular junction (Choice H) occur in myasthenia gravis, which presents with diplopia and ptosis that worsens towards the end
of the day.

Defects of the retina (Choice I) may produce visual disturbances but do not cause acute onset diplopia or deficits of extraocular movements.

Educational Objective: The medial longitudinal fasciculus (MLF) contains interneurons that allow coordinated, conjugate, horizontal gaze
movements by connecting the oculomotor nucleus with the contralateral paramedian pontine reticular formation and abducens nucleus. Lesions of
the MLF cause internuclear ophthalmoplegia (INO), which presents with an ipsilateral adduction deficit, characterized by an inability of the affected
eye to cross midline, and an associated contralateral abduction saccade. ....

r ~, ~ ~ r,
Previous Next Score Report
https://t.me/USMLENBME2CK Lab Values Calculator Help Pause
Exam Section: Item 6 of 50 National Board of Medical Examiners
■ Mark Clinical Neurology Self-Assessment

6. A 47-year-old woman with chronic alcoholism is brought to the emergency department acutely intoxicated. Other than her intoxication, physical and
neurologic examinations show no abnormalities. Which of the following complications is most likely to be prevented by immediate administration of
vitamin B 1 (thiamine)?

A) Acute alcoholic myopathy


B) Anterograde amnesi
C) Burning paresthesias of the feet
D) Hallucinations
E) Seizures
Correct Answer: B.

Patients who chronical ly use alcohol typical ly demonstrate a significant vitamin B 1 (thiamine) deficiency caused by poor nutritional
intake, thiamine malabsorption, and impaired cellular utilization of thiamine. Over time, thiamine deficiency can result in damage and
atrophy of the mammillary bodies, visible on MRI of the brain, which manifests as Wernicke encephalopathy. To prevent Wernicke
encephalopathy, all patients with sustained heavy alcohol use should be given oral thiamine supplementation, and patients presenting
with acute intoxication should generally be given parenteral thiamine. The typical clinical presentation of Wernicke encephalopathy is a
triad of acutely altered mental status, ophthalmoplegia (along with nystagmus), and ataxia. The symptoms of Wernicke encephalopathy
are reversible but increase the risk for developing Wernicke-Korsakoff syndrome in patients with alcohol use disorder if not immediately
treated with high-dose parenteral thiamine. Wernicke-Korsakoff syndrome is an irreversible condition that features psychosis,
anterograde and retrograde amnesia, and the consequent tendency to confabulate, or verbalize false stories or memories, to
compensate for the inability to remember.

Incorrect Answers: A, C, D, and E.

Acute alcoholic myopathy (Choice A) is a complication of chronic alcohol use (especially in binge-pattern use) that likely results from
alcohol's effect on the permeability of the skeletal muscle cell membrane, manifesting as muscle pain and swel ling. Thiamine
supplementation does not decrease the risk for acute alcoholic myopathy.

Burning paresthesias of the feet (Choice C) may occur as a manifestation of subacute combined degeneration, resu lting from vitamin
B 12 (cobalamin) deficiency, or from alcoholic neuropathy. Though patients who use alcohol chronically may have vitamin B 12 deficiency
caused by malnutrition, the administration of vitamin B 1 (thiamine) would not prevent complications of vitamin B 12 deficiency.

Hallucinations (Choice D) may occur in alcohol withdrawal as a manifestation of alcoholic hallucinosis or delirium tremens.
Hallucinations are not a typical feature of Wernicke-Korsakoff syndrome and would not be prevented by thiamine administration. ....

r ~, ~ ~ r,
Previous Next Score Report
https://t.me/USMLENBME2CK Lab Values Calculator Help Pause
Exam Section: Item 6 of 50 National Board of Medical Examiners
■ Mark Clinical Neurology Self-Assessment

E) Seizures
...
Correct Answer: B.

Patients who chronical ly use alcohol typical ly demonstrate a significant vitamin B 1 (thiamine) deficiency caused by poor nutritional
intake, thiamine malabsorption, and impaired cellular utilization of thiamine. Over time, thiamine deficiency can result in damage and
atrophy of the mammillary bodies, visible on MRI of the brain, which manifests as Wernicke encephalopathy. To prevent Wernicke
encephalopathy, all patients with sustained heavy alcohol use should be given oral thiamine supplementation, and patients presenting
with acute intoxication should generally be given parenteral thiamine. The typical clinical presentation of Wernicke encephalopathy is a
triad of acutely altered mental status, ophthalmoplegia (along with nystagmus), and ataxia. The symptoms of Wernicke encephalopathy
are reversible but increase the risk for developing Wernicke-Korsakoff syndrome in patients with alcohol use disorder if not immediately
treated with high-dose parenteral thiamine. Wernicke-Korsakoff syndrome is an irreversible condition that features psychosis,
anterograde and retrograde amnesia, and the consequent tendency to confabulate, or verbalize false stories or memories, to
compensate for the inability to remember.

Incorrect Answers: A, C, D, and E.

Acute alcoholic myopathy (Choice A) is a complication of chronic alcohol use (especially in binge-pattern use) that likely results from
alcohol's effect on the permeability of the skeletal muscle cell membrane, manifesting as muscle pain and swel ling. Thiamine
supplementation does not decrease the risk for acute alcoholic myopathy.

Burning paresthesias of the feet (Choice C) may occur as a manifestation of subacute combined degeneration, resu lting from vitamin
B 12 (cobalamin) deficiency, or from alcoholic neuropathy. Though patients who use alcohol chronically may have vitamin B 12 deficiency
caused by malnutrition, the administration of vitamin B 1 (thiamine) would not prevent complications of vitamin B 12 deficiency.

Hallucinations (Choice D) may occur in alcohol withdrawal as a manifestation of alcoholic hallucinosis or delirium tremens.
Hallucinations are not a typical feature of Wernicke-Korsakoff syndrome and would not be prevented by thiamine administration.

Seizures (Choices E) may occur in alcohol withdrawal. Thiamine does not prevent alcohol withdrawal-induced seizures.

Educational Objective: Chronic alcohol use and associated malnutrition may result in significant thiamine deficiency, which can cause
Wernicke-Korsakoff syndrome. The syndrome is characterized by altered mental status, ophthalmoplegia, ataxia, psychosis, and
anterograde and retrograde amnesia. Thiamine supplementation can prevent the development of Wernicke-Korsakoff syndrome in
patients with alcohol use disorder.

.,.

r ~, ~ ~ r,
Previous Next Score Report
https://t.me/USMLENBME2CK Lab Values Calculator Help Pause
Exam Section: Item 7 of 50 National Board of Medical Examiners
■ Mark Clinical Neurology Self-Assessment

7. An 82-year-old woman has had a chron ic bitemporal headache and diffuse achiness of the shou lders for 6 months. She has no history of serious
illness and takes no med ications. Her temperature is 37°C (98.6°F), pu lse is 80/min, respirations are 12/min, and blood pressure is 150/85 mm Hg.
There is mild deformity of the distal interphalangeal joints; range of motion is full, and there is no edema or erythema of the joints. Exam ination
shows normal fund i. Neurologic exam ination shows no focal findings. Erythrocyte sed imentation rate is 120 mm/h. Which of the following is the
most likely diagnosis?

A) Ankylosing spondylitis
B) Fibromyositis
C) Polymyalgia rheumatica
D) Polymyositis
E) Rheumatoid arthritis
Correct Answer: C.

Polymyalgia rheumatica (PMR) is the most likely diagnosis in this elderly woman with myalgia, bitemporal headache, and markedly
increased erythrocyte sedimentation rate (ESR). PMR is an inflammatory disorder that often occurs concurrently with giant cel l arteritis
(GCA), a large vessel vasculitis. Symptoms include proximal muscle and joint (eg, shoulder and hip girdle) pain and stiffness. If
untreated, muscle weakness may occur from disuse atrophy. Systemic symptoms such as fatigue are commonly seen. The diagnosis is
clinical, and treatment is with low-dose prednisone for up to a year. It is common for patients to respond to prednisone almost
immediately, thus the absence of response should prompt consideration of a different diagnosis. As GCA occurs concurrently in up to
20% of patients who have PMR, symptoms of throbbing temporal headache, vision changes, or jaw claudication should prompt
temporal artery biopsy and initiation of high-dose prednisone.

Incorrect Answers: A, B, D, and E.

Ankylosing spondylitis (Choice A) is a seronegative inflammatory arthritis that most commonly affects younger men with a history of
chronic low back and buttock pain that is worse in the morning and improves with exercise. Concurrent findings may also include
uveitis, increased inflammatory markers, and positive HLA-B27. Pelvic x-rays may show sacroiliitis, although MRI is more sensitive.

Fibromyositis (Choice B), commonly known as fibromyalgia, is considered a central pain processing disorder. Patients are usually
younger women who present with chronic fatigue, exercise intolerance, and diffuse or multifocal muscle tenderness in the absence of
abnormal laboratory findings. This is an unlikely diagnosis in this patient.

Polymyositis (Choice D) is an inflammatory myopathy that can also present with proximal muscle pain, but it is frequently accompanied
by weakness. Inflammatory markers may be increased but are nonspecific. Serum creatinine kinase may also be increased and can ....
, ~, -- ---- - - - ------ \j ------- - -- -- - -- - --------

~ ~ r,
Previous Next Score Report
https://t.me/USMLENBME2CK Lab Values Calculator Help Pause
Exam Section: Item 7 of 50 National Board of Medical Examiners
■ Mark Clinical Neurology Self-Assessment
...
C) Polymyalgia rheumatica
D) Polymyositis
E) Rheumatoid arthritis
Correct Answer: C.

Polymyalgia rheumatica (PMR) is the most likely diagnosis in this elderly woman with myalgia, bitemporal headache, and markedly
increased erythrocyte sedimentation rate (ESR). PMR is an inflammatory disorder that often occurs concurrently with giant cell arteritis
(GCA), a large vessel vasculitis. Symptoms include proximal muscle and joint (eg, shoulder and hip girdle) pain and stiffness. If
untreated, muscle weakness may occur from disuse atrophy. Systemic symptoms such as fatigue are commonly seen. The diagnosis is
clinical, and treatment is with low-dose prednisone for up to a year. It is common for patients to respond to prednisone almost
immediately, thus the absence of response should prompt consideration of a different diagnosis. As GCA occurs concurrently in up to
20% of patients who have PMR, symptoms of throbbing temporal headache, vision changes, or jaw claudication should prompt
temporal artery biopsy and initiation of high-dose prednisone.

Incorrect Answers: A, B, D, and E.

Ankylosing spondylitis (Choice A) is a seronegative inflammatory arthritis that most commonly affects younger men with a history of
chronic low back and buttock pain that is worse in the morning and improves with exercise. Concurrent findings may also include
uveitis, increased inflammatory markers, and positive HLA-B27. Pelvic x-rays may show sacroiliitis, although MRI is more sensitive.

Fibromyositis (Choice B), commonly known as fibromyalgia, is considered a central pain processing disorder. Patients are usually
younger women who present with chronic fatigue, exercise intolerance, and diffuse or multifocal muscle tenderness in the absence of
abnormal laboratory findings. This is an unlikely diagnosis in this patient.

Polymyositis (Choice D) is an inflammatory myopathy that can also present with proximal muscle pain, but it is frequently accompanied
by weakness. Inflammatory markers may be increased but are nonspecific. Serum creatinine kinase may also be increased and can
provide a clue to diagnosis. Muscle biopsy is required for definitive diagnosis.

Rheumatoid arthritis (RA) (Choice E) commonly presents with symmetric arthritis of the metacarpophalangeal and wrist joints in
younger patients, but may also present with arthritis of the shoulder, elbow, ankles, and proximal interphalangeal joints. Laboratory
studies often, but not always, show positive rheumatoid factor or anti-cyclic citrullinated peptide. This patient's age is atypical for RA.

Educational Objective: PMR is an inflammatory disorder that commonly affects the elderly and presents with a markedly increased ESR
(>100 mm/h), proximal muscle pain, joint stiffness, and systemic symptoms.

.,.

r ~, ~ ~ r,
Previous Next Score Report
https://t.me/USMLENBME2CK Lab Values Calculator Help Pause
Exam Section: Item 8 of 50 National Board of Medical Examiners
■ Mark Clinical Neurology Self-Assessment

8. A 57-year-old man comes to the physician because of a 1-year history of progressive difficulty hearing, especially on the right. He frequently needs
to ask people to repeat what they are saying, especially children. His partner has told him that he has been increasing the volume of their television
at home. The patient does not recall any related trauma or previous problems with his ears. For 20 years, he has managed a nightclub that features
live performances. He has no history of serious illness. He occasionally takes aspirin, ibuprofen, or acetaminophen for headache or low back pain.
There is no fam ily history of hearing loss. His pulse is 76/min and regular, and blood pressure is 124/74 mm Hg. On examination, the external ear
canals and tympanic membranes appear normal. There is a small amount of cerumen in both ears. He has difficulty hearing whispered voice
bilaterally, especially on the right. A vibrating tuning fork placed on the forehead is heard better on the left. Air conduction is greater than bone
conduction bilaterally but clearer on the left. Which of the following is the most likely explanation for these fi ndings?

A) Adverse effect of aspirin


B) Loss of mobility of the auditory ossicles
C) Meniere disease
D) Scarring of the tympanic membranes
E) Sensorineural hearing loss
Correct Answer: E.

This patient's history of occupational noise exposure, as well as his presenting signs and symptoms of high frequency hearing loss and
normal conduction testing are consistent with a diagnosis of sensorineural hearing loss. Sensorineural hearing loss is typically caused
by damage or loss of function of the hair cel ls of the organ of Corti within the inner ear. Presbycusis is the most common cause of
sensorineural hearing loss and often occurs with prolonged exposure to loud noise. Examination typical ly discloses high frequency
hearing loss along with a Rinne test demonstrating normal air conduction and decreased hearing in the affected ear, and a Weber test
with decreased hearing in the affected ear.

Incorrect Answers: A, B, C, and D.

Adverse effects of aspirin (Choice A) include tinnitus and sensorineural hearing loss with high doses or prolonged dosing, which are
typically reversible with cessation of aspirin usage. This patient's history of occasional aspirin usage is not suggestive of aspirin as a
likely cause of his symptoms.

Loss of mobility of the auditory ossicles (Choice B) is caused by otosclerosis, in which the small bones of the midd le ear undergo bony
remodeling and loss of mobi lity that impairs their abi lity to transmit mechanical vibrations. Otosclerosis presents with low frequency
conductive hearing loss and tinnitus.

Meniere disease (Choice C) is a condition of episodic vertigo and sensorineural hearing loss that results from pressure accumu lation in ....
- - - - -- -- - - - -- - -- -- - - -- -- -- - - -- --- - ---- - - - -- - - -- - - - - - - - - - - - - - - - - - - - - - - -- ---
r ~, ~ ~ r,
Previous Next Score Report
https://t.me/USMLENBME2CK Lab Values Calculator Help Pause
Exam Section: Item 8 of 50 National Board of Medical Examiners
■ Mark Clinical Neurology Self-Assessment Please Walt
...
C) Meniere disease
D) Scarri ng of the tympanic membranes
E) Sensorineural hearing loss
Correct Answer: E.

This patient's history of occupational noise exposure, as well as his presenting signs and symptoms of high frequency hearing loss and
normal conduction testing are consistent with a diagnosis of sensorineural hearing loss. Sensorineural hearing loss is typically caused
by damage or loss of function of the hair cells of the organ of Corti within the inner ear. Presbycusis is the most common cause of
sensorineural hearing loss and often occurs with prolonged exposure to loud noise. Examination typically discloses high frequency
hearing loss along with a Rinne test demonstrating normal air conduction and decreased hearing in the affected ear, and a Weber test
with decreased hearing in the affected ear.

Incorrect Answers: A, B, C, and D.

Adverse effects of aspirin (Choice A) include tinnitus and sensorineural hearing loss with high doses or prolonged dosing, which are
typically reversible with cessation of aspirin usage. This patient's history of occasional aspirin usage is not suggestive of aspirin as a
likely cause of his symptoms.

Loss of mobility of the auditory ossicles (Choice B) is caused by otosclerosis, in which the small bones of the middle ear undergo bony
remodeling and loss of mobility that impairs their ability to transmit mechanical vibrations. Otosclerosis presents with low frequency
conductive hearing loss and tinnitus.

Meniere disease (Choice C) is a condition of episodic vertigo and sensorineural hearing loss that results from pressure accumulation in
the endolymph of the inner ear. This patient lacks vertigo and presents with symptoms that are constant and progressive rather than
episodic.

Scarring of the tympanic membranes (Choice D) may occur secondary to otitis externa or otitis media, and results in conductive hearing
loss.

Educational Objective: Sensorineural hearing loss commonly occurs as a result of presbycusis, in which the hair cells of the organ of
Corti are damaged. Patients often have a history of prolonged noise exposure. Examination typically discloses high frequency hearing
loss along with a Rinne test demonstrating normal air conduction and decreased hearing in the affected ear, and a Weber test with
decreased hearing in the affected ear.

.,.

r ~, ~ ~ r,
Previous Next Score Report
https://t.me/USMLENBME2CK Lab Values Calculator Help Pause
Exam Section: Item 9 of 50 National Board of Medical Examiners
■ Mark Clinical Neurology Self-Assessment

9. A 77-year-old man comes to the physician because of a 3-week history of low-grade fever, fatigue, severe headache, sensitivity to brig ht light, and
pain in the hips and shou lders. He has also had a 3.2-kg (7-lb) weight loss due to a decreased appetite during this period . He appears ill. His
temperature is 38.2°C (100.8°F). Exam ination shows mild photophobia but no men ingeal signs. There is moderate pain on palpation of the large
muscle groups of the shoulders and hips. The remainder of the examination shows no abnormalities. His hemog lobin concentration is 11.1 g/dl,
leukocyte count is 11,600/mm 3 with a normal differential, and erythrocyte sedimentation rate is 85 mm/h. Which of the following is the most
appropriate next step in diagnosis?

A) Cerebral angiography
B) MRI of the brain
C) Lum bar puncture
D) Nerve and muscle biopsy
E) Temporal artery biopsy
Correct Answer: E.

This patient's presenting findings of fever, weight loss, fatigue, headache, proximal muscle pain, and photophobia are consistent with a
diagnosis of giant cell arteritis (GCA). GCA is an autoimmune inflammatory vasculitis of medium and large vessels. Classic findings
include systemic symptoms of fever, chills, and weight loss, headache, history of preceding transient vision loss (amaurosis fugax), jaw
claudication, tenderness of the scalp, and pale optic disc edema. New headaches in an elderly patient should raise suspicion for GCA.
GCA frequently occurs in association with polymyalgia rheumatica, which is characterized by stiffness and pain of proximal muscles.
Results of laboratory studies show increased inflammatory markers, including erythrocyte sedimentation rate and C-reactive protein,
and thrombocytosis. GCA is characterized by a predilection to involve the vessels of the head and neck, and often affects the
ophthalmic artery, inflammatory occlusion of which can result in sudden, ischemic damage to the retina. Once vision loss occurs, it is
typically irreversible, and is commonly followed by vision loss in the contralateral eye within days or weeks. The diagnosis is confirmed
with biopsy of the temporal artery demonstrating granulomatous vasculitis. Treatment with high-dose corticosteroids should begin
immediately once GCA is suspected in order to preserve vision. Corticosteroid treatment does not generally compromise biopsy results
so long as the biopsy is performed within 1 to 2 weeks.

Incorrect Answers: A, B, C, and D.

Cerebral angiography (Choice A) is useful for the diagnosis of a carotid-cavernous fistula, which presents with headache, proptosis,
and conjunctiva! injection. It is not typically used for the diagnosis of GCA.

MRI of the brain (Choice B) is useful for the detection of ischemic stroke or mass lesions. It is not an optimal modality for the detection
of granulomatous vascular inflammation. ....

r ~, ~ ~ r,
Previous Next Score Report
https://t.me/USMLENBME2CK Lab Values Calculator Help Pause
Exam Section: Item 9 of 50 National Board of Medical Examiners
■ Mark Clinical Neurology Self-Assessment Please Walt

E) Temporal artery biopsy


...
Correct Answer: E.

This patient's presenting findings of fever, weight loss, fatigue, headache, proximal muscle pain, and photophobia are consistent with a
diagnosis of giant cell arteritis (GCA). GCA is an autoimmune inflammatory vasculitis of medium and large vessels. Classic findings
include systemic symptoms of fever, chills, and weight loss, headache, history of preceding transient vision loss (amaurosis fugax), jaw
claudication, tenderness of the scalp, and pale optic disc edema. New headaches in an elderly patient should raise suspicion for GCA.
GCA frequently occurs in association with polymyalgia rheumatica, which is characterized by stiffness and pain of proximal muscles.
Results of laboratory studies show increased inflammatory markers, including erythrocyte sedimentation rate and C-reactive protein,
and thrombocytosis. GCA is characterized by a predilection to involve the vessels of the head and neck, and often affects the
ophthalmic artery, inflammatory occlusion of which can result in sudden, ischemic damage to the retina. Once vision loss occurs, it is
typically irreversible, and is commonly followed by vision loss in the contralateral eye within days or weeks. The diagnosis is confirmed
with biopsy of the temporal artery demonstrating granulomatous vasculitis. Treatment with high-dose corticosteroids should begin
immediately once GCA is suspected in order to preserve vision. Corticosteroid treatment does not generally compromise biopsy results
so long as the biopsy is performed within 1 to 2 weeks.

Incorrect Answers: A, B, C, and D.

Cerebral angiography (Choice A) is useful for the diagnosis of a carotid-cavernous fistula, which presents with headache, proptosis,
and conjunctiva! injection. It is not typically used for the diagnosis of GCA.

MRI of the brain (Choice B) is useful for the detection of ischemic stroke or mass lesions. It is not an optimal modality for the detection
of granulomatous vascular inflammation.

Lumbar puncture (Choice C) is useful for the diagnosis of meningitis, idiopathic intracranial hypertension, encephalitis, multiple
sclerosis, or leptomeningeal disease. GCA does not typically cause changes in intracranial pressure or cerebrospinal fluid contents.

Nerve and muscle biopsy (Choice D) are useful for the diagnosis of vasculitis or myositis. These modalities have poor sensitivity for the
detection of GCA, which does not typically cause nerve or muscular inflammation.

Educational Objective: Giant cell arteritis is a vasculitis of medium and large vessels in elderly patients and commonly presents with
systemic symptoms, headache, jaw claudication, and proximal muscle stiffness (PMR). Vasculitis of the ophthalmic artery may occur
and cause blindness. The diagnosis is confirmed with biopsy of the temporal artery demonstrating granulomatous vasculitis. Immediate
high-dose corticosteroid treatment should begin once the diagnosis is suspected and does not generally compromise biopsy results as
long as the biopsy is performed within 1 to 2 weeks.

.,.

r ~, ~ ~ r,
Previous Next Score Report
https://t.me/USMLENBME2CK Lab Values Calculator Help Pause
Exam Section : Item 10 of 50 National Board of Medical Examiners
■ Mark Clinical Neurology Self-Assessment

10. A previously healthy 42-year-old man comes to the physician because of a 1-week history of moderate back pain. The pain originates in his right
buttock and radiates down the posterior thigh and calf to the back of his foot. He has not had any changes in bladder or bowel fu nction . He has no
history of trauma to the back. His pulse is 82/min, respirations are 16/min, and blood pressure is 132/84 mm Hg. Sensation to lig ht touch is
decreased over the right posterior leg and lateral foot. Muscle strength and deep tendon reflexes are normal. Which of the following is the most
appropriate next step in management?

A) Myelography
B) Gabapentin therapy
C) Ibuprofen therapy;
D) Prednisone therapy
E) Nerve conduction studies
Correct Answer: C.

Ibuprofen therapy is the most appropriate next step in management of this patient who has signs and symptoms consistent with a right
S 1 radiculopathy. Radiculopathy refers to the neurologic symptoms secondary to the compression of spinal nerve roots either as a
result of a herniated intervertebral disc, a narrow or stenotic neural foramen, or compression from extrinsic sources such as tumors or
infection. Patients present with pain and occasionally with weakness in the dermatome and/or myotome supplied by the affected nerve
root. Radiculopathy is most common in the lumbosacral region. This patient, who presents with sensory changes affecting the buttocks,
posterior leg, and lateral foot most likely has radiculopathy from compression of the S1 nerve root. The presence of sensory symptoms
in the absence of motor symptoms suggests compression of the posterior nerve root, as this contains sensory fibers. Compression
leads to edema and inflammation of the nerve root and surrounding structures, which causes pain. This is most effectively treated with
anti-inflammatory medications such as ibuprofen. Over time, most patients will improve with conservative care and graded exercise or
physical therapy.

Incorrect Answers: A, B, D, and E.

Myelography (Choice A) involves the injection of intrathecal contrast under fluoroscopy to assess for sites of abnormal thecal sac and
neural foraminal narrowing. It is unnecessary in this case as the diagnosis can be made with physical examination alone.

Gabapentin therapy (Choice B) is commonly used in patients with painful peripheral neuropathy. It does not treat the underlying
inflammation associated with radiculopathy and is not considered first-line therapy.

Prednisone therapy (Choice D) is not the correct answer. While glucocorticoids such as dexamethasone are sometimes used for
patients with acute spinal cord compression from trauma or expanding tumors, there are many adverse effects associated with ....
- -- --- -- -- - - -- -- - -- - - -- - - -- - - - - - - - - - -- -- - - - - --
r ~, ~ ~ r,
Previous Next Score Report
https://t.me/USMLENBME2CK Lab Values Calculator Help Pause
Exam Section : Item 10 of 50 National Board of Medical Examiners
■ Mark Clinical Neurology Self-Assessment Please Walt
...
Correct Answer: C.

Ibuprofen therapy is the most appropriate next step in management of this patient who has signs and symptoms consistent with a right
S 1 radiculopathy. Radiculopathy refers to the neurologic symptoms secondary to the compression of spinal nerve roots either as a
result of a herniated intervertebral disc, a narrow or stenotic neural foramen, or compression from extrinsic sources such as tumors or
infection. Patients present with pain and occasionally with weakness in the dermatome and/or myotome supplied by the affected nerve
root. Radiculopathy is most common in the lumbosacral region. This patient, who presents with sensory changes affecting the buttocks,
posterior leg, and lateral foot most likely has radiculopathy from compression of the S1 nerve root. The presence of sensory symptoms
in the absence of motor symptoms suggests compression of the posterior nerve root, as this contains sensory fibers. Compression
leads to edema and inflammation of the nerve root and surrounding structures, which causes pain. This is most effectively treated with
anti-inflammatory medications such as ibuprofen. Over time, most patients will improve with conservative care and graded exercise or
physical therapy.

Incorrect Answers: A, B, D, and E.

Myelography (Choice A) involves the injection of intrathecal contrast under fluoroscopy to assess for sites of abnormal thecal sac and
neural foraminal narrowing. It is unnecessary in this case as the diagnosis can be made with physical examination alone.

Gabapentin therapy (Choice B) is commonly used in patients with painful peripheral neuropathy. It does not treat the underlying
inflammation associated with radiculopathy and is not considered first-line therapy.

Prednisone therapy (Choice D) is not the correct answer. While glucocorticoids such as dexamethasone are sometimes used for
patients with acute spinal cord compression from trauma or expanding tumors, there are many adverse effects associated with
glucocorticoids. This patient's radiculopathy is more appropriately treated with ibuprofen.

Nerve conduction studies (Choice E) can be performed in instances where the examination findings are not diagnostically helpful. In
this case, nerve conduction studies would confirm an S1 radiculopathy, but they are not necessary as the diagnosis can be made by
physical examination alone.

Educational Objective: Lumbosacral radiculopathy is a common condition and can result in pain and/or weakness in areas supplied by
the affected nerve root. S1 radiculopathy results in pain and sensory changes in the posterior leg and lateral foot. If the motor nerve
root is affected, weakness of plantar flexion and absence of the Achilles reflex are typical. Treatment involves the use of ibuprofen or a
similar non-steroidal anti-inflammatory drug. Over time, most patients will improve with conservative care and graded exercise or
physical therapy.

.,.

r ~, ~ ~ r,
Previous Next Score Report
https://t.me/USMLENBME2CK Lab Values Calculator Help Pause
Exam Section : Item 11 of 50 National Board of Medical Examiners
■ Mark Clinical Neurology Self-Assessment

11. A 4 ?-year-old woman is brought to the physician because of progressive loss of


visual acu ity over the past 6 months. Snel len chart testing shows a visual acuity of
20/180. A photograph of the optic fu ndus is shown. The remainder of the
exam ination shows no abnormalities. Wh ich of the following is the most likely cause
of these findings?
A) Acute glaucoma
B) Detached retina
C) Diabetic retinopathYi
D) Macular degeneration
E) Optic neuritis
Correct Answer: C.

This patient's fundus findings of intraretinal hemorrhage, retinal nerve


fiber layer infarctions, exudates, and multiple foci of neovascularization
along the superior arcade are suggestive of proliferative diabetic
retinopathy. Chronic hyperglycemia caused by poorly control led
diabetes mellitus results in deposition of advanced glycation end
products within capi llary vessels, leading to microvascular damage.
Within the retina, this process results in retinal ischemia and clinical
signs of microvascu lar damage such as microaneurysms, retinal
hemorrhage, retinal nerve fiber layer infarctions, and intraretinal edema
and exudate deposition. These findings are characteristic of the several
stages of non-proliferative diabetic retinopathy. Once retinal ischemia
becomes sufficiently severe, the increased production of vascular
endothelial growth factor (VEGF) by the ischemic retina leads to retinal
neovascularization, the hallmark of proliferative diabetic retinopathy.
Neovascularization may lead to tractional retinal detachment, vitreous
hemorrhage, and neovascular glaucoma, and it may cause permanent
blindness. Non-proliferative diabetic retinopathy is treated with rigorous
glycemic control. Once neovascularization occurs, patients are treated
with either intravitreal injections of monoclonal antibodies against
VEGF (eg, bevacizumab, ranibizumab, aflibercept) or argon laser
panretinal photocoagulation to reduce the intraocular burden of VEGF ....

r ~, ~ ~ r,
Previous Next Score Report
https://t.me/USMLENBME2CK Lab Values Calculator Help Pause
Exam Section : Item 11 of 50 National Board of Medical Examiners
■ Mark Clinical Neurology Self-Assessment

This patient's fundus findings of intraretinal hemorrhage, retinal nerve


fiber layer infarctions, exudates, and multiple foci of neovascularization
along the superior arcade are suggestive of proliferative diabetic
retinopathy. Chronic hyperglycemia caused by poorly controlled
diabetes mellitus results in deposition of advanced glycation end
products within capillary vessels, leading to microvascular damage.
Within the retina, this process results in retinal ischemia and clinical
signs of microvascular damage such as microaneurysms, retinal
hemorrhage, retinal nerve fiber layer infarctions, and intraretinal edema
and exudate deposition. These findings are characteristic of the several
stages of non-proliferative diabetic retinopathy. Once retinal ischemia
becomes sufficiently severe, the increased production of vascular
endothelial growth factor (VEGF) by the ischemic retina leads to retinal
neovascularization, the hallmark of proliferative diabetic retinopathy.
Neovascularization may lead to tractional retinal detachment, vitreous
hemorrhage, and neovascular glaucoma, and it may cause permanent
blindness. Non-proliferative diabetic retinopathy is treated with rigorous
glycemic control. Once neovascularization occurs, patients are treated
with either intravitreal injections of monoclonal antibodies against
VEGF (eg, bevacizumab, ranibizumab, aflibercept) or argon laser
panretinal photocoagulation to reduce the intraocular burden of VEGF
and to prevent the complications of neovascularization.

Incorrect Answers: A, B, D, and E.

Acute glaucoma (Choice A) typically presents with pain, conjunctiva!


injection, a fixed, mid-dilated pupil, increased intraocular pressure, and
no visible fundus abnormalities. Optic nerve cupping is a feature of
retinal ganglion cell loss resulting from increased intraocular pressure,
but it typically does not manifest for months after an episode of acute
angle closure glaucoma.

A detached retina (Choice B) presents with decreased visual acuity and


a visible region of subretinal fluid. In tractional retinal detachment
caused by diabetes mellitus, the traction is produced by a visible area
of fibrovascular proliferation extending from the retinal surface into the ....

r ~, ~ ~ r,
Previous Next Score Report
https://t.me/USMLENBME2CK Lab Values Calculator Help Pause
Exam Section : Item 11 of 50 National Board of Medical Examiners
■ Mark Clinical Neurology Self-Assessment Please Walt

and to prevent the complications of neovascularization.


...

Incorrect Answers: A, B, D, and E.

Acute glaucoma (Choice A) typically presents with pain, conjunctiva!


injection, a fixed, mid-dilated pupil, increased intraocular pressure, and
no visible fundus abnormalities. Optic nerve cupping is a feature of
retinal ganglion cell loss resulting from increased intraocular pressure,
but it typical ly does not manifest for months after an episode of acute
angle closure glaucoma.

A detached retina (Choice B) presents with decreased visual acuity and


a visible region of subretinal fluid. In tractional retinal detachment
caused by diabetes mellitus, the traction is produced by a visible area
of fibrovascular proliferation extending from the retinal surface into the
vitreous space.

Macular degeneration (Choice D) is caused by dysfunction of the


retinal pigment epithelium and presents with signs and symptoms of
visual distortion, scotoma, drusen, geographic atrophy, and
neovascu larization.

Optic neuritis (Choice E) presents with acute onset vision loss, relative
afferent pupillary defect, and pain exacerbated by extraocular
movements. It is often caused by demyelination and is potentially
associated with the subsequent development of multiple sclerosis.
Fundus findings are often normal but may include optic disc edema.

Educational Objective: Diabetic retinopathy results from microvascular


damage and retinal ischemia, and is characterized by microaneurysms,
retinal hemorrhage, retinal nerve fiber layer infarctions, intraretinal
edema and exudate deposition, and retinal neovascularization.
Patients require strict glycemic control, while patients who manifest
neovascularization require either intravitreal injections of monoclonal
antibodies against VEGF or argon laser panretinal photocoagulation to
reduce the intraocular burden of VEGF and to prevent the
complications of neovascularization.
....

r ~, ~ ~ r,
Previous Next Score Report
https://t.me/USMLENBME2CK Lab Values Calculator Help Pause
Exam Section : Item 12 of 50 National Board of Medical Examiners
■ Mark Clinical Neurology Self-Assessment

12. A 42-year-old man, who immigrated to the USA 3 years ago, tells his physician that his hands are numb. He is being treated with isoniazid for
tuberculosis. His features are mildly coarsened. Exam ination shows widespread patches of loss of pinprick and light touch sensations, especially
over the tip of the nose and the ears. The patches are oval or irregular and measure 2 to 3 cm. The common peroneal nerves feel thickened.
Which of the following is the most likely diagnosis?

A) Acute intermittent porphyria


B) Alcoholic neuropathy
C) Charcot-Marie-Tooth disease
D) Diabetic amyotrophy
E) Guillain-Barre syndrome
F) LeprosYi
G) Motor neuropathy with conduction block
H) Paraneoplastic neuropathy
I) Paraproteinemic neuropathy
Correct Answer: F.

Leprosy is a mycobacterial disease involving the skin and peripheral nerves, as the involved mycobacterium has a predilection for the
cooler areas of the body. The disease is endemic to developing countries. Patients typically present with pigmented, hypoesthetic skin
lesions with associated paresthesia along with thickened nerves and sensorimotor neuropathy, likely as a result of an immune response
leading to segmental demyelination. The most commonly affected nerves are the ulnar, median, common peroneal, facial, and greater
auricular nerves. Presentations range from one or a few well-demarcated skin lesions with associated focal neuropathy (known as
tuberculoid disease) to diffuse skin lesions and neuropathy (known as lepromatous disease). If left untreated, neuropathy can progress
and cause permanent disability, and skin lesions can become nodular and repeatedly ulcerate, which may lead to coarsened facial
features. Diagnosis is made by skin biopsy showing acid-fast bacilli within a cutaneous nerve. Treatment requires dapsone and
rifampin, along with clofazimine in lepromatous disease. Neuritis is treated with steroids.

Incorrect Answers: A, B, C, D, E, G, H, and I.

Acute intermittent porphyria (Choice A) refers to the abnormal accumulation of the red blood cell protein porphyrin and is characterized
by episodic abdominal pain, polyneuropathy, psychological symptoms, and port wine-colored urine. The associated polyneuropathy
typically presents as acute sensory and motor neuropathy of the extremities. Skin patches and coarse facial features would be atypical.
....
I ••• •• •• • •• • ~ •• • • • • •••• •• •• • •• • • • • •• ' • •• • ••• • •• •• •• • •

r ~, ~ ~ r,
Previous Next Score Report
https://t.me/USMLENBME2CK Lab Values Calculator Help Pause
Exam Section : Item 12 of 50 National Board of Medical Examiners
■ Mark Clinical Neurology Self-Assessment
...
Correct Answer: F.

Leprosy is a mycobacterial disease involving the skin and peripheral nerves, as the involved mycobacterium has a predilection for the
cooler areas of the body. The disease is endemic to developing countries. Patients typically present with pigmented, hypoesthetic skin
lesions with associated paresthesia along with thickened nerves and sensorimotor neuropathy, likely as a result of an immune response
leading to segmental demyelination. The most commonly affected nerves are the ulnar, median, common peroneal, facial, and greater
auricular nerves. Presentations range from one or a few well-demarcated skin lesions with associated focal neuropathy (known as
tuberculoid disease) to diffuse skin lesions and neuropathy (known as lepromatous disease). If left untreated, neuropathy can progress
and cause permanent disability, and skin lesions can become nodular and repeatedly ulcerate, which may lead to coarsened facial
features . Diagnosis is made by skin biopsy showing acid-fast bacilli within a cutaneous nerve. Treatment requires dapsone and
rifampin, along with clofazimine in lepromatous disease. Neuritis is treated with steroids.

Incorrect Answers: A, B, C, D, E, G, H, and I.

Acute intermittent porphyria (Choice A) refers to the abnormal accumulation of the red blood cell protein porphyrin and is characterized
by episodic abdominal pain, polyneuropathy, psychological symptoms, and port wine-colored urine. The associated polyneuropathy
typically presents as acute sensory and motor neuropathy of the extremities. Skin patches and coarse facial features would be atypical.

Alcoholic neuropathy (Choice B) and paraproteinemic neuropathy (Choice I) typically present with sensorimotor polyneuropathy that
begins with sensory loss in the distal extremities, spreading proximally and involving motor nerves over time. Skin patches, coarse
facial features, and facial neuropathy would be atypical. As well, alcoholic and paraproteinemic neuropathy are not endemic to
developing areas. Paraproteinemic neuropathy refers to a heterogeneous group of neuropathies caused by immunoglobulin
accumulation.

In Charcot-Marie-Tooth disease (Choice C), an autosomal dominant mutation expressed in Schwann cells leads to symmetric motor
and sensory polyneuropathy that begins in the distal lower extremities, resu lting in skeletal deformities such as hammer toes, and may
progress to the upper extremities. This patient demonstrates predominantly sensory neuropathy along with skin lesions.

Diabetic amyotrophy (Choice D) is an uncommon neuropathy typically presenting with acute pain and weakness of the proximal lower
extremity. This patient demonstrates predominantly sensory, generalized neuropathy along with skin lesions.

Guillain-Barre syndrome (Choice E) arises from an autoimmune attack of the spinal nerve roots and typically presents with acute-onset,
ascending, bilateral muscle weakness. This patient demonstrates predominantly sensory neuropathy along with skin lesions.

Motor neuropathy with conduction block (Choice G) is an acquired autoimmune demyelinating neuropathy that causes slowly
progressing muscle weakness. This patient demonstrates predominantly sensory neuropathy along with skin lesions.

Paraneoplastic neuropathy (Choice H) refers to a heterogeneous group of neuropathies caused by cancer. This young patient lacks ....

r ~, ~ ~ r,
Previous Next Score Report
https://t.me/USMLENBME2CK Lab Values Calculator Help Pause
Exam Section : Item 12 of 50 National Board of Medical Examiners
■ Mark Clinical Neurology Self-Assessment Please Wait

leading to segmental demyelination. The most commonly affected nerves are the ulnar, median, common peroneal, facial, and greater ...
auricular nerves. Presentations range from one or a few well-demarcated skin lesions with associated focal neuropathy (known as
tuberculoid disease) to diffuse skin lesions and neuropathy (known as lepromatous disease). If left untreated, neuropathy can progress
and cause permanent disability, and skin lesions can become nodular and repeatedly ulcerate, which may lead to coarsened facial
features . Diagnosis is made by skin biopsy showing acid-fast bacilli within a cutaneous nerve. Treatment requires dapsone and
rifampin, along with clofazimine in lepromatous disease. Neuritis is treated with steroids.

Incorrect Answers: A, B, C, D, E, G, H, and I.

Acute intermittent porphyria (Choice A) refers to the abnormal accumulation of the red blood cell protein porphyrin and is characterized
by episodic abdominal pain, polyneuropathy, psychological symptoms, and port wine-colored urine. The associated polyneuropathy
typically presents as acute sensory and motor neuropathy of the extremities. Skin patches and coarse facial features would be atypical.

Alcoholic neuropathy (Choice B) and paraproteinemic neuropathy (Choice I) typically present with sensorimotor polyneuropathy that
begins with sensory loss in the distal extremities, spreading proximally and involving motor nerves over time. Skin patches, coarse
facial features, and facial neuropathy would be atypical. As well, alcoholic and paraproteinemic neuropathy are not endemic to
developing areas. Paraproteinemic neuropathy refers to a heterogeneous group of neuropathies caused by immunoglobulin
accumulation.

In Charcot-Marie-Tooth disease (Choice C), an autosomal dominant mutation expressed in Schwann cells leads to symmetric motor
and sensory polyneuropathy that begins in the distal lower extremities, resu lting in skeletal deformities such as hammer toes, and may
progress to the upper extremities. This patient demonstrates predominantly sensory neuropathy along with skin lesions.

Diabetic amyotrophy (Choice D) is an uncommon neuropathy typically presenting with acute pain and weakness of the proximal lower
extremity. This patient demonstrates predominantly sensory, generalized neuropathy along with skin lesions.

Guillain-Barre syndrome (Choice E) arises from an autoimmune attack of the spinal nerve roots and typically presents with acute-onset,
ascending, bilateral muscle weakness. This patient demonstrates predominantly sensory neuropathy along with skin lesions.

Motor neuropathy with conduction block (Choice G) is an acquired autoimmune demyelinating neuropathy that causes slowly
progressing muscle weakness. This patient demonstrates predominantly sensory neuropathy along with skin lesions.

Paraneoplastic neuropathy (Choice H) refers to a heterogeneous group of neuropathies caused by cancer. This young patient lacks
evidence of or risk factors for cancer and shows generalized skin lesions consistent with leprosy.

Educational Objective: Leprosy is a mycobacterial disease involving the skin and peripheral nerves that is endemic to developing
countries. Patients typically present with pigmented, hypoesthetic skin lesions along with sensorimotor neuropathy in presentations that
range from focal to generalized. ....

r ~, ~ ~ r,
Previous Next Score Report
https://t.me/USMLENBME2CK Lab Values Calculator Help Pause
Exam Section: Item 13 of 50 National Board of Medical Examiners
■ Mark Clinical Neurology Self-Assessment

13. A 77-year-old man is brought to the emergency department because of confusion, lethargy, and excessive sleepiness over the past 3 weeks.
Three days ago, he fell and struck his head. On arrival, he has a generalized tonic-clonic seizure that lasts 40 seconds. He has hypertension and
type 2 diabetes mellitus and has a history of frequent urinary tract infections and renal calculi. His medications include losartan, glyburide, and
trimethoprim-sulfamethoxazole. He is lethargic but easily aroused. His temperature is 37°C (98.6°F), pulse is 110/min, respirations are 20/min, and
blood pressure is 190/ 105 mm Hg. Exam ination shows an ecchymosis over the left scalp. Pupils are equally round and reactive to light. Doll's eye
(oculocephalic) maneuver shows full eye movement. Cranial nerves are intact. He moves all four extremities in response to painful stimuli. Deep
tendon reflexes are absent in the upper and lower extremities. Babinski sign is absent bilaterally. There is an occasional myoclonic jerk in the lower
extremities. He is disoriented to place and time. Laboratory studies show:
Hemoglobin 9 g/dL
Leukocyte count 7500/mm 3
Serum
Urea nitrogen 105 mg/dL
Creatinine 5.3 mg/dL
Alkaline phosphatase 150 U/L
ALT 47 U/L

Which of the following is the most likely cause of the patient's lethargy?

A) Hepatic failure
B) Pontine hematoma
C) Status epilepticus
D) Subdural hematoma
E) Urem ic encephalopathy
Correct Answer: E.

Acute or chronic renal failure can lead to the accumulation of toxins, such as urea, which may result in uremic encephalopathy. This
patient possesses several risk factors for acute renal failure including diabetes mellitus, hypertension, and frequent renal calcu li (which
may lead to obstructive uropathy). The severity of mental status changes typically correlates with the degree of azotemia;
encephalopathy typical ly occurs when the blood urea nitrogen is greater than 100 mg/dl. Patients typical ly present with reversible
neurologic symptoms including lethargy, disorientation, hallucinations, and diffuse weakness. Signs of neurologic excitability including
tremor, myoclonus, and generalized seizures are also common. Coma may occur in severe cases. Increased blood urea nitrogen
confirms the diagnosis. Hypertension, hyperkalemia, metabolic acidosis, and anemia are also commonly seen in patients with renal
failure (secondary to decreased filtration, potassium secretion, proton secretion, and erythropoietin production, respectively).

r ~~---•-····
~ , -· - ······ - ' ... - - -• . ...... - ... ~ ~ r,
Previous Next Score Report
https://t.me/USMLENBME2CK Lab Values Calculator Help Pause
Exam Section: Item 13 of 50 National Board of Medical Examiners
■ Mark Clinical Neurology Self-Assessment Please Wait

E) Uremic encephalopathy ...


Correct Answer: E.

Acute or chronic renal failure can lead to the accumulation of toxins, such as urea, which may result in uremic encephalopathy. This
patient possesses several risk factors for acute renal failure including diabetes mellitus, hypertension, and frequent renal calcu li (which
may lead to obstructive uropathy). The severity of mental status changes typical ly correlates with the degree of azotemia;
encephalopathy typical ly occurs when the blood urea nitrogen is greater than 100 mg/dl. Patients typical ly present with reversible
neurologic symptoms including lethargy, disorientation, hallucinations, and diffuse weakness. Signs of neurologic excitabi lity including
tremor, myoclonus, and generalized seizures are also common. Coma may occur in severe cases. Increased blood urea nitrogen
confirms the diagnosis. Hypertension, hyperkalemia, metabolic acidosis, and anemia are also commonly seen in patients with renal
failure (secondary to decreased filtration, potassium secretion, proton secretion, and erythropoietin production, respectively).
Management requires hemodialysis, which typical ly leads to symptom resolution.

Incorrect Answers: A, B, C, and D.

Acute hepatic failure (Choice A) presents with confusion or stupor along with neurologic symptoms such as cortical blindness, asterixis,
tremor, myoclonus, and frontal release signs. This patient does not have a history of chronic liver disease or risk factors for chronic liver
disease and does not show increased transaminases to indicate acute liver failure.

Pontine hematoma (Choice B) may result from chronic hypertension and can present with brainstem signs and hemiparesis, or coma
and quadriparesis (depending on the extent of the hematoma). This patient's motor function is intact, and brainstem signs are absent.

Status epilepticus (Choice C) may present with persistent motor behavior (if convulsive) or decreased consciousness (if
nonconvulsive). In nonconvulsive status epilepticus, subtle neurologic findings such as eye movement abnormalities, myoclonus, and
aphasia may be demonstrated. Status epilepticus typically occurs in patients with epi lepsy or critically ill patients.

Subdural hematoma (Choice D) refers to bleeding between the dura and arachnoid membranes, most commonly caused by head
trauma. However, this patient's altered mental status predated his fall.

Educational Objective: Acute or chronic renal failure can lead to the accumulation of toxins, such as urea, which may result in uremic
encephalopathy. Patients typically present with lethargy and confusion along with signs of neurologic excitability such as tremor,
myoclonus, and generalized seizures. Resu lts of laboratory studies show azotemia (along with other evidence of acute renal failure
such as hyperkalemia and metabolic acidosis), which differentiates uremic encephalopathy from other metabolic causes of
encephalopathy.

....

r ~, ~ ~ r,
Previous Next Score Report
https://t.me/USMLENBME2CK Lab Values Calculator Help Pause
Exam Section : Item 14 of 50 National Board of Medical Examiners
■ Mark Clinical Neurology Self-Assessment

14. For 10 days, a 40-year-old man has had pain in his left buttock and posterior th igh that is exacerbated by cough ing or sneezing. Deep tendon
reflexes are absent at the left ankle. Passive flexion of the right hip with the knee extended produces pain in the left lower extremity. An x-ray and
an MRI are shown. Wh ich of the following is the most likely diagnosis?

A) Fracture of the pars interarticularis


B) Herniated nucleus pulposus
C) Mechanical low back strain
D) Metastatic carcinoma
E) Spinal stenosis
Correct Answer: B.

Herniation of the nucleus pulposus through a tear in the surrounding annulus fibrosus is the mechanism behind a herniated lumbar
disc. This herniated disc can potentially compress adjacent spinal nerve roots, leading to severe back pain with associated paresthesia,
numbness, pain, and/or weakness of the lower extremities in a dermatomal and myotomal distribution. A herniated disc may be caused
by degenerative changes, trauma, or acute strain on the vertebral column. The L5-S1 disc is the most common site of disc herniation.
The straight-leg raise test, or Lasegue sign, is performed by passively flexing the hip while keeping the knee extended. A positive ....

r ~, ~ ~ r,
Previous Next Score Report
https://t.me/USMLENBME2CK Lab Values Calculator Help Pause
Exam Section : Item 14 of 50 National Board of Medical Examiners
■ Mark Clinical Neurology Self-Assessment Please Walt
...
Correct Answer: B.

Herniation of the nucleus pulposus through a tear in the surrounding annulus fibrosus is the mechanism behind a herniated lumbar
disc. This herniated disc can potentially compress adjacent spinal nerve roots, leading to severe back pain with associated paresthesia,
numbness, pain, and/or weakness of the lower extremities in a dermatomal and myotomal distribution. A herniated disc may be caused
by degenerative changes, trauma, or acute strain on the vertebral column. The L5-S1 disc is the most common site of disc herniation.
The straight-leg raise test, or Lasegue sign, is performed by passively flexing the hip while keeping the knee extended. A positive
straight-leg raise test is highly sensitive, but not specific, for an ipsilateral herniated disc. In this case, the MRI also shows a posterior
herniation of the L5-S1 disc with associated effacement of the adjacent central spinal canal and neural foramen. Treatment involves
nonsteroidal anti-inflammatory drugs (NSAIDs), physical therapy, and local heat application. Importantly, the patient should continue
their regular activities as much as possible rather than rest, as may be their instinct. If the patient has any signs of neurologic deficit
including weakness of the lower extremities, or bladder or bowel dysfunction, surgical intervention may be indicated.

Incorrect Answers: A, C, D, and E.

Fracture of the pars interarticularis (Choice A) occurs secondary to movement that causes a load on the spine. It is seen in chi ld and
adolescent ath letes most often, and presents with low back pain during activity, which increases gradually. It may also cause
paresthesias and radiating pain to the buttock or thigh secondary to associated spondylolisthesis. On examination, lumbar extension,
not a straight-leg raise, will elicit pain.

Mechanical low back strain (Choice C) may result from heavy lifting and typically presents with dull, aching, bilateral low back pain. It
does not typically present with radicu lar or belt-like pain. The straight-leg raise would not elicit pain.

Metastatic carcinoma (Choice D) may spread to the epidural space of the spinal cord, which may lead to spinal cord compression.
Symptoms of spinal cord compression include lower extremity numbness, weakness, paralysis, abnormal deep tendon reflexes,
perinea! anesthesia, fecal incontinence with poor rectal tone, and overflow incontinence marked by the inability to void with associated
urinary retention.

Spinal stenosis (Choice E) is narrowing of the spinal canal, often secondary to degenerative joint disease, which results in compression
of the nerve roots or spinal cord. Patients may present with symptoms of radiculopathy, including back pain that radiates to the buttocks
and legs, leg numbness, paresthesia, and/or weakness. Hip extension increases pressure on the nerve roots, whi le flexion of the hips
relieves pressure in the case of lumbar spinal stenosis.

Educational Objective: Herniation of the nucleus pulposus through a tear in the surrounding annulus fibrosus is the mechanism behind
a herniated lumbar disc, which may manifest with severe back pain and paresthesia, numbness, pain, and/or weakness of the lower
extremities depending on the specific spinal nerve root affected. Treatment involves NSAIDs, physical therapy, local heat application,
and the continued performance of normal activity.
....

r ~, ~ ~ r,
Previous Next Score Report
https://t.me/USMLENBME2CK Lab Values Calculator Help Pause
Exam Section: Item 15 of 50 National Board of Medical Examiners
■ Mark Clinical Neurology Self-Assessment

15. A 52-year-old man comes to the physician because of a 3-month history of insomnia. He has had difficulty falling asleep because of restlessness
and a feeling of "ants crawling inside my legs." He notes that he has to constantly move his legs to rel ieve the sensation. He often has to uncover
his legs or hang them over the side of the bed to get some relief. These symptoms now occur almost nightly. He has milder symptoms during the
day but is generally moving around so the sensation is not as noticeable. Physical exam ination shows no abnormalities. Laboratory studies show:
Hemoglobin 13.5 g/dl
Hemoglobin A 1c 5.6%
Serum ferriti n 15 ng/ml

Thyroid fu nction tests show no abnormalities. Which of the following is the most appropriate pharmacotherapy?

A) lmipram ine
B) Paroxetine
C) Ropinirole
D) Trazodone
E) Triazolam
Correct Answer: C.

Restless legs syndrome (RLS) is a common, idiopathic movement disorder characterized by unpleasant sensations in the legs during
periods of rest or sleep that are partially alleviated by movement and lead to a significant degree of emotional distress or functional
impairment. The cause of RLS is not currently known but may involve abnormalities in iron homeostasis and dopaminergic signaling.
Patients often have a positive family history. A variety of structures have been implicated in the pathogenesis of RLS, including the
thalamus and basal ganglia. First-line treatment is with lifestyle modifications to improve sleep hygiene and with dopaminergic agonists,
including ropinirole.

Incorrect Answers: A, B, D, and E.

lmipramine (Choice A) is a tricyclic antidepressant that is useful for the treatment of depression and anxiety. Tricyclic antidepressants
may trigger or worsen RLS in some patients and are not commonly used for its treatment.

Paroxetine (Choice B) is a selective serotonin reuptake inhibitor (SSRI) and is useful for the treatment of depression. Like tricyclic
antidepressants, SSRls may worsen or trigger RLS.

Trazodone (Choice D) is an antidepressant. While it is useful for the treatment of refractory RLS, it is not considered first-line treatment
. ~

r ~, ~ ~ r,
Previous Next Score Report
https://t.me/USMLENBME2CK Lab Values Calculator Help Pause
Exam Section: Item 15 of 50 National Board of Medical Examiners
■ Mark Clinical Neurology Self-Assessment Please Walt
. -- . - • .. - -- - - ' ' - - • .. - - --- -- - . - -- -- . ...
A) lmipram ine
B) Paroxetine
C) Ropinirole
D) Trazodone
E) Triazolam
Correct Answer: C.

Restless legs syndrome (RLS) is a common, idiopathic movement disorder characterized by unpleasant sensations in the legs during
periods of rest or sleep that are partially alleviated by movement and lead to a significant degree of emotional distress or functional
impairment. The cause of RLS is not currently known but may involve abnormalities in iron homeostasis and dopaminergic signaling.
Patients often have a positive family history. A variety of structures have been implicated in the pathogenesis of RLS, including the
thalamus and basal gang lia. First-line treatment is with lifestyle modifications to improve sleep hygiene and with dopaminergic agonists,
including ropinirole.

Incorrect Answers: A, B, D, and E.

lmipramine (Choice A) is a tricyclic antidepressant that is useful for the treatment of depression and anxiety. Tricyclic antidepressants
may trigger or worsen RLS in some patients and are not commonly used for its treatment.

Paroxetine (Choice B) is a selective serotonin reuptake inhibitor (SSRI) and is useful for the treatment of depression. Like tricyclic
antidepressants, SSRls may worsen or trigger RLS.

Trazodone (Choice D) is an antidepressant. Whi le it is useful for the treatment of refractory RLS, it is not considered first-line treatment
for RLS.

Triazolam (Choice E) is a short-acting benzodiazepine and may be useful for improving sleep quality in patients with refractory RLS. It
is, however, not an optimal first-line therapy given the significant adverse effects and abuse potential of benzodiazepines.

Educational Objective: Restless legs syndrome is a common, idiopathic movement disorder characterized by unpleasant sensations in
the legs during periods of rest or sleep that are partially alleviated by movement. First-line treatment is with lifestyle modifications to
improve sleep hygiene and with dopaminergic agonists, including ropinirole.

.,.

r ~, ~ ~ r,
Previous Next Score Report
https://t.me/USMLENBME2CK Lab Values Calculator Help Pause
Exam Section : Item 16 of 50 National Board of Medical Examiners
■ Mark Clinical Neurology Self-Assessment
...

16. A 57-year-old man comes to the physician because of increasing problems with memory. He reports that he was fired from his job as an
accountant recently because he was no longer able to maintain accurate accounts. He has a 5-year history of facial grimacing and involuntary arm
and leg spasms that have become increasingly severe. He has hypertension treated with captopril. There is no fam ily history of neurologic illness.
The patient's father died at the age of 28 years of injuries sustained in a motor vehicle collision. The patient has never smoked cigarettes, and he
does not drink alcohol or use illicit drugs. His tem perature is 36.8°C (98.2°F), pulse is 80/min, respirations are 20/min, and blood pressure is
142/82 mm Hg. Neurologic examination shows frequent, asymmetric, involuntary movements of all extremities that are random and jerk-like. On
mental status examination, the patient recalls zero of three objects after 5 minutes; he cannot name the current president or recall the meaning of
proverbs. An MRI of the brain is most likely to show which of the following?

A) Atrophy of the caudate nucleus


B) Atrophy of the substantia nigra
C) Atrophy of the thalamus
D) Increased T2 signal in the globus pallidus
E) Increased T2 signal in the putamen
F) Increased T2 signal in the subthalamic nuclei
Correct Answer: A.

Huntington disease (HD) is a hereditary, progressive neurodegenerative disorder typically arising between ages 30 and 70 years
associated with atrophy of the caudate nuclei in the basal ganglia, which can be seen on MRI of the brain. The disease usually
presents with an insidious onset of psychiatric (eg, irritability, depression, psychosis) and cognitive symptoms. Motor symptoms (eg,
chorea, hypotonia, dystonia, loss of voluntary control of movements) typically appear later in the disease course. People with HD
typically die 10 to 40 years after disease onset. Chorea, a key HD symptom, can be initially experienced as restlessness. Treatment is
symptomatic and can include tetrabenazine or deutetrabenazine, which target chorea by depleting dopamine stores. HD is caused by
expansion of a trinucleotide repeat (CAG) on chromosome four, which occurs when a series of three nucleotides in repetition are
replicated incorrectly and too many repeats are included in the replicated product. HD also demonstrates anticipation, a genetic
inheritance phenomenon in which future generations are affected at a younger age or with more severe symptoms.

Incorrect Answers: B, C, D, E, and F.

The substantia nigra (Choice B) is a nucleus for dopaminergic cells and is critical for the modulation of movement in coordination with
the basal ganglia. Pathology affecting this area is associated wHh Parkinson disease. It is not affected in Huntington disease.

Atrophy of the thalamus (Choice C) may be an isolated finding in multiple sclerosis. However, HD is characterized by atrophy in the
caudate nucleus, not the thalamus. ....

r ~, ~ ~ r,
Previous Next Score Report
https://t.me/USMLENBME2CK Lab Values Calculator Help Pause
Exam Section : Item 16 of 50 National Board of Medical Examiners
■ Mark Clinical Neurology Self-Assessment Please Walt
...
B) Atrophy of the substantia nigra
C) Atrophy of the thalam us
D) Increased T2 signal in the globus pallidus
E) Increased T2 signal in the putamen
F) Increased T2 signal in the subthalamic nuclei
Correct Answer: A.

Huntington disease (HD) is a hereditary, progressive neurodegenerative disorder typically arising between ages 30 and 70 years
associated with atrophy of the caudate nuclei in the basal ganglia, which can be seen on MRI of the brain. The disease usually
presents with an insidious onset of psychiatric (eg, irritability, depression, psychosis) and cognitive symptoms. Motor symptoms (eg,
chorea, hypotonia, dystonia, loss of voluntary control of movements) typically appear later in the disease course. People with HD
typically die 10 to 40 years after disease onset. Chorea, a key HD symptom, can be initially experienced as restlessness. Treatment is
symptomatic and can include tetrabenazine or deutetrabenazine, which target chorea by depleting dopamine stores. HD is caused by
expansion of a trinucleotide repeat (CAG) on chromosome four, which occurs when a series of three nucleotides in repetition are
replicated incorrectly and too many repeats are included in the replicated product. HD also demonstrates anticipation, a genetic
inheritance phenomenon in which future generations are affected at a younger age or with more severe symptoms.

Incorrect Answers: B, C, D, E, and F.

The substantia nigra (Choice B) is a nucleus for dopaminergic cells and is critical for the modu lation of movement in coordination with
the basal ganglia. Pathology affecting this area is associated wHh Parkinson disease. It is not affected in Huntington disease.

Atrophy of the thalamus (Choice C) may be an isolated finding in mu ltiple sclerosis. However, HD is characterized by atrophy in the
caudate nucleus, not the thalamus.

Increased T2 signal in the globus pallidus (Choice D), putamen (Choice E), or subthalamic nuclei (Choice F) wou ld indicate edema,
infarct, inflammation, infection, or tumor in these structures, not atrophy. Along with the caudate nuclei, these structures make up the
basal ganglia and are responsible for stimulating or inhibiting movements. Only the caudate nucleus is atrophic in HD. Atrophy does not
cause an increased T2 signal on MRI.

Educational Objective: Huntington disease is a hereditary, progressive neurodegenerative disorder associated with atrophy of the
caudate nuclei in the basal gang lia, which can be appreciated on MRI of the brain. HD is caused by expansion of a trinucleotide repeat
(CAG) on chromosome 4.

.,.

r ~, ~ ~ r,
Previous Next Score Report
https://t.me/USMLENBME2CK Lab Values Calculator Help Pause
Exam Section : Item 17 of 50 National Board of Medical Examiners
■ Mark Clinical Neurology Self-Assessment

17. A47-year-old woman is brought to the emergency department 1 hour after the sudden onset of headache. She says that she was walking from
work to her car when she felt an explosion in her head causing her to fall to the ground. She is not sure if she lost consciousness. Since then, she
has had a severe headache characterized by double vision and throbbing pain exacerbated by head movement. She has hypertension treated with
atenolol. She has smoked two packs of cigarettes daily for 20 years. Her pulse is 80/min, respirations are 16/min, and blood pressure is 148/82
mm Hg. Examination shows left ptosis. The left pupil is 6.5 mm, and the right pupil is 4.5 mm. Upward gaze and adduction of the left eye are
limited. A CT scan of the head shows diffuse subarachnoid blood. A lumbar puncture yields grossly bloody cerebrospinal fluid. Cerebral
angiography is most likely to show a saccular aneurysm in which of the following locations?

A) Basilar artery
B) Bifurcation of the left middle cerebral artery
C) Left posterior communicating arterYi
D) Origin of left anterior cerebral artery
E) Origin of left ophthalmic artery
Correct Answer: C.

Saccular aneurysms are focal dilatations of cranial blood vessels caused by underlying weakness of the vascular wall, and frequently
occur with vascular risk factors such as hypertension or smoking, or with inherited collagen vascular diseases such as Marfan or
Ehlers-Danlos syndromes. Rupture of a saccular aneurysm causes subarachnoid hemorrhage, which presents with severe headache
of precipitous onset, nausea, vomiting, meningismus, coma, and potentially death. This patient's symptoms of ptosis, hypotropia,
exotropia, and mydriasis, are consistent with a diagnosis of third cranial nerve (ocu lomotor nerve) palsy. A new oculomotor nerve palsy
raises concern for the presence of a saccular aneurysm of the ipsilateral posterior communicating artery (PCA). Upon leaving the
brainstem, the oculomotor nerve courses beneath the posterior cerebral and posterior communicating arteries, and it is vu lnerable to
aneurysmal compression at this location. The fibers responsible for control of the pupil are found on the superficial aspect of the
oculomotor nerve and are thus first affected by external compression. This distinguishes acquired oculomotor nerve palsy caused by
aneurysmal compression from palsy caused by microvascular ischemia, which typically spares the pupil. The detection of a new
oculomotor nerve palsy is a medical emergency and should prompt an immediate search for a potential aneurysm, as aneurysmal
rupture can lead to life-threatening subarachnoid hemorrhage. PCA aneurysm is diagnosed via CT scan, MR angiography, or
conventional angiography. Unruptured aneurysms can be treated with clipping or endovascular coiling.

Incorrect Answers: A, B, D, and E.

The basilar artery (Choice A) is a potential, although less common, site of saccular aneurysm formation. This patient's presentation,
including a new oculomotor nerve palsy, is more suggestive of PCA aneurysm.

r ~,
- - - -- - - - -- - -- - - - - - - - - - - -- -- - --- -- - - - - - -- - -
~
-- - - - -
~
- - - -- -r ,
- - - - - - - -- -- - -
....

Previous Next Score Report


https://t.me/USMLENBME2CK Lab Values Calculator Help Pause
Exam Section : Item 17 of 50 National Board of Medical Examiners
■ Mark Clinical Neurology Self-Assessment Please Walt

E) Origin of left ophthalmic artery ...


Correct Answer: C.

Saccular aneurysms are focal dilatations of cranial blood vessels caused by underlying weakness of the vascular wal l, and frequently
occur with vascular risk factors such as hypertension or smoking, or with inherited collagen vascular diseases such as Marfan or
Ehlers-Danlos syndromes. Rupture of a saccular aneurysm causes subarachnoid hemorrhage, which presents with severe headache
of precipitous onset, nausea, vomiting, meningismus, coma, and potentially death. This patient's symptoms of ptosis, hypotropia,
exotropia, and mydriasis, are consistent with a diagnosis of third cranial nerve (ocu lomotor nerve) palsy. A new ocu lomotor nerve palsy
raises concern for the presence of a saccu lar aneurysm of the ipsilateral posterior communicating artery (PCA). Upon leaving the
brainstem, the oculomotor nerve courses beneath the posterior cerebral and posterior communicating arteries, and it is vu lnerable to
aneurysmal compression at this location. The fibers responsible for control of the pupil are found on the superficial aspect of the
ocu lomotor nerve and are thus first affected by external compression. This distinguishes acquired ocu lomotor nerve palsy caused by
aneurysmal compression from palsy caused by microvascu lar ischemia, which typically spares the pupil. The detection of a new
ocu lomotor nerve palsy is a medical emergency and should prompt an immediate search for a potential aneurysm, as aneurysmal
rupture can lead to life-threatening subarachnoid hemorrhage. PCA aneurysm is diagnosed via CT scan, MR angiography, or
conventional angiography. Unruptured aneurysms can be treated with clipping or endovascular coiling.

Incorrect Answers: A, B, D, and E.

The basilar artery (Choice A) is a potential, although less common, site of saccular aneurysm formation. This patient's presentation,
including a new oculomotor nerve palsy, is more suggestive of PCA aneurysm.

The bifurcation of the left middle cerebral artery (Choice B) and the origin of left anterior cerebral artery (Choice D) are both common
sites of saccular aneurysms. However, saccular aneurysms at these locations do not lead to compression of the oculomotor nerve and
do not cause oculomotor nerve palsy.

The origin of left ophthalmic artery (Choice E) is found near the· junction of the left internal carotid artery and the circle of Wil lis.
Saccular aneurysms are not commonly found at this location.

Educational Objective: Upon leaving the brainstem, the ocu lomotor nerve courses beneath the posterior cerebral and posterior
communicating arteries and is vulnerable to compression by a saccular aneurysm of the posterior communicating artery at this location.
Rupture of a saccular aneurysm leads to subarachnoid hemorrhage, which presents with severe headache of precipitous onset,
nausea, vomiting, meningismus, coma, and potentially death. The detection of a new ocu lomotor nerve palsy is a medical emergency
and should prompt an immediate search for a potential aneurysm via CT scan, MR angiography, or conventional angiography.

.,.

r ~, ~ ~ r,
Previous Next Score Report
https://t.me/USMLENBME2CK Lab Values Calculator Help Pause
Exam Section : Item 18 of 50 National Board of Medical Examiners
■ Mark Clinical Neurology Self-Assessment

18. A 72-year-old woman comes to the physician because of a 4-week history of involuntary movements of her tongue and jaw. She has a 3-year
history of intermittent nausea. Medications include psyllium, metoclopramide, ranitidine, calcium carbonate, and bismuth subsalicylate.
Examination shows licking movements of the tongue and side-to-side movements of the jaw. There is mild slowness of limb movements. Which of
the following drugs is the most likely cause of these fi ndings?

A) Bismuth subsalicylate
B) Calcium carbonate
C) Metoclopramide
D) Psyllium
E) Ranitidine
Correct Answer: C.

Metoclopram ide is a dopamine-2 (D 2) receptor antagonist and muscarinic receptor agonist utilized for the treatment of nausea and
gastroparesis. Metoclopramide targets nausea via central and peripheral D 2 antagonism. D 2 antagonism in the basal ganglia can
potentially cause parkinsonism (leading to this patient's bradykinesia). The chronic use of antidopaminergic agents may upregulate D 2
receptors in the basal ganglia, leading to tardive dyskinesia. Tardive dyskinesia is a syndrome of involuntary movements (eg, lip
smacking, choreoathetoid movements of the tongue). Since tard ive dyskinesia is not fully reversible for some patients, those on
antipsychotic medication or metoclopramide should be closely monitored for the development of tardive dyskinesia (or alternative
agents should be selected). Management involves reducing the dose of the offending medication and/or switching to a medication that
does not cause potent D 2 antagonism. Vesicular monoamine transporter-2 inhibitors (eg, tetrabenazine) can also improve tardive
dyskinesia by depleting dopamine stores.

Incorrect Answers: A, B, D, and E.

Bismuth subsalicylate (Choice A) is an antisecretory agent that treats diarrhea. Calcium carbonate (Choice B) neutralizes gastric acid
and relieves heartburn . Psyll ium (Choice D) is a soluble fiber that serves as a bulk-forming laxative. None of these medications act on
the central nervous system (CNS).

Ranitidine (Choice E) is a histamine-2 (H 2) receptor antagonist that decreases gastric acid secretion and is utilized to relieve reflux
symptoms or dyspepsia. H 2 antagonists may act on the CNS to cause somnolence and confusion but do not lead to involuntary motor
dysfunction.

Educational Objective: Metoclopramide is a dopamine-2 (D 2) receptor antagonist utilized for the treatment of nausea and gastroparesis
that may cause tard ive dyskinesia. Tardive dyskinesia is a syndrome of involuntary movements (eg, lip smacking, choreoathetoid ....
-- - - -- - - - - -- --- - - - -- - - -- -- -- -- - - - - - - - - - - -- - - - - - - - - -
r ~, ~ ~ r,
Previous Next Score Report
https://t.me/USMLENBME2CK Lab Values Calculator Help Pause
Exam Section : Item 18 of 50 National Board of Medical Examiners
■ Mark Clinical Neurology Self-Assessment Please Walt
- -- • .. . -· - - .. - . . - - -· - . .. . • .. - . -· , , - ...
the following drugs is the most likely cause of these fi ndings?

A) Bismuth subsalicylate
B) Calcium carbonate
C) Metoclopramide
D) Psyllium
E) Ranitidine
Correct Answer: C.

Metoclopram ide is a dopamine-2 (D 2) receptor antagonist and muscarinic receptor agonist utilized for the treatment of nausea and
gastroparesis. Metoclopramide targets nausea via central and peripheral D 2 antagonism. D 2 antagonism in the basal ganglia can
potentially cause parkinsonism (leading to this patient's bradykinesia). The chronic use of antidopaminergic agents may upregulate D 2
receptors in the basal ganglia, leading to tardive dyskinesia. Tardive dyskinesia is a syndrome of involuntary movements (eg, lip
smacking, choreoathetoid movements of the tongue). Since tard ive dyskinesia is not fully reversible for some patients, those on
antipsychotic medication or metoclopramide should be closely monitored for the development of tardive dyskinesia (or alternative
agents should be selected). Management involves reducing the dose of the offending medication and/or switching to a medication that
does not cause potent D 2 antagonism. Vesicular monoamine transporter-2 inhibitors (eg, tetrabenazine) can also improve tardive
dyskinesia by depleting dopamine stores.

Incorrect Answers: A, B, D, and E.

Bismuth subsalicylate (Choice A) is an antisecretory agent that treats diarrhea. Calcium carbonate (Choice B) neutralizes gastric acid
and relieves heartburn . Psyll ium (Choice D) is a soluble fiber that serves as a bulk-forming laxative. None of these medications act on
the central nervous system (CNS).

Ranitidine (Choice E) is a histamine-2 (H 2) receptor antagonist that decreases gastric acid secretion and is utilized to relieve reflux
symptoms or dyspepsia. H 2 antagonists may act on the CNS to cause somnolence and confusion but do not lead to involuntary motor
dysfunction.

Educational Objective: Metoclopramide is a dopamine-2 (D 2) receptor antagonist utilized for the treatment of nausea and gastroparesis
that may cause tard ive dyskinesia. Tardive dyskinesia is a syndrome of involuntary movements (eg, lip smacking , choreoathetoid
movements of the tongue) that resu lts from the chronic use of antidopaminergic agents.

.,.

r ~, ~ ~ r,
Previous Next Score Report
https://t.me/USMLENBME2CK Lab Values Calculator Help Pause
Exam Section : Item 19 of 50 National Board of Medical Examiners
■ Mark Clinical Neurology Self-Assessment

19. A 22-year-old woman, gravida 2, para 1, aborta 1, comes to the physician because of two episodes of blurred vision in the left eye and one
episode of right-sided weakness during the past 3 years. She also has had urinary urgency and frequency with some episodes of incontinence
during this time. The first episode of blurred vision occurred after a motor vehicle collision 3 years ago during wh ich she sustained a fractu re of her
left humerus and soft-tissue contusions. She has used an oral contraceptive for the past 2 years. She has smoked one pack of cigarettes daily for
3 years. She is of northern European descent and has lived in the southern United States for her entire life. Vital signs are within normal limits .
Fund uscopic examination shows mild pal lor of the left optic disc. Visual acu ity is 20/80 on the left and 20/25 on the right. Neurologic examination
shows mild right hemiparesis and bilateral ataxia on fi nger-nose and heel-shin testi ng. The remainder of the examination shows no abnormalities.
An MRI of the brain shows multiple areas of increased T2 signal in the periventricular wh ite matter and the white matter of the cerebellum. Which
of the following is the strongest pred isposing factor for th is patient's condition?

A) AncestrYi
B) History of spontaneous abortion
C) History of trauma
D) Residence in warm climate
E) Use of oral contraceptives
Correct Answer: A.

The patient's presentation with neurologic symptoms separated in time and space, and MRI findings demonstrating T2 hyperintense
lesions in the periventricular and cerebellar white matter suggest a diagnosis of multiple sclerosis. Multiple sclerosis is an immune-
mediated demyelinating disease. Risk factors include female sex, white race, age between 20 to 40 years, residence in temperate
climates, and ancestry. It is associated with HLA-DR2. Presenting symptoms depend on the location of the lesion. Optic neuritis is a
common initial manifestation and typical ly presents with poor visual acuity and a consequent afferent pupil lary defect on physical
examination. According to the McDonald criteria for multiple sclerosis diagnosis, patients must have evidence of demyelinating lesions
that are separated in time and location. This evidence can be based on subjective symptoms and/or imaging evidence. This patient
presents with episodic, progressive symptoms over the course of three years, with physical examination and MRI findings involving the
left eye, corona radiata, and cerebellum. Oligoclonal bands of immunoglobulins, indicating immune overactivation, can be found in the
cerebrospinal fluid but are not necessary for diagnosis. Treatment includes corticosteroids for symptomatic management of acute flares
and long-term disease-modifying treatments such as interferons or monoclonal antibodies.

Incorrect Answers: B, C, D, and E.

History of spontaneous abortion (Choice B) does not increase the risk for the development of multiple sclerosis. Numerous etiologies
can cause a sing le occurrence of pregnancy loss. Recurrent pregnancy loss is associated with thrombophilia, such as antiphospholipid
antibody syndrome. These conditions are characterized by a hypercoagulable state with risk for thromboembolic disease. ....

r ~, ~ ~ r,
Previous Next Score Report
https://t.me/USMLENBME2CK Lab Values Calculator Help Pause
Exam Section : Item 19 of 50 National Board of Medical Examiners
■ Mark Clinical Neurology Self-Assessment

E) Use of oral contraceptives


...
Correct Answer: A.

The patient's presentation with neurologic symptoms separated in time and space, and MRI findings demonstrating T2 hyperintense
lesions in the periventricular and cerebellar white matter suggest a diagnosis of multiple sclerosis. Multiple sclerosis is an immune-
mediated demyelinating disease. Risk factors include female sex, white race, age between 20 to 40 years, residence in temperate
climates, and ancestry. It is associated with HLA-DR2. Presenting symptoms depend on the location of the lesion. Optic neuritis is a
common initial manifestation and typically presents with poor visual acuity and a consequent afferent pupillary defect on physical
examination. According to the McDonald criteria for multiple sclerosis diagnosis, patients must have evidence of demyelinating lesions
that are separated in time and location. This evidence can be based on subjective symptoms and/or imaging evidence. This patient
presents with episodic, progressive symptoms over the cou rse of three years, with physical examination and MRI findings involving the
left eye, corona radiata, and cerebellum. Oligoclonal bands of immunoglobulins, indicating immune overactivation, can be found in the
cerebrospinal fluid but are not necessary for diagnosis. Treatment includes corticosteroids for symptomatic management of acute flares
and long-term disease-modifying treatments such as interferons or monoclonal antibodies.

Incorrect Answers: B, C, D, and E.

History of spontaneous abortion (Choice B) does not increase the risk for the development of multiple sclerosis. Numerous etiologies
can cause a single occurrence of pregnancy loss. Recurrent pregnancy loss is associated with thrombophilia, such as antiphospholipid
antibody synd rome. These conditions are characterized by a hypercoagulable state with risk for thromboembolic disease.

History of trauma (Choice C) is a risk factor for traumatic brain injury and post-concussive syndrome. The patient does not have a
reported history of trauma involving the head. Fracture of a long bone such as the humerus may be complicated by deep venous
thrombosis of the upper extremity in the acute setting.

Residence in a warm climate (Choice D) is not a predisposing factor for multiple sclerosis. Rather, residence in a temperate climate has
been associated with an increased prevalence of the disease.

Use of oral contraceptives (Choice E) increases the risk for venous thromboembolism formation. Patients may present with deep
venous thrombosis, pulmonary embolism, or, less common ly, stroke or transient ischemic attack if a patent foramen ovale or cardiac
septal defect is present.

Educational Objective: Multiple sclerosis is a progressive autoimmune demyelinating disease that affects neurons of the central
nervous system, resulting in neurologic symptoms that are separated in time and space. Risk factors include female sex, white race,
age between 20 to 40 years, residence in temperature climates, and ancestry (HLA-DR2 haplotype ).

.,.

r ~, ~ ~ r,
Previous Next Score Report
https://t.me/USMLENBME2CK Lab Values Calculator Help Pause
Exam Section: Item 20 of 50 National Board of Medical Examiners
■ Mark Clinical Neurology Self-Assessment

20. Four days after hospitalization for internal fixation of a femoral fracture sustained in a motor vehicle collision, a 32-year-old man becomes agitated
and disruptive. His temperature is 38°C (100.4 °F), pulse is 124/min, respirations are 24/min, and blood pressure is 180/100 mm Hg. He is
diaphoretic and picking at his bedsheets. Funduscopic and motor and sensory exam inations show no abnormalities. Mental status exam ination
shows psychomotor agitation and disorientation. His hematocrit is 40%, and leukocyte count is 7000/mm 3. Which of the following is the most likely
diagnosis?

A) Cryptococcal meningitis
B) Delirium tremens
C) Fat embolism
D) Hypoglycemia
E) Subdural hematoma
Correct Answer: B.

This patient has likely developed delirium tremens, a serious complication of alcohol withdrawal. Alcohol increases the central nervous
system activity of gamma-aminobutyric acid, the main inhibitory neurotransmitter. Abrupt discontinuation of alcohol therefore leads to
relative sympathetic excess. Uncomplicated alcohol withdrawal typically presents with tremors, anxiety, restlessness, headache,
nausea, and diaphoresis, which can occur hours after the cessation of alcohol use. Delirium tremens, the most concerning and life-
threatening complication of alcohol withdrawal, involves severe confusion and disorientation, fluctuations in consciousness, agitation,
visual hallucinations, and autonomic instabi lity (variable pulse and blood pressure with hyperthermia). As in this patient, delirium
tremens typically begins 2 to 5 days after the patient's last alcoholic drink. Patients with delirium tremens may die from associated
dysrhythmias or respiratory failure. Delirium tremens is managed supportively with benzodiazepines to address agitation and prevent
the symptoms of withdrawal, intravenous fluids, nutritional supplementation, and frequent assessment including vital sign checks. Other
complications of alcohol withdrawal include alcoholic hallucinosis (auditory or visual hallucinations without confusion or autonomic
instability) and seizures.

Incorrect Answers: A, C, D, and E.

Cryptococcal meningitis (Choice A) is an invasive fungal infection commonly seen in immunocompromised patients. Meningitis typically
presents with headache, neck stiffness, and fever, and may feature altered mental status. This patient shows a normal leukocyte count
and no history of immunocompromise. As well, severe hypertension and symptom onset a few days after hospital admission are more
consistent with delirium tremens.

Fat embolism (Choice C) is a rare complication of long bone fracture, featuring the presence of fat globules in the pulmonary

r ~,
- --- - - - - --- - -- -- - -- - -- - - - - - - - -- - - - - - - - -- - -- - -- -
~
---
~
.
circu lation, that occurs 1 to 3 days after the disruption and dislodgement of fat tissue and/or bone marrow. Patients typically present
- - - -- - - - - - - ---
r,
- --- - - --
....

Previous Next Score Report


https://t.me/USMLENBME2CK Lab Values Calculator Help Pause
Exam Section: Item 20 of 50 National Board of Medical Examiners
■ Mark Clinical Neurology Self-Assessment
...
This patient has likely developed delirium tremens, a serious complication of alcohol withdrawal. Alcohol increases the central nervous
system activity of gamma-aminobutyric acid, the main inhibitory neurotransmitter. Abrupt discontinuation of alcohol therefore leads to
relative sympathetic excess. Uncomplicated alcohol withdrawal typically presents with tremors, anxiety, restlessness, headache,
nausea, and diaphoresis, which can occur hours after the cessation of alcohol use. Delirium tremens, the most concerning and life-
threatening complication of alcohol withdrawal, involves severe confusion and disorientation, fluctuations in consciousness, agitation,
visual hallucinations, and autonomic instability (variable pulse and blood pressure with hyperthermia). As in this patient, delirium
tremens typically begins 2 to 5 days after the patient's last alcoholic drink. Patients with delirium tremens may die from associated
dysrhythmias or respiratory failure. Delirium tremens is managed supportively with benzodiazepines to address agitation and prevent
the symptoms of withdrawal, intravenous fluids, nutritional supplementation, and frequent assessment including vital sign checks. Other
complications of alcohol withdrawal include alcoholic hallucinosis (auditory or visual hallucinations without confusion or autonomic
instability) and seizures.

Incorrect Answers: A, C, D, and E.

Cryptococcal meningitis (Choice A) is an invasive fungal infection commonly seen in immunocompromised patients. Meningitis typically
presents with headache, neck stiffness, and fever, and may feature altered mental status. This patient shows a normal leukocyte count
and no history of immunocompromise. As well, severe hypertension and symptom onset a few days after hospital admission are more
consistent with delirium tremens.

Fat embolism (Choice C) is a rare complication of long bone fracture, featuring the presence of fat globules in the pulmonary
circulation, that occurs 1 to 3 days after the disruption and dislodgement of fat tissue and/or bone marrow. Patients typically present
with respiratory distress, focal neurologic deficits or altered mental status, and a petechial rash. Disseminated intravascular coagulation
and consequent anemia may occur. Hypotension would be more likely than hypertension, and hyperthermia would be atypical.

Hypoglycemia (Choice D) may manifest with sympathoadrenal symptoms (tremors, palpitations, diaphoresis, mild hypertension, mild
tachycardia) along with cognitive impairment. This patient lacks known risk factors for hypoglycemia and has severe, not mild,
hypertension and tachycardia. Hyperthermia would also be atypical.

A traumatic subdural hematoma (Choice E), defined by bleeding between the dura and arachnoid membranes, would be more common
in an older adult as a result of cerebral atrophy. It would likely present with progressive neurologic decline and evidence of increased
intracranial pressure. This patient is young and does not show papilledema on fundoscopic examination or neurologic deficits on
physical examination. Hyperthermia would also be atypical.

Educational Objective: Delirium tremens is a life-threatening complication of alcohol withdrawal that presents with severe confusion,
agitation, visual hallucinations, and autonomic instability. Delirium tremens typically begins 2 to 5 days after the patient's last alcoholic
drink.

....

r ~, ~ ~ r,
Previous Next Score Report
https://t.me/USMLENBME2CK Lab Values Calculator Help Pause
Exam Section : Item 21 of 50 National Board of Medical Examiners
■ Mark Clinical Neurology Self-Assessment

2 1. A 57-year-old woman with metastatic breast cancer comes to the physician for a follow-up exam ination. She reports that her pain is poorly
controlled with her cu rrent dosage of oral sustai ned-release morphine. One month ago, she reported the same problem and the dosage of
morphine was increased. On exam ination today, she is ag itated and tearful and insists on another increase in her morphine dosage. Wh ich of the
following is the most likely explanation for this patient's request?

A) Inadequate pain control


B) Intent to attempt suicide
C) Major depressive disorder
D) Substance abuse
E) Surreptitious use of morphine by fam ily
Correct Answer: A.

Inadequate pain control is the most likely explanation for the patient's request. Opioid analgesics are indicated for the management of
pain associated with malignancy. Dosage often needs to be increased because of progression of metastatic disease as well as the
development of opioid tolerance. Tolerance describes the need for an increasing dose of medication to achieve the same effect as prior
doses. This occurs as a resu lt of the downregulation of opioid receptors with recurrent and/or prolonged exposure. It is important to
recognize that pain caused by cancer is associated with significant limitations in functional status and quality of life .

Incorrect Answers: B, C, D, and E.

Intent to attempt suicide (Choice B) is less likely in this scenario than inadequate pain control. Prescription of opioid analgesics (as well
as other central nervous system depressants) carries a risk for unintentional or intentional overdose. Major depressive disorder (Choice
C) is associated with many types of cancer, and patients should be assessed for symptoms and self-harm ideations. Patients should
also be counseled on the risks of overdose, recognition of the signs of overdose, and prescribed a naloxone kit with instructions on use
in an emergency.

Substance abuse (Choice D) and surreptitious use of morphine by fami ly (Choice E) are also risks of prescription opioid analgesics, but
less likely than inadequate pain control for the dosage increase request in this scenario. It is the responsibility of the prescriber to
assess for the inappropriate use of opioid analgesics, such as misuse, diverting pills, or obtaining simi lar medications from multiple
prescribers. A thorough history should be taken to examine for prior difficulties with substance abuse, comorbid psychiatric disorders
should be appropriately managed, and continuous monitoring for patterns of aberrant behavior shou ld be performed.

Educational Objective: Pain is a common debilitating symptom associated with metastatic malignancy. Dosage increases caused by
progressive spread and growth of lesions as wel l as the development of opioid tolerance is expected. Monitoring for aberrant behavior, ....
- -- -- -- -- -- - - - -- - - -- - - --- - -- -- - - -- - - - -- - - - - - - - - - - - - - - - - - - - - - - - -
r ~, ~ ~ r,
Previous Next Score Report
https://t.me/USMLENBME2CK Lab Values Calculator Help Pause
Exam Section : Item 21 of 50 National Board of Medical Examiners
■ Mark Clinical Neurology Self-Assessment Please Wait
- - - .. --
. -- - - - . - . . - - - ---
..,. -- . - - -- . . -- -- -
morphine was increased. On exam ination today, she is ag itated and tearful and insists on another increase in her morphine dosage. Wh ich of the
...
following is the most likely explanation for this patient's request?

A) Inadequate pain control


B) Intent to attempt suicide
C) Major depressive disorder
D) Substance abuse
E) Surreptitious use of morphine by fam ily
Correct Answer: A.

Inadequate pain control is the most likely explanation for the patient's request. Opioid analgesics are indicated for the management of
pain associated with malignancy. Dosage often needs to be increased because of progression of metastatic disease as well as the
development of opioid tolerance. Tolerance describes the need for an increasing dose of medication to achieve the same effect as prior
doses. This occurs as a resu lt of the downregulation of opioid receptors with recurrent and/or prolonged exposure. It is important to
recognize that pain caused by cancer is associated with significant limitations in functional status and quality of life .

Incorrect Answers: B, C, D, and E.

Intent to attempt suicide (Choice B) is less likely in this scenario than inadequate pain control. Prescription of opioid analgesics (as well
as other central nervous system depressants) carries a risk for unintentional or intentional overdose. Major depressive disorder (Choice
C) is associated with many types of cancer, and patients should be assessed for symptoms and self-harm ideations. Patients should
also be counseled on the risks of overdose, recognition of the signs of overdose, and prescribed a naloxone kit with instructions on use
in an emergency.

Substance abuse (Choice D) and surreptitious use of morphine by fami ly (Choice E) are also risks of prescription opioid analgesics, but
less likely than inadequate pain control for the dosage increase request in this scenario. It is the responsibility of the prescriber to
assess for the inappropriate use of opioid analgesics, such as misuse, diverting pills, or obtaining simi lar medications from multiple
prescribers. A thorough history should be taken to examine for prior difficulties with substance abuse, comorbid psychiatric disorders
should be appropriately managed, and continuous monitoring for patterns of aberrant behavior shou ld be performed.

Educational Objective: Pain is a common debilitating symptom associated with metastatic malignancy. Dosage increases caused by
progressive spread and growth of lesions as wel l as the development of opioid tolerance is expected. Monitoring for aberrant behavior,
such as frequent lost pil ls or recurrent early refil ls, should be performed to maintain safe opioid prescribing practices.

.,.

r ~, ~ ~ r,
Previous Next Score Report
https://t.me/USMLENBME2CK Lab Values Calculator Help Pause
Exam Section: Item 22 of 50 National Board of Medical Examiners
■ Mark Clinical Neurology Self-Assessment
...

22. A 62-year-old man comes to the physician because of a 9-month history of weakness in his left arm and light-headedness when he plays tennis.
His left hand and arm are cool to the touch during these episodes, and his symptoms usually resolve after 15 minutes of rest. He plays tennis for
60 minutes three times weekly, and the episodes occur twice weekly. He is left-handed. He does not recall any trauma to his left arm and has not
had any other symptoms. He smoked one and one-half packs of cigarettes daily for 27 years but stopped 10 years ago. He drinks one glass of
wine daily. His pulse is 78/min and regular, and blood pressure is 150/88 mm Hg in the rig ht arm and 136/70 mm Hg in the left arm. The lungs are
clear to auscultation. S 1 and S 2 are normal. The left radial pulse is decreased. The remainder of the examination, including neurologic examination,
shows no abnormalities. Wh ich of the following is the most appropriate next step in diagnosis?

A) Ambulatory ECG monitoring


B) Tilt test
C) Echocardiography
D) Cardiac stress scintigraphy
E) MR angiography
Correct Answer: E.

The patient is presenting with signs and symptoms concerning for subclavian steal syndrome. Clinical manifestations include exercise-
induced vertigo, lightheadedness, upper extremity fatigue, asymmetric upper extremity pu lses, and impaired perfusion to the affected
limb. The underlying pathophysiology is a result of stenosis of the subclavian artery proximal to the origin of the vertebral artery, which
leads to decreased flow in the distal subclavian artery. Consequently, the affected arm may be perfused through retrograde flow in the
ipsilateral vertebral artery, diminishing posterior cerebral circulation and resulting in symptoms of vertebrobasilar insufficiency (eg,
vertigo, lightheadedness, disequilibrium, ataxia, and nystagmus). The most common cause of subclavian steal syndrome is
atherosclerosis. Diagnosis is made by imaging the arteries of the upper extremities and neck to assess for stenotic lesions and the
direction of flow via ultrasonography or MR angiography. Subclavian steal syndrome is a compensatory mechanism that is typical ly not
life-threatening by itself, but instead indicates significant atherosclerotic disease that shou ld be managed with surgical bypass or
angioplasty (with or without stenting) along with lifestyle changes to prevent further atherosclerosis.

Incorrect Answers: A, B, C, and D.

Ambu latory ECG monitoring (Choice A) is indicated if there is concern for a cardiac arrhythmia as a potential cause for the patient's
symptoms. Exercise-induced presyncope may be caused by a cardiac arrhythmia that warrants extended ECG monitoring. However,
the patient's unilateral upper extremity symptoms, asymmetric pu lses, and blood pressure difference are more suggestive of subclavian
steal syndrome.

r ~, - - - --- - - - -- -- - - -- - -- - -- -- - - - -- - -- - -- - - --
~ ~
.
Tilt test (Choice B) may be used in the evaluation of presyncope and syncope to identify a vasovagal or orthostatic cause. The patient's
-- - ---
r, -- - - -- ....

Previous Next Score Report


https://t.me/USMLENBME2CK Lab Values Calculator Help Pause
Exam Section: Item 22 of 50 National Board of Medical Examiners
■ Mark Clinical Neurology Self-Assessment Please Walt

Correct Answer: E. ...

The patient is presenting with signs and symptoms concerning for subclavian steal syndrome. Clinical manifestations include exercise-
induced vertigo, lightheadedness, upper extremity fatigue, asymmetric upper extremity pu lses, and impaired perfusion to the affected
limb. The underlying pathophysiology is a result of stenosis of the subclavian artery proximal to the origin of the vertebral artery, which
leads to decreased flow in the distal subclavian artery. Consequently, the affected arm may be perfused through retrograde flow in the
ipsilateral vertebral artery, diminishing posterior cerebral circulation and resulting in symptoms of vertebrobasilar insufficiency (eg,
vertigo, lightheadedness, disequilibrium, ataxia, and nystagmus). The most common cause of subclavian steal syndrome is
atherosclerosis. Diagnosis is made by imaging the arteries of the upper extremities and neck to assess for stenotic lesions and the
direction of flow via ultrasonography or MR angiography. Subclavian steal syndrome is a compensatory mechanism that is typical ly not
life-threatening by itself, but instead indicates significant atherosclerotic disease that shou ld be managed with surgical bypass or
angioplasty (with or without stenting) along with lifestyle changes to prevent further atherosclerosis.

Incorrect Answers: A, B, C, and D.

Ambu latory ECG monitoring (Choice A) is indicated if there is concern for a cardiac arrhythmia as a potential cause for the patient's
symptoms. Exercise-induced presyncope may be caused by a cardiac arrhythmia that warrants extended ECG monitoring. However,
the patient's unilateral upper extremity symptoms, asymmetric pu lses, and blood pressure difference are more suggestive of subclavian
steal syndrome.

Tilt test (Choice B) may be used in the evaluation of presyncope and syncope to identify a vasovagal or orthostatic cause. The patient's
pu lse, blood pressure, and symptoms are observed as the table is rotated from supine to upright. Patients typical ly report symptoms
with changes in position.

Echocardiography (Choice C) is useful to evaluate for structural heart disease, valvu lar disease, or heart failure that may be causing
exercise-induced symptoms. Cardiac arrhythmias may be noted during the study. Unilateral upper extremity signs and symptoms are
more suggestive of subclavian steal syndrome, and MR angiography is more likely to confirm the diagnosis in this case.

Cardiac stress scintigraphy (Choice D) observes myocardial perfusion at rest and in a stressed state and is utilized in the evaluation of
angina and coronary artery disease. Given the patient's smoking history and likely atherosclerotic disease as the cause of his
subclavian steal syndrome, he should undergo cardiac stress testing; however, MR angiography is the better initial test to obtain the
diagnosis.

Educational Objective: Subclavian steal syndrome results from stenosis or occlusion of the subclavian artery proximal to the vertebral
artery origin, which causes retrograde flow from the vertebral artery to perfuse the limb. Diagnosis can be made via imaging that
evaluates for stenotic lesions and flow within the vasculature of the neck and upper extremities, such as MR angiography.

....

r ~, ~ ~ r,
Previous Next Score Report
https://t.me/USMLENBME2CK Lab Values Calculator Help Pause
Exam Section: Item 23 of 50 National Board of Medical Examiners
■ Mark Clinical Neurology Self-Assessment

23. A 35-year-old man with a 3-year history of dermatomyositis has had progressive weakness for 3 weeks. He had previously responded well to
high-dose pred nisone therapy. He appears cushingoid; serum creatine kinase and aldolase activities are now normal. Which of the following is the
most likely diagnosis?

A) Corticosteroid myopathYi
B) Hypothyroidism
C) Major depressive disorder
D) Peripheral neuropathy
E) Prog ressive myositis
Correct Answer: A.

Myopathy is a potential adverse effect of long-term corticosteroid use, which is frequently required for the treatment of dermatomyositis.
Myopathy occurs as a result of the catabolic effect of glucocorticoids (eg, prednisone) on muscle. It typically manifests with the
progressive onset of proximal muscle weakness followed by muscle atrophy. Myalgias and muscle tenderness do not typically occur.
Other adverse effects of corticosteroids, including a cushingoid appearance, may also be present. Corticosteroid myopathy is not
associated with inflammation or cellular damage; thus, creatine kinase and aldolase levels are normal. This is in contrast to
dermatomyositis, which is caused by an inflammatory infiltrate of the muscle causing myocyte damage, leading to the release of
intracellular molecules, including creatine kinase and aldolase. Active dermatomyositis would also be expected to show cutaneous
features such as pink papules over the dorsal fingers (Gottron papules), pink to light purple patches over the upper eyelids (heliotrope
sign), light pink to lilac-colored patches with telangiectasias on the upper back and sun-exposed chest (shawl sign), and dilated
capillaries with drop out on the proximal nail folds.

Incorrect Answers: B, C, D, and E.

Hypothyroidism (Choice B) may present with myopathy and myalgias, typically associated with increased creatine kinase levels. Other
symptoms of hypothyroidism are commonly present, includ ing weight gain, fatigue, constipation, cold intolerance, and menstrual
irregularities. Physical examination may show bradycardia, dry, edematous skin, and delayed relaxation of deep tendon reflexes.

Major depressive disorder (Choice C) is characterized by depressed mood, decreased energy, anhedonia, sleep disturbances, appetite
disturbances, guilt, poor concentration, and psychomotor agitation or retardation. Decreased energy or psychomotor retardation may be
perceived as muscle weakness, but this patient's history of dermatomyositis, lack of other depressive symptoms, and demonstration of
manifestations of glucocorticoid use indicate this as a more likely cause.

Peripheral neuropathy (Choice D) has a variety of causes including diabetes mellitus, vitamin deficiency, and medication use, and can ....
- - - -- - ----- -- - - - -- - .. - - -- - - -- --- - -- - -- -- - - - --- - -- - --- - -- - - - - - - - -- - - -- - - - - --
r ~, ~ ~ r,
Previous Next Score Report
https://t.me/USMLENBME2CK Lab Values Calculator Help Pause
Exam Section: Item 23 of 50 National Board of Medical Examiners
■ Mark Clinical Neurology Self-Assessment

Correct Answer: A. ...

Myopathy is a potential adverse effect of long-term corticosteroid use, which is frequently required for the treatment of dermatomyositis.
Myopathy occurs as a result of the catabolic effect of glucocorticoids (eg, prednisone) on muscle. It typically manifests with the
progressive onset of proximal muscle weakness followed by muscle atrophy. Myalgias and muscle tenderness do not typically occur.
Other adverse effects of corticosteroids, including a cushingoid appearance, may also be present. Corticosteroid myopathy is not
associated with inflammation or cellular damage; thus, creatine kinase and aldolase levels are normal. This is in contrast to
dermatomyositis, which is caused by an inflammatory infiltrate of the muscle causing myocyte damage, leading to the release of
intracellular molecules, including creatine kinase and aldolase. Active dermatomyositis would also be expected to show cutaneous
features such as pink papules over the dorsal fingers (Gottron papules), pink to light purple patches over the upper eyelids (heliotrope
sign), light pink to lilac-colored patches with telangiectasias on the upper back and sun-exposed chest (shawl sign), and dilated
capillaries with drop out on the proximal nail folds.

Incorrect Answers: B, C, D, and E.

Hypothyroidism (Choice B) may present with myopathy and myalgias, typically associated with increased creatine kinase levels. Other
symptoms of hypothyroidism are common ly present, including weight gain, fatigue, constipation, cold intolerance, and menstrual
irregularities. Physical examination may show bradycardia, dry, edematous skin, and delayed relaxation of deep tendon reflexes.

Major depressive disorder (Choice C) is characterized by depressed mood, decreased energy, anhedonia, sleep disturbances, appetite
disturbances, guilt, poor concentration, and psychomotor agitation or retardation. Decreased energy or psychomotor retardation may be
perceived as muscle weakness, but this patient's history of dermatomyositis, lack of other depressive symptoms, and demonstration of
manifestations of glucocorticoid use indicate this as a more likely cause.

Peripheral neuropathy (Choice D) has a variety of causes including diabetes mellitus, vitamin deficiency, and medication use, and can
be primarily sensory, motor, or both. While motor neuropathy may cause progressive weakness, it is not seen in dermatomyositis and
would not explain the patient's cushingoid appearance.

Progressive myositis (Choice E) may occur when dermatomyositis is inadequately treated. A perimysial CD4+ T-lymphocyte
inflammatory infiltrate causes damage to the myocytes, leading to release of creatine kinase and aldolase, intracellular molecules. This
patient's dermatomyositis is well controlled , and the normal creatine kinase and aldolase levels suggest an alternative cause of muscle
weakness.

Educational Objective: Corticosteroid myopathy is a potential adverse effect of long-term corticosteroid use. It occurs because of the
catabolic effect of glucocorticoids on muscle, and it is typically manifest by progressive proximal muscle weakness followed by muscle
atrophy without increased creatine kinase or aldolase levels.

....

r ~, ~ ~ r,
Previous Next Score Report
https://t.me/USMLENBME2CK Lab Values Calculator Help Pause
Exam Section: Item 24 of 50 National Board of Medical Examiners
■ Mark Clinical Neurology Self-Assessment

24. A 72-year-old woman is brought to the physician because of increasing cl umsiness, restlessness, and confusion over the past 3 weeks. She
currently lives with her daughter. She can no longer remember how to make coffee or button her clothes. She has urinary incontinence. Her
med ications incl ude digoxin, furosemide, potassium, ibuprofen, ranitidine, and diphenhydramine. One month ago, she began taking meperid ine for
a rotator cuff injury. Examination shows disorientation, memory loss, and dyspraxia. A CT scan of the head shows mi ld cortical atrophy. Which of
the following is the most appropriate next step in management?

A) Discontinue anticholinergic medications


B) Discuss obtaining durable power of attorney
C) EEG
D) Fluorescent treponemal antibody assay
E) Prescribe an antidepressant
F) Recommend 24-hour in-home nursing assistance
G) Suggest labeling the drawers and cabinets for contents
Correct Answer: A.

Acetylcholine activity in the brain mediates attention and motivation. Anticholinergic agents commonly cause cognitive impairment or
delirium (an acute confused state) in elderly patients, who possess a decreased cognitive reserve, and are therefore one of a number
of medications to use with caution in elderly patients. Diphenhydramine is an antihistamine medication used for seasonal allergies and
causes anticholinergic adverse effects. Meperidine is an opioid medication used for analgesia that also causes anticholinergic adverse
effects. Along with cognitive impairment (leading to this patient's disorientation and memory loss), anticholinergic effects can include
sedation, dyspraxia (difficulty with the planning and execution of movement), ataxia, visual disturbances (cycloplegia), dry mouth,
urinary retention (which may lead to overflow incontinence), constipation, and flushed skin. In this scenario, the subacute onset of
cognitive symptoms coincides with the initiation of meperidine. Meperidine and diphenhydramine should be discontinued. All elderly
patients with new-onset cognitive impairment should be assessed for reversible causes with a thorough history, physical examination,
laboratory testing, neuroimaging, and medication review.

Incorrect Answer: B, C, D, E, F, and G.

Obtaining a durable power of attorney (Choice B), recommending 24-hour in-home nursing assistance (Choice F), and labeling the
drawers and cabinets for contents (Choice G) should be considered in patients with irreversible cognitive impairment that prevents
them from making their own decisions or functioning at home. T his patient's subacute confusion instead coincides with the addition of
an anticholinergic medication and will likely reverse with medication discontinuation. This patient's mild cortical atrophy on CT scan may
reflect normal aging changes. ....

r ~, ~ ~ r,
Previous Next Score Report
https://t.me/USMLENBME2CK Lab Values Calculator Help Pause
Exam Section: Item 24 of 50 National Board of Medical Examiners
■ Mark Clinical Neurology Self-Assessment

Correct Answer: A. ...

Acetylcholine activity in the brain mediates attention and motivation. Anticholinergic agents commonly cause cognitive impairment or
delirium (an acute confused state) in elderly patients, who possess a decreased cognitive reserve, and are therefore one of a number
of medications to use with caution in elderly patients. Diphenhydramine is an antihistamine medication used for seasonal allergies and
causes anticholinergic adverse effects. Meperidine is an opioid medication used for analgesia that also causes anticholinergic adverse
effects. Along with cognitive impairment (leading to this patient's disorientation and memory loss), anticholinergic effects can include
sedation, dyspraxia (difficu lty with the planning and execution of movement), ataxia, visual disturbances (cycloplegia), dry mouth,
urinary retention (which may lead to overflow incontinence), constipation, and flushed skin. In this scenario, the subacute onset of
cognitive symptoms coincides with the initiation of meperidine. Meperidine and diphenhydramine should be discontinued. All elderly
patients with new-onset cognitive impairment should be assessed for reversible causes with a thorough history, physical examination,
laboratory testing, neuroimaging, and medication review.

Incorrect Answer: B, C, D, E, F, and G.

Obtaining a durable power of attorney (Choice B), recommending 24-hour in-home nursing assistance (Choice F), and labeling the
drawers and cabinets for contents (Choice G) should be considered in patients with irreversible cognitive impairment that prevents
them from making their own decisions or functioning at home. This patient's subacute confusion instead coincides with the addition of
an anticholinergic medication and will likely reverse with medication discontinuation. This patient's mild cortical atrophy on CT scan may
reflect normal aging changes.

EEG (Choice C) may assist in diagnosing delirium or cognitive impairment when the cause of mental statu s change is unclear.
However, the diagnosis of delirium or cognitive impairment can frequently be made clinically. This patient with a likely precipitant of
cognitive impairment should have the precipitating cause addressed.

Fluorescent treponemal antibody assay (Choice D) assists in diagnosing syphilis, which can spread to the central nervous system to
cause neurosyphilis. Neurosyphilis may result in a chronically progressive dementia. Though syphilis testing is considered in patients
with cognitive impairment who possess risk factors for syphilis, this patient's subacute confusion instead coincides with the addition of
an anticholinergic medication.

Prescribing an antidepressant (Choice E) would be appropriate in patients with pseudodementia, or cognitive impairment resulting from
major depressive disorder. This patient does not show neurovegetative symptoms (eg, decreased energy or changes in appetite or
sleep) or other depressive symptoms.

Educational Objective: Anticholinergic agents (eg, diphenhydramine, meperidine [also a narcotic]) commonly cause cognitive
impairment in elderly patients. All elderly patients with new-onset cognitive impairment should be assessed for reversible causes with a
thorough history, physical examination, laboratory testing , neuroimaging, and medication review. Offending medications should be
discontinued. ....

r ~, ~ ~ r,
Previous Next Score Report
https://t.me/USMLENBME2CK Lab Values Calculator Help Pause
Exam Section: Item 25 of 50 National Board of Medical Examiners
■ Mark Clinical Neurology Self-Assessment

25. A 62-year-old man is brought to the physician by his wife because of progressive behavior changes over the past 8 weeks. His initial symptoms
included anxiety and fati gue that were followed by decreased memory and disorientation. Two weeks later, he developed progressive imbalance
that has progressed to the point where he is unable to walk unassisted. Over the past week, he has had brief body jerks that are precipitated by
loud noises. Exam ination shows disorientation, decreased memory, inability to walk due to marked ataxia, and stimulus-induced myoclonus. An
MRI of the brain shows generalized cerebral atrophy. Which of the following is the most likely diagnosis?

A) Benzodiazepine intoxication
B) Creutzfeldt-Jakob disease
C) Dementia, Alzheimer type
D) Hydrocephalus
E) Hypothyroidism
F) Multi-infarct (vascular) dementia
G) Progressive supranuclear palsy
H) Wernicke-Korsakoff syndrome
Correct Answer: B.

Creutzfeldt-Jakob disease (CJD) is a human prion disease in which prion proteins replicate throughout the brain and cause
neurotoxicity. Patients typically present with neuropsychiatric symptoms (eg, dementia, anxiety, behavioral disturbances, deficits in
higher cortical functions such as aphasia and apraxia), cerebellar manifestations (eg, ataxia), corticospinal tract dysfunction (eg, spastic
weakness), and basal ganglia signs (eg, hypokinesia, rigid ity). Stimulus-induced myoclonus is a specific sign of CJD. Over time, the
dementia rapidly progresses, invariably leading to death within a year. A probable CJD diagnosis can be made clinically and through
cerebrospinal fluid analysis, though an autopsy showing spongiform degeneration of the cortex confirms the diagnosis. Treatment is
supportive (eg, hospice evaluation, education, benzodiazepines for myoclonus).

Incorrect Answers: A, C, D, E, F, G, and H.

Benzodiazepine intoxication (Choice A) presents with a toxidrome of slurred speech, ataxia, and altered mental status. Eight weeks of
consistent benzodiazepine intoxication would be atypical, and benzodiazepine intoxication does not manifest with stimulus-induced
myoclonus or generalized cerebral atrophy.

Dementia, Alzheimer type (Choice C) is the most prevalent dementia, presenting with progressive cognitive decline that begins with
short-term memory impairment, progresses to apraxia and language abnormalities, and culminates in behavioral and personality
changes preventing the patient from performing basic activities of daily living. Rapidly progressive deficits, balance difficulty early in the ....

r ~, ~ ~ r,
Previous Next Score Report
https://t.me/USMLENBME2CK Lab Values Calculator Help Pause
Exam Section: Item 25 of 50 National Board of Medical Examiners
■ Mark Clinical Neurology Self-Assessment
...
Correct Answer: B.

Creutzfeldt-Jakob disease (CJD) is a human prion disease in which prion proteins replicate throughout the brain and cause
neurotoxicity. Patients typically present with neuropsychiatric symptoms (eg, dementia, anxiety, behavioral disturbances, deficits in
higher cortical functions such as aphasia and apraxia), cerebellar manifestations (eg, ataxia), corticospinal tract dysfunction (eg, spastic
weakness), and basal ganglia signs (eg, hypokinesia, rigidity). Stimulus-induced myoclonus is a specific sign of CJD. Over time, the
dementia rapidly progresses, invariably leading to death within a year. A probable CJD diagnosis can be made clinically and through
cerebrospinal fluid analysis, though an autopsy showing spongiform degeneration of the cortex confirms the diagnosis. Treatment is
supportive (eg, hospice evaluation, education, benzodiazepines for myoclonus).

Incorrect Answers: A, C, D, E, F, G, and H.

Benzodiazepine intoxication (Choice A) presents with a toxidrome of slurred speech, ataxia, and altered mental status. Eight weeks of
consistent benzodiazepine intoxication would be atypical, and benzodiazepine intoxication does not manifest with stimulus-induced
myoclonus or generalized cerebral atrophy.

Dementia, Alzheimer type (Choice C) is the most prevalent dementia, presenting with progressive cognitive decline that begins with
short-term memory impairment, progresses to apraxia and language abnormalities, and culminates in behavioral and personality
changes preventing the patient from performing basic activities of daily living. Rapidly progressive deficits, balance difficulty early in the
disease course, and stimulus-induced myoclonus would be atypical.

Hydrocephalus (Choice D) typically presents with signs of increased intracranial pressure (eg, positional headache, papilledema,
nausea, vomiting). Rapidly progressive dementia, ataxia, and stimulus-induced myoclonus would be atypical. MRI of the brain would
show ventriculomegaly.

Hypothyroidism (Choice E) presents with cold intolerance, weight gain, dry skin, fatigue, and mood changes. Rapidly progressive
dementia, ataxia, and stimulus-induced myoclonus would be atypical.

Multi-infarct (vascular) dementia (Choice F) refers to cognitive impairment caused by cerebrovascular disease. Patients typically
demonstrate stepwise, not progressive, focal neurologic deficits and functional decline, and stimulus-induced myoclonus would be
atypical. MRI of the brain usually shows multiple infarcts.

Progressive supranuclear palsy (Choice G) is a parkinsonian syndrome that typically presents with ophthalmoplegia, postural instability,
akinesia, and cognitive dysfunction. Stimulus-induced myoclonus and rapidly progressive deficits would be atypical.

Wernicke-Korsakoff (Choice H) syndrome, caused by chronic alcohol use, is characterized by altered mental status, ophthalmoplegia,
ataxia, and retrograde and anterograde amnesia. Startle-induced myoclonus would be atypical, and MRI of the brain would likely show
focal atro h e , mammillar bodies .

Previous Score Report


https://t.me/USMLENBME2CK Lab Values Calculator Help Pause
Exam Section: Item 25 of 50 National Board of Medical Examiners
■ Mark Clinical Neurology Self-Assessment

higher cortical functions such as aphasia and apraxia), cerebellar manifestations (eg, ataxia), corticospinal tract dysfunction (eg, spastic ...
weakness), and basal ganglia signs (eg, hypokinesia, rigidity). Stimulus-induced myoclonus is a specific sign of CJD. Over time, the
dementia rapidly progresses, invariably leading to death within a year. A probable CJD diagnosis can be made clinically and through
cerebrospinal fluid analysis, though an autopsy showing spongiform degeneration of the cortex confirms the diagnosis. Treatment is
supportive (eg, hospice evaluation, education, benzodiazepines for myoclonus).

Incorrect Answers: A, C, D, E, F, G, and H.

Benzodiazepine intoxication (Choice A) presents with a toxidrome of slurred speech, ataxia, and altered mental status. Eight weeks of
consistent benzodiazepine intoxication would be atypical, and benzodiazepine intoxication does not manifest with stimulus-induced
myoclonus or generalized cerebral atrophy.

Dementia, Alzheimer type (Choice C) is the most prevalent dementia, presenting with progressive cognitive decline that begins with
short-term memory impairment, progresses to apraxia and language abnormalities, and culminates in behavioral and personality
changes preventing the patient from performing basic activities of daily living. Rapidly progressive deficits, balance difficulty early in the
disease course, and stimulus-induced myoclonus would be atypical.

Hydrocephalus (Choice D) typically presents with signs of increased intracranial pressure (eg, positional headache, papilledema,
nausea, vomiting). Rapidly progressive dementia, ataxia, and stimulus-induced myoclonus would be atypical. MRI of the brain would
show ventriculomegaly.

Hypothyroidism (Choice E) presents with cold intolerance, weight gain, dry skin, fatigue, and mood changes. Rapidly progressive
dementia, ataxia, and stimulus-induced myoclonus would be atypical.

Multi-infarct (vascular) dementia (Choice F) refers to cognitive impairment caused by cerebrovascular disease. Patients typically
demonstrate stepwise, not progressive, focal neurologic deficits and functional decline, and stimulus-induced myoclonus would be
atypical. MRI of the brain usually shows multiple infarcts.

Progressive supranuclear palsy (Choice G) is a parkinsonian syndrome that typically presents with ophthalmoplegia, postural instability,
akinesia, and cognitive dysfunction. Stimulus-induced myoclonus and rapidly progressive deficits would be atypical.

Wernicke-Korsakoff (Choice H) syndrome, caused by chronic alcohol use, is characterized by altered mental status, ophthalmoplegia,
ataxia, and retrograde and anterograde amnesia. Startle-induced myoclonus would be atypical, and MRI of the brain would likely show
focal atrophy (eg, mammillary bodies).

Educational Objective: Creutzfeldt-Jakob disease (CJD) is a human prion disease that presents with rapidly progressing dementia,
neuropsychiatric symptoms, cerebellar dysfunction, corticospinal tract dysfunction, basal ganglia dysfunction, and stimulus-induced
myoclonus. ....

r ~, ~ ~ r,
Previous Next Score Report
https://t.me/USMLENBME2CK Lab Values Calculator Help Pause
Exam Section: Item 26 of 50 National Board of Medical Examiners
■ Mark Clinical Neurology Self-Assessment

26. A 72-year-old man is broug ht to the emergency department because of a decreased level of consciousness for the past 6 hours. Th ree days ago,
he had fever, shortness of breath , and productive coug h treated with an antibiotic, but his symptoms did not improve. On arrival, his temperature is
39°C (102.2°F), pulse is 110/mi n, respirations are 28/min, and blood pressure is 11 0/75 mm Hg. Breath sounds are decreased over the ri ght
midlung field. On neu rolog ic exam ination, he is unarousable but responds to tactile stimuli by moaning. Cranial nerves are intact. There is
resistance to passive flexion of the neck. Which of the following is the most likely pathogen?

A) Herpes simplex virus 1


B) Listeria monocytogenes
C) Pseudomonas aeruginosa
D) Streptococcus pneumoniae
E) Toxop/asma gondii
Correct Answer: D.

This patient's signs and symptoms of fever, obtundation, and meningismus with pneumonia are suggestive of bacterial meningitis,
which results from bacterial infection and resultant inflammation of the meninges surrounding the central nervous system. Among adults
of all ages, Streptococcus pneumoniae, an a-hemolytic, Gram-positive diplococcus, is the most common causative pathogen of
bacterial meningitis. Less common agents of bacterial meningitis in adults include Neisseria meningitidis and Listeria monocytogenes.
Findings of bacterial meningitis include fever, altered mental status, headache, and meningeal signs such as neck stiffness, and
present Brudzinski or Kernig signs. Streptococcal meningitis is commonly distinguished from meningococcal meningitis by the absence
of a petechial rash. The diagnosis is confirmed through lumbar puncture, demonstrating the presence of bacterial organisms and
polymorphonuclear cells, a decreased concentration of glucose, and an increased concentration of protein in the cerebrospinal fluid
(CSF). Treatment includes supportive therapy and broad-spectrum antibiotics, typically with a third-generation cephalosporin and
vancomycin. The addition of ampicillin to this regimen is necessary if Listeria is suspected as a causative agent. Even with appropriate
therapy, bacterial meningitis causes substantial morbidity and mortality. Glucocorticoids may be co-administered which may prevent
complications such as resultant cranial nerve dysfunction.

Incorrect Answers: A, B, C, and E.

Herpes simplex virus 1 (Choice A) presents similarly to bacterial meningitis but is less likely in this patient given his recent history of
lobar pneumonia. Viral meningitis can be distinguished from bacterial meningitis by a normal concentration of glucose, the presence of
lymphocytes, and the absence of bacterial organisms within CSF.

Listeria monocytogenes (Choice B) is a Gram-positive bacillus that may cause bacterial meningitis in infants and elderly adults and
exhibits characteristic tumbling motility on light microscopy. It is transm itted through contaminated foods, such as soft cheeses or cold ....
- - -- - - - - ---- - - - - - - - - - - - - - -- - - -- - - -- - - - - - - - - -- - - - - - - - - - - - - -- - -- - -
r ~, ~ ~ r,
Previous Next Score Report
https://t.me/USMLENBME2CK Lab Values Calculator Help Pause
Exam Section: Item 26 of 50 National Board of Medical Examiners
■ Mark Clinical Neurology Self-Assessment
...
Correct Answer: D.

This patient's signs and symptoms of fever, obtundation, and meningismus with pneumonia are suggestive of bacterial meningitis,
which results from bacterial infection and resultant inflammation of the meninges surrounding the central nervous system. Among adults
of all ages, Streptococcus pneumoniae, an a-hemolytic, Gram-positive diplococcus, is the most common causative pathogen of
bacterial meningitis. Less common agents of bacterial meningitis in adu lts include Neisseria meningitidis and Listeria monocytogenes.
Findings of bacterial meningitis include fever, altered mental status, headache, and meningeal signs such as neck stiffness, and
present Brudzinski or Kernig signs. Streptococcal meningitis is commonly distinguished from meningococcal meningitis by the absence
of a petechial rash. The diagnosis is confirmed through lumbar puncture, demonstrating the presence of bacterial organisms and
polymorphonuclear cells, a decreased concentration of glucose, and an increased concentration of protein in the cerebrospinal fluid
(CSF). Treatment includes supportive therapy and broad-spectrum antibiotics, typically with a third-generation cephalosporin and
vancomycin. The addition of ampicillin to this regimen is necessary if Listeria is suspected as a causative agent. Even with appropriate
therapy, bacterial meningitis causes substantial morbidity and mortality. Glucocorticoids may be co-administered which may prevent
complications such as resu ltant cranial nerve dysfunction.

Incorrect Answers: A, B, C, and E.

Herpes simplex virus 1 (Choice A) presents similarly to bacterial meningitis but is less likely in this patient given his recent history of
lobar pneumonia. Viral meningitis can be distinguished from bacterial meningitis by a normal concentration of glucose, the presence of
lymphocytes, and the absence of bacterial organisms within CSF.

Listeria monocytogenes (Choice B) is a Gram-positive baci llus that may cause bacterial meningitis in infants and elderly adu lts and
exhibits characteristic tumbling motility on light microscopy. It is transmitted through contaminated foods, such as soft cheeses or cold
deli meats. It presents with typical signs and symptoms of meningitis. Listeria meningitis has a high mortality rate.

Pseudomonas aeruginosa (Choice C) is a Gram-negative, aerobic bacil lus that causes a variety of infections, including pneumonia,
urinary tract infection, osteomyelitis, and bacteremia. It is an infrequent cause of bacterial meningitis.

Toxop/asma gondii (Choice E) may cause central nervous system infections in immunocompromised adults. Acquired toxoplasmosis
presents with characteristic multiple, ring-enhancing cerebral lesions on MRI. T. gondii is an infrequent cause of meningitis.

Educational Objective: Among adults of all ages, Streptococcus pneumoniae, an a-hemolytic, Gram-positive diplococcus, is the most
common causative pathogen of bacterial meningitis, which presents with fever, altered mental status, headache, and meningeal signs
such as neck stiffness, and present Brudzinski or Kernig signs. Treatment includes supportive therapy and broad-spectrum antibiotics,
typically with a third-generation cephalosporin and vancomycin

....

r ~, ~ ~ r,
Previous Next Score Report
https://t.me/USMLENBME2CK Lab Values Calculator Help Pause
Exam Section: Item 27 of 50 National Board of Medical Examiners
■ Mark Clinical Neurology Self-Assessment

27. A 37-year-old woman comes to the physician because she has stumbled several times during the past 4 days. She has also noticed intermittent
numbness of her left shin and foot. She has no history of serious illness and takes no medications. She delivered her two children after
uncomplicated pregnancies 8 and 10 years ago. Vital signs are within normal limits. Muscle strength is 3/5 on dorsiflexion of the left foot. Range of
motion of the hip does not produce any symptoms. Deep tendon reflexes are normal. Sensation to pinprick is decreased over the lateral aspect of
the left shin and the dorsal surface of the left foot. The remainder of the examination shows no abnormalities. Which of the following is the most
appropriate initial recommendation?

A) Avoiding sitting with crossed legs


B) Starting an exercise program
C) Wearing flat shoes that fit comfortably
D) Placing an orthotic pad in the left shoe
E) Taking nonsteroidal anti-inflammatory drugs
Correct Answer: A.

Peripheral nerves, particularly in areas that are not protected by surrounding soft tissue, are vulnerable to injury. One such nerve is the
peroneal nerve, which provides sensory innervation through its superficial branch to the dorsal foot and lateral shin and motor
innervation through its deep branch to the tibialis anterior muscle, the primary dorsiflexor of the ankle joint. The peroneal nerve is
relatively superficial as it wraps around the lateral aspect of the proximal fibula, before dividing into deep and superficial branches.
Because of its location, compression of the peroneal nerve potentially occurs when certain positions are kept for a prolonged period,
such as sitting with crossed legs or lying in a lateral decubitus position on a hard surface. Prolonged compression of the peroneal nerve
will potentially cause damage and dysfunction of the nerve, which may manifest as diminished strength in ankle dorsiflexion, as seen in
this patient. Sensory loss and paresthesias over the dorsal foot and lateral shin are also usually present. Treatment is to relieve the
source of compression of the nerve, typically by using cushioning while sleeping or avoiding sitting with crossed legs during the day.
During the recovery period, patients should also be given an ankle-foot orthosis to keep the foot dorsiflexed until motor strength has
recovered.

Incorrect Answers: B, C, D, and E.

Starting an exercise program (Choice B) will not help to relieve the peroneal nerve compression and may be dangerous before
addressing the associated motor weakness because of the risk for fall or injury. Only after the source of peroneal compression has
been removed and an appropriate orthosis has been placed is walking beneficial to recovery.

Neither wearing flat shoes that fit comfortably (Choice C) nor placing an orthotic pad in the left shoe (Choice D) will address the
underlying compression of the peroneal nerve causing the patient's motor and sensory deficits. Shoes with a supportive arch should be ....
- - - -- -- - - - - - - - - ----- - - - -- - ---
r ~, ~ ~ r,
Previous Next Score Report
https://t.me/USMLENBME2CK Lab Values Calculator Help Pause
Exam Section : Item 27 of 50 National Board of Medical Examiners
Clinical Neurology Self-Assessment Please Walt
■ Mark

E)
Correct Answer: A.

Peripheral nerves, particularly in areas that are not protected by surrounding soft tissue, are vulnerable to injury. One such nerve is the
peroneal nerve, which provides sensory innervation through its superficial branch to the dorsal foot and lateral shin and motor
innervation through its deep branch to the tibialis anterior muscle, the primary dorsiflexor of the ankle joint. The peroneal nerve is
relatively superficial as it wraps around the lateral aspect of the proximal fibula, before dividing into deep and superficial branches.
Because of its location, compression of the peroneal nerve potentially occurs when certain positions are kept for a prolonged period,
such as sitting with crossed legs or lying in a lateral decubitus position on a hard surface. Prolonged compression of the peroneal nerve
will potentially cause damage and dysfunction of the nerve, which may manifest as diminished strength in ankle dorsiflexion, as seen in
this patient. Sensory loss and paresthesias over the dorsal foot and lateral shin are also usually present. Treatment is to relieve the
source of compression of the nerve, typically by using cushioning while sleeping or avoiding sitting with crossed legs during the day.
During the recovery period, patients should also be given an ankle-foot orthosis to keep the foot dorsiflexed until motor strength has
recovered.

Incorrect Answers: B, C, D, and E.

Starting an exercise program (Choice B) will not help to relieve the peroneal nerve compression and may be dangerous before
addressing the associated motor weakness because of the risk for fall or injury. Only after the source of peroneal compression has
been removed and an appropriate orthosis has been placed is walking beneficial to recovery.

Neither wearing flat shoes that fit comfortably (Choice C) nor placing an orthotic pad in the left shoe (Choice D) will address the
underlying compression of the peroneal nerve causing the patient's motor and sensory deficits. Shoes with a supportive arch should be
recommended over flat shoes without arch support.

Taking nonsteroidal anti-inflammatory drugs (NSAIDs) (Choice E) is unnecessary as there is no inflammatory component to the
patient's disease. Peroneal nerve compression does not typically cause pain and there is no need for use of NSAIDs or any other pain
medication in this case.

Educational Objective: The peroneal nerve, which provides sensory innervation through its superficial branch to the dorsal foot and
lateral shin and motor innervation through its deep branch to the tibialis anterior muscle, the primary dorsiflexor of the ankle joint, may
be compressed as it wraps around the lateral aspect of the proximal fibula. Strategies to avoid its compression, such as avoiding sitting
with crossed legs, are first-line steps in management.

....

r ~, ~ ~ r,
Previous Next Score Report
https://t.me/USMLENBME2CK Lab Values Calculator Help Pause
Exam Section: Item 28 of 50 National Board of Medical Examiners
■ Mark Clinical Neurology Self-Assessment
...

28. An 82-year-old woman with osteoarthritis is brought to the emergency department because of a 4-day history of increasing confusion and
progressive generalized weakness. During the past 2 months, she has fallen several times; there was no loss of consciousness. She has had no
difficulty sleeping. Two weeks ago, hydrochlorothiazide and metoprolol were started for hypertension. Her temperature is 36. 7°C (98°F), pulse is
68/min, and blood pressure is 140/86 mm Hg. Examination shows a right pronator drift. Babinski sign is present on the rig ht. She is drowsy,
inattentive to questions, and gives several inaccurate answers. Which of the following is the most likely explanation for this patient's mental status
changes?

A) Adverse effect of metoprolol


B) Cerebral infarction
C) Herpes encephalitis
D) Hypothyroidism
E) Subdural hematoma
Correct Answer: E.

Head trauma, the use of anticoagu lants, physical abuse, alcohol use disorder, cerebral atrophy, and malignancy are associated with an
increased risk for developing a subdural hematoma (SDH). On imaging, SDH appears as a crescent-shaped collection abutting the
internal surface of the skul l, not bound by suture lines. In some cases, the SDH may be large enough to compress adjacent brain
parenchyma; the displacement of the cerebral hemisphere can resu lt in neurologic deficits, altered mental status, seizures, herniation
syndromes, and signs of increased intracranial pressure. Patients may manifest upper motoneuron examination findings such as
spasticity, hyperreflexia, and extensor plantar reflex (Babinski sign). Elderly patients are at risk for cerebral atrophy, which places
traction on bridging cortical veins. In this setting, minor trauma such as falls can result in shearing of the veins leading to hemorrhage.
Untreated, the SDH may continue to grow in size. The intracranial compartment is a fixed space within a rigid sku ll; pathologic lesions
such as SDH can result in midline shift and herniation of central nervous system structures if large enough. When such mass effect is
present or at risk of developing, surgical interventions such as decompressive burr holes or a craniotomy may be indicated to evacuate
the hematoma.

Incorrect Answers: A, B, C, and D.

Orthostatic hypotension can occur as an adverse effect of metoprolol (Choice A), a ~-adrenergic blocker that decreases blood pressure
and pulse. Orthostatic hypotension can cause presyncope, syncope, or lightheadedness that is intermittent and positional. This
patient's mental status changes are constant, and she is normotensive, not hypotensive.

Cerebral infarction (Choice B), or stroke, can present with a sudden change in mental status, as wel l as focal neurologic deficits and

r . --
~,
upper motoneuron signs. However, symptoms of cerebral infarction are typically acute, not progressive over days such as in this
- -- - - - -- - - - -- - -- -- - -- -- --- -- - -- ---
~ ~
-- -- - - --- -
r,
....

Previous Next Score Report


https://t.me/USMLENBME2CK Lab Values Calculator Help Pause
Exam Section: Item 28 of 50 National Board of Medical Examiners
■ Mark Clinical Neurology Self-Assessment Please Walt

Correct Answer: E. ...

Head trauma, the use of anticoagu lants, physical abuse, alcohol use disorder, cerebral atrophy, and malignancy are associated with an
increased risk for developing a subdural hematoma (SDH). On imaging, SDH appears as a crescent-shaped collection abutting the
internal surface of the skul l, not bound by suture lines. In some cases, the SDH may be large enough to compress adjacent brain
parenchyma; the displacement of the cerebral hemisphere can resu lt in neurologic deficits, altered mental status, seizures, herniation
syndromes, and signs of increased intracranial pressure. Patients may manifest upper motoneuron examination findings such as
spasticity, hyperreflexia, and extensor plantar reflex (Babinski sign). Elderly patients are at risk for cerebral atrophy, which places
traction on bridging cortical veins. In this setting, minor trauma such as falls can result in shearing of the veins leading to hemorrhage.
Untreated, the SDH may continue to grow in size. The intracranial compartment is a fixed space within a rigid sku ll; pathologic lesions
such as SDH can result in midline shift and herniation of central nervous system structures if large enough. When such mass effect is
present or at risk of developing, surgical interventions such as decompressive burr holes or a craniotomy may be indicated to evacuate
the hematoma.

Incorrect Answers: A, B, C, and D.

Orthostatic hypotension can occur as an adverse effect of metoprolol (Choice A), a ~-adrenergic blocker that decreases blood pressure
and pulse. Orthostatic hypotension can cause presyncope, syncope, or lightheadedness that is intermittent and positional. This
patient's mental status changes are constant, and she is normotensive, not hypotensive.

Cerebral infarction (Choice B), or stroke, can present with a sudden change in mental status, as wel l as focal neurologic deficits and
upper motoneuron signs. However, symptoms of cerebral infarction are typically acute, not progressive over days such as in this
patient. Patients typically present with acute focal neurologic deficits, especially on one side of the body, and may have speech
difficulty, dysarthria, or aphasia.

Herpes encephalitis (Choice C) can present in patients of all ages and may present with fever, headache, seizures, focal neurologic
deficits, and/or confusion. Imaging may show inflammation in one or both temporal lobes. This patient is afebrile and SDH is more likely
caused by the history of prior fal ls.

Severe hypothyroidism (Choice D) can present with altered mental status but would be unlikely to cause focal neurologic deficits.
Patients with hypothyroidism typically present with weight gain, fatigue, constipation, cold intolerance, and menstrual irregularities.
Physical examination may disclose bradycardia, dry, edematous skin, and delayed relaxation of deep tendon reflexes. This patient does
not show any signs or symptoms to suggest hypothyroidism.

Educational Objective: Elderly patients are at risk for cerebral atrophy, which causes traction on bridging cortical veins, such that minor
trauma can result in venous injury leading to a subdural hematoma.

....

r ~, ~ ~ r,
Previous Next Score Report
https://t.me/USMLENBME2CK Lab Values Calculator Help Pause
Exam Section: Item 29 of 50 National Board of Medical Examiners
■ Mark Clinical Neurology Self-Assessment
...

29. A previously healthy 58-year-old man is adm itted to the hospital because of three episodes of amaurosis fugax in the right eye and two episodes of
temporary weakness of the left hand and arm over the past 3 weeks. Which of the following is the most likely cause?

A) Occlusion of the right common carotid artery


B) Occlusion of the left internal carotid artery
C) Occlusion of the right vertebral artery
D) Ulcerative plaq ue of the basilar artery
E) Ulcerative plaque of the right internal carotid artery
Correct Answer: E.

Amaurosis fugax describes brief episodes of transient, unilateral, painless vision loss and is indicative of retinal ischemia caused by
microembolization of plaque within the ophthalmic artery. It often precedes the onset of central retinal artery occlusion, which is
considered an ophthalmic stroke. Amaurosis fugax should prompt an emergent evaluation for sources of potential emboli, the majority
of which originate from the carotid arteries and cardiac valves. This patient's amaurosis fugax and contralateral motor weakness are
suggestive of transient ischemic attacks affecting the right ophthalmic artery and the primary motor cortex, likely caused by an
ulcerative plaque of the right internal carotid artery. Evaluation should include prompt carotid duplex ultrasonography, as well as
echocardiography. Treatment of carotid vascular or cardiac valvu lar disease can prevent central retinal arterial occlusion, which may
cause irreversible blindness, and stroke.

Incorrect Answers: A, B, C, and D.

Occlusion of the right common carotid artery (Choice A) or left internal carotid artery (Choice B) wi ll cause ischemic stroke, rather than
transient ischemic attack. Occlusion of the left internal carotid artery would cause vision loss affecting the left, rather than the right eye.

Occlusion of the right vertebral artery (Choice C) wi ll cause a stroke of the posterior circu lation and present with ataxia, dysphagia,
nausea, vertigo, nystagmus, and may include vision loss. Vision loss caused by ischemic stroke is not transient and is usually not
reversible. Vertebral artery occlusion does not typically present with amaurosis fugax, which is more suggestive of ophthalmic artery
microembolization.

Ulcerative plaque of the basilar artery (Choice D) may resu lt in basilar ischemia or stroke, which may present with truncal ataxia,
diplopia, nystagmus, locked-in state, or coma. Basi lar artery plaque embolization does not typically present with amaurosis fugax,
which is more suggestive of ophthalmic artery embolization.

Educational Objective: Amaurosis fugax describes brief episodes of transient, unilateral vision loss and is indicative of retinal ischemia ....
-- - - -- -- - -- - -- - -- - - - -- -- - - --- -- - -- -- -- - --- - -
r ~, ~ ~ r,
Previous Next Score Report
https://t.me/USMLENBME2CK Lab Values Calculator Help Pause
Exam Section: Item 29 of 50 National Board of Medical Examiners
■ Mark Clinical Neurology Self-Assessment Please Walt
i W I 1 · I . • • . • . • • I i • • J.

C) Occlusion of the right vertebral artery


D) Ulcerative plaq ue of the basilar artery
E) Ulcerative plaque of the right internal carotid artery
Correct Answer: E.

Amaurosis fugax describes brief episodes of transient, unilateral, painless vision loss and is indicative of retinal ischemia caused by
microembolization of plaque within the ophthalmic artery. It often precedes the onset of central retinal artery occlusion, which is
considered an ophthalmic stroke. Amaurosis fugax should prompt an emergent evaluation for sources of potential emboli, the majority
of which originate from the carotid arteries and cardiac valves. This patient's amaurosis fugax and contralateral motor weakness are
suggestive of transient ischemic attacks affecting the right ophthalmic artery and the primary motor cortex, likely caused by an
ulcerative plaque of the right internal carotid artery. Evaluation should include prompt carotid duplex ultrasonography, as well as
echocardiography. Treatment of carotid vascular or cardiac valvu lar disease can prevent central retinal arterial occlusion, which may
cause irreversible blindness, and stroke.

Incorrect Answers: A, B, C, and D.

Occlusion of the right common carotid artery (Choice A) or left internal carotid artery (Choice B) will cause ischemic stroke, rather than
transient ischemic attack. Occlusion of the left internal carotid artery would cause vision loss affecting the left, rather than the right eye.

Occlusion of the right vertebral artery (Choice C) wi ll cause a stroke of the posterior circu lation and present with ataxia, dysphagia,
nausea, vertigo, nystagmus, and may include vision loss. Vision loss caused by ischemic stroke is not transient and is usually not
reversible. Vertebral artery occlusion does not typically present with amaurosis fugax, which is more suggestive of ophthalmic artery
microembolization.

Ulcerative plaque of the basilar artery (Choice D) may resu lt in basilar ischemia or stroke, which may present with truncal ataxia,
diplopia, nystagmus, locked-in state, or coma. Basi lar artery plaque embolization does not typically present with amaurosis fugax,
which is more suggestive of ophthalmic artery embolization.

Educational Objective: Amaurosis fugax describes brief episodes of transient, unilateral vision loss and is indicative of retinal ischemia
caused by emboli. In concomitant motor weakness, amaurosis fugax is suggestive of transient ischemic attack secondary to an
ulcerative plaque of the ipsilateral internal carotid artery. Patients who present with amaurosis fugax should undergo an emergent
evaluation with carotid duplex ultrasonography and echocardiography to evaluate for the definitive source.

.,.

r ~, ~ ~ r,
Previous Next Score Report
https://t.me/USMLENBME2CK Lab Values Calculator Help Pause
Exam Section: Item 30 of 50 National Board of Medical Examiners
■ Mark Clinical Neurology Self-Assessment
...

30. A previously healthy 18-year-old man is brought to the emergency department because of facial weakness and ear pain that began this morning.
When he awoke th is morning, he looked in the mirror and noticed that his face appeared twisted. While he was eating breakfast, milk drooled from
the left side of his mouth. The ear pain has not been relieved by acetaminophen. Examination shows weakness of both the upper and lower face
on the left. Sensation over the face is normal. Which of the following is the most likely location of the abnormality?

A) Anterior horn cell


B) Muscle
C) Myoneural junction
D) Nerve
E) Nerve root
Correct Answer: D.

An abnormality of the facial nerve is the most likely cause of this patient's left facial droop. Involvement of the forehead strongly
suggests a peripheral nerve injury as opposed to a central nervous system injury, such as a cerebral infarction, which wou ld cause an
upper motoneuron pattern of facial weakness, sparing the muscles of the forehead. When complete ipsilateral facial nerve palsy occurs
in isolation, it is known as Bel l palsy, which is thought to most commonly be related to the reactivation of herpes simplex virus, though it
is often idiopathic. It is typically a benign condition that gradually improves over time, although treatment with oral steroids and
occasionally with acyclovir is indicated to expedite recovery. The facial nerve, also known as cranial nerve VII, provides motor
innervation to the muscles of the face, parasympathetic innervation to the lacrimal, submandibular, and sublingual salivary glands, and
sensory innervation to the distal part of the tongue and the external auditory canal. Abnormalities of hearing and taste can also occur
but typically improve over time.

Incorrect Answers: A, B, C, and E.

Anterior horn cells (Choice A), also known as alpha-motoneurons, are found in the anterior horns of the spinal cord and course along a
specific spinal nerve to innervate skeletal muscles. Injury to anterior horn cel ls causes skeletal muscle weakness of the upper or lower
extremities.

Muscle (Choice B) is not the correct answer. The facial nerve innervates numerous facial muscles, and it is the loss of innervation that
causes weakness; the facial muscles do not exhibit any particular pathology.

Myoneural junction (Choice C), also known as the neuromuscular junction (NMJ), is the area where nerve fibers make contact with
skeletal muscle. Diseases that affect the NMJ are classified as presynaptic (myasthenic syndrome) or postsynaptic (myasthenia
gravis). These diseases typically present with diffuse weakness. Patients with Bell palsy have an intact NMJ.
....

r ~, ~ ~ r,
Previous Next Score Report
https://t.me/USMLENBME2CK Lab Values Calculator Help Pause
Exam Section: Item 30 of 50 National Board of Medical Examiners
■ Mark Clinical Neurology Self-Assessment

E) Nerve root
...
Correct Answer: D.

An abnormality of the facial nerve is the most likely cause of this patient's left facial droop. Involvement of the forehead strongly
suggests a peripheral nerve injury as opposed to a central nervous system injury, such as a cerebral infarction, which wou ld cause an
upper motoneuron pattern of facial weakness, sparing the muscles of the forehead. When complete ipsilateral facial nerve palsy occurs
in isolation, it is known as Bel l palsy, which is thought to most commonly be related to the reactivation of herpes simplex virus, though it
is often idiopathic. It is typically a benign condition that gradually improves over time, although treatment with oral steroids and
occasionally with acyclovir is indicated to expedite recovery. The facial nerve, also known as cranial nerve VII, provides motor
innervation to the muscles of the face, parasympathetic innervation to the lacrimal, submandibular, and sublingual salivary glands, and
sensory innervation to the distal part of the tongue and the external auditory canal. Abnormalities of hearing and taste can also occur
but typically improve over time.

Incorrect Answers: A, B, C, and E.

Anterior horn cells (Choice A), also known as alpha-motoneurons, are found in the anterior horns of the spinal cord and course along a
specific spinal nerve to innervate skeletal muscles. Injury to anterior horn cel ls causes skeletal muscle weakness of the upper or lower
extremities.

Muscle (Choice B) is not the correct answer. The facial nerve innervates numerous facial muscles, and it is the loss of innervation that
causes weakness; the facial muscles do not exhibit any particular pathology.

Myoneural junction (Choice C), also known as the neuromuscular junction (NMJ), is the area where nerve fibers make contact with
skeletal muscle. Diseases that affect the NMJ are classified as presynaptic (myasthenic syndrome) or postsynaptic (myasthenia
gravis). These diseases typically present with diffuse weakness. Patients with Bell palsy have an intact NMJ.

Nerve roots (Choice E) typically describe either the anterior (motor) or dorsal (sensory) components of spinal nerves. The two roots join
to form a spinal nerve that carries both sensory and motor fibers to peripheral tissues. In contrast, cranial nerves do not have distinct
nerve roots.

Educational Objective: Bel l palsy refers to isolated facial nerve palsy that is typical ly secondary to the reactivation of herpes simplex
virus. Patients present with complete ipsilateral facial weakness that involves the forehead and can also experience dull ear pain in
conjunction with taste abnormalities. Treatment is with glucocorticoids for all patients and occasional ly with antivirals. Most patients
improve within several months.

.,.

r ~, ~ ~ r,
Previous Next Score Report
https://t.me/USMLENBME2CK Lab Values Calculator Help Pause
Exam Section : Item 31 of 50 National Board of Medical Examiners
■ Mark Clinical Neurology Self-Assessment

31. A 22-year-old college student has a 4-year history of severe, throbbing, generalized headaches that occur once weekly. The headaches are
preceded by a 20-minute period of seeing bright spots. Which of the following drugs is most likely to be effective in red ucing the frequency of this
patient's headaches?

A) Anticholinerg ic drugs
B) ~-Adrenergic blocking drugs
C) Ergotam ine tartrate
D) Sal icylates
E) Sympathomimetic drugs
Correct Answer: B.

Migraine is a common condition that presents with recurrent primary headaches. Migraine headaches are typically severe, unilateral,
throbbing, and associated with nausea, vomiting, photophobia, and phonophobia. The pain is usually located behind or adjacent to the
eye. Some patients additionally experience an aura before or during the headache, which may present with visual or sensory symptoms
such as transient vision loss followed by the perception of scintillating shapes or lines and/or paresthesia. Migraine triggers include
stress, menstruation, weather changes, fasting, and alcohol. Migraines are postulated to arise from deficient serotonin inhibition of
descending pain pathways. Acute migraine treatment includes triptans (serotonin 1b/1 d agonists) and/or nonsteroidal anti-inflammatory
drugs (NSAIDs). For patients with disabling or frequent migraines, prophylactic treatment to reduce the frequency of migraine
headaches is also recommended and includes ~-adrenergic blocking agents (eg, propranolol, metoprolol), tricyclic antidepressants,
serotonin-norepinephrine reuptake inhibitors, and anticonvulsants such as topiramate or valproate.

Incorrect Answers: A, C, D, and E.

Anticholinergic drugs (Choice A) are utilized to treat chronic obstructive pulmonary disease and overactive bladder and may impair
cognitive function. Anticholinergic drugs are not useful for the prevention of migraine.

Ergotamine tartrate (Choice C) acts as a serotonin agonist on the intracranial vasculature and is utilized in acute migraine treatment.
When used chronical ly, ergot medications may increase the frequency of migraine headaches.

Salicylates (Choice D), such as aspirin, treat acute migraine headaches. Salicylates are not effective for chronic migraine prevention.

Sympathomimetic drugs (Choice E), such as stimulants and monoamine oxidase inhibitors, increase the concentrations of endogenous
catecholamines (epinephrine, norepinephrine, dopamine). These medications would not be helpful in preventing migraines.
....

r ~, r,
- - - - - - A - - - - - I ; - - - - - - - - - - - - - - - - - - - - - - - - - - - - - - - - - - - - - - - - - - - - - - - - - - -- - -

~ ~
Previous Next Score Report
https://t.me/USMLENBME2CK Lab Values Calculator Help Pause
Exam Section : Item 31 of 50 National Board of Medical Examiners
■ Mark Clinical Neurology Self-Assessment Please Walt
...
C) Ergotam ine tartrate
D) Sal icylates
E) Sympathomimetic drugs
Correct Answer: B.

Migraine is a common condition that presents with recurrent primary headaches. Migraine headaches are typically severe, unilateral,
throbbing, and associated with nausea, vomiting, photophobia, and phonophobia. The pain is usually located behind or adjacent to the
eye. Some patients additionally experience an aura before or during the headache, which may present with visual or sensory symptoms
such as transient vision loss followed by the perception of scintillating shapes or lines and/or paresthesia. Migraine triggers include
stress, menstruation, weather changes, fasting, and alcohol. Migraines are postulated to arise from deficient serotonin inhibition of
descending pain pathways. Acute migraine treatment includes triptans (serotonin 1b/1 d agonists) and/or nonsteroidal anti-inflammatory
drugs (NSAIDs). For patients with disabling or frequent migraines, prophylactic treatment to reduce the frequency of migraine
headaches is also recommended and includes ~-adrenergic blocking agents (eg, propranolol, metoprolol), tricyclic antidepressants,
serotonin-norepinephrine reuptake inhibitors, and anticonvulsants such as topiramate or valproate.

Incorrect Answers: A, C, D, and E.

Anticholinergic drugs (Choice A) are utilized to treat chronic obstructive pu lmonary disease and overactive bladder and may impair
cognitive function. Anticholinergic drugs are not useful for the prevention of migraine.

Ergotamine tartrate (Choice C) acts as a serotonin agonist on the intracranial vascu lature and is utilized in acute migraine treatment.
When used chronical ly, ergot medications may increase the frequency of migraine headaches.

Salicylates (Choice D), such as aspirin, treat acute migraine headaches. Salicylates are not effective for chronic migraine prevention.

Sympathomimetic drugs (Choice E), such as stimulants and monoamine oxidase inhibitors, increase the concentrations of endogenous
catecholamines (epinephrine, norepinephrine, dopamine). These medications would not be helpful in preventing migraines.

Educational Objectives: Migraines are typically unilateral and pulsating headaches that are associated with nausea, vomiting,
photophobia, and phonophobia, and may also present with visual or sensory auras. In patients with severe or recurrent migraines, ~-
adrenergic blocking agents, antidepressants, and anticonvulsants are commonly utilized to reduce the frequency of future migraine
headaches. NSAIDs and triptans are common ly utilized to abort acute migraine headaches.

.,.

r ~, ~ ~ r,
Previous Next Score Report
https://t.me/USMLENBME2CK Lab Values Calculator Help Pause
Exam Section: Item 32 of 50 National Board of Medical Examiners
■ Mark Clinical Neurology Self-Assessment

32. A 62-year-old woman comes to the physician because of right-sided facial pain for 3 days and a facial rash for 1 day. She describes the pai n as a
burning uncomfortable feeling in her cheek. She had cold sym ptoms 1 week ago that have now resolved. She has hypertension treated with
hydrochlorothiazide but is otherwise healthy. She cares for her two grandchildren, ages 1 and 3 years, every weekday after school. Examination
shows a vesicu lar rash over the face. Neurologic examination shows no abnormalities. Which of the following is the most appropriate measu re to
prevent transmission to the unvacci nated grandchildren?

A) Avoid contact with the grandmother until the rash has resolved
B) Admi nistration of varicel la vacci ne to the children
C) Acyclovir therapy for the children
D) Acyclovir therapy for the grandmother
E) No preventive measures avai lable
Correct Answer: A.

This patient's facial pain and vesicular eruption is consistent with reactivation of varicella-zoster virus (VZV). VZV, also known as
human herpesvirus-3 (HHV-3), is transmitted via direct contact with infected skin lesions or respiratory secretions. VZV causes two
diseases: varicella and herpes zoster. Varicella, or chickenpox, occurs in immunocompetent children and is characterized by a pruritic,
asynchronous rash with fluid-filled vesicles that crust over and may be transmitted through respiratory secretions or contact with active
skin lesions. Herpes zoster, or shingles, is caused by the recurrence of VZV after it has been reactivated from its dormancy in the
dorsal root ganglion of a spinal nerve. It most commonly presents as a unilateral, dermatomal erythematous patch with overlying clear
to yellow, grouped vesicles. It can be associated with neuropathic pain, which can both precede and outlast the rash. The thoracic
spinal nerves and trigeminal nerve, as in this patient, are particularly common distributions of reactivation. Treatment for herpetic
infections including herpes zoster infections involves drugs that inhibit viral DNA polymerase, classically by guanosine analogs such as
acyclovir, valacyclovir, and famciclovir. Prevention of transmission of herpes zoster includes avoiding contact with affected individuals
while skin lesions are active; once crusted over, they are no longer infectious.

Incorrect Answers: B, C, D, and E.

Administration of varicella vaccine to the children (Choice B) would not prevent transmission of the virus between family members but
would allow an affected child to quickly mount an antibody response and prevent symptom development. The varicella vaccine is
recommended as part of the standard childhood vaccination series.

Acyclovir therapy for the children (Choice C) is unnecessary, as they do not show signs of infection. Prophylactic antiviral treatment is
not effective in preventing transmission. Likewise, while acyclovir therapy for the grandmother (Choice D) may hasten the resolution of
skin lesions and decrease the length of time she may transmit the virus, it is less effective in preventing transmission than avoiding ....
-- - - - - -- - - -
r ~, ~ ~ r,
Previous Next Score Report
https://t.me/USMLENBME2CK Lab Values Calculator Help Pause
Exam Section: Item 32 of 50 National Board of Medical Examiners
■ Mark
... '., .... , ....... . Clinical Neurology Self-Assessment

E) No preventive measures avai lable


Correct Answer: A.

This patient's facial pain and vesicular eruption is consistent with reactivation of varicella-zoster virus (VZV). VZV, also known as
human herpesvirus-3 (HHV-3), is transmitted via direct contact with infected skin lesions or respiratory secretions. VZV causes two
diseases: varicella and herpes zoster. Varicella, or chickenpox, occurs in immunocompetent children and is characterized by a pruritic,
asynchronous rash with fluid-filled vesicles that crust over and may be transmitted through respiratory secretions or contact with active
skin lesions. Herpes zoster, or shing les, is caused by the recurrence of VZV after it has been reactivated from its dormancy in the
dorsal root ganglion of a spinal nerve. It most commonly presents as a unilateral, dermatomal erythematous patch with overlying clear
to yellow, grouped vesicles. It can be associated with neuropathic pain, which can both precede and outlast the rash. The thoracic
spinal nerves and trigeminal nerve, as in this patient, are particularly common distributions of reactivation. Treatment for herpetic
infections including herpes zoster infections involves drugs that inhibit viral DNA polymerase, classically by guanosine analogs such as
acyclovir, valacyclovir, and famciclovir. Prevention of transmission of herpes zoster includes avoiding contact with affected individuals
while skin lesions are active; once crusted over, they are no longer infectious.

Incorrect Answers: B, C, D, and E.

Administration of varicella vaccine to the children (Choice B) would not prevent transmission of the virus between family members but
would allow an affected child to quickly mount an antibody response and prevent symptom development. The varicella vaccine is
recommended as part of the standard childhood vaccination series.

Acyclovir therapy for the children (Choice C) is unnecessary, as they do not show signs of infection. Prophylactic antiviral treatment is
not effective in preventing transmission. Likewise, while acyclovir therapy for the grandmother (Choice D) may hasten the resolution of
skin lesions and decrease the length of time she may transmit the virus, it is less effective in preventing transmission than avoiding
contact altogether.

After the varicella-zoster virus has traveled to the dorsal root ganglion and becomes dormant, there are no measures available (Choice
E) to completely prevent reactivation of the virus. However, there are effective means of preventing transmission once a person shows
symptoms, including limiting contact with affected individuals.

Educational Objective: Varicella zoster virus (VZV) causes herpes zoster, or shing les, after it has been reactivated from its dormancy in
the dorsal root ganglion of a spinal nerve. It most commonly presents as a unilateral, dermatomal erythematous patch with overlying
clear to yellow, grouped vesicles. The cornerstone of transmission prevention is avoiding contact with affected individuals until skin
lesions are crusted over and thus no longer infectious.

.,.

r ~, ~ ~ r,
Previous Next Score Report
https://t.me/USMLENBME2CK Lab Values Calculator Help Pause
Exam Section: Item 33 of 50 National Board of Medical Examiners
■ Mark Clinical Neurology Self-Assessment

33. A 17-year-old girl is brought to the emergency department 30 minutes after striking her forehead on the dashboard in a motor vehicle collision.
Examination shows lethargy and confusion. A CT scan of the head shows a frontal lobe injury. Which of the following is the most likely
manifestation of this injury?

A) Dysdiadochokinesia
B) ldeomotor apraxia
C) Problems with conceptual planning
D) Prosopagnosia
E) Spatial disorientation
Correct Answer: C.

Damage to the frontal lobes typically causes impairment in higher-order functions such as conceptual planning. The frontal lobe is the
largest brain lobe, located anterior to the central sulcus, and is divided into three general areas: the primary motor cortex, the
supplementary and premotor cortices, and the prefrontal cortex. While the former two areas control contralateral motor function, the
prefrontal cortex is an association area unique to humans that mediates motivation, planning, goal-oriented behavior, social behavior,
and personality. Damage to the frontal lobes may occur as a result of trauma, cerebrovascular accidents, tumors, or
neurodegeneration. Patients typical ly present with amotivation, poor judgment and planning, behavioral disinhibition, and personality
changes. Management involves correcting the underlying cause; however, deficits may not be fully reversible.

Incorrect Answers: A, B, D, and E.

Dysdiadochokinesia (Choice A) refers to the inability to perform rapid alternating movements. Dysdiadochokinesia typically resu lts from
damage to the ipsilateral cerebellar hemisphere.

ldeomotor apraxia (Choice B) refers to the inability to execute a motor sequence despite intact motor abilities. The left parietal lobe (eg,
supramarginal gyrus) is the most common site of damage.

Prosopagnosia (Choice D), the inability to recognize faces, typically results from fusiform gyrus damage. The fusiform gyrus is located
at the border of the temporal and occipital lobes.

Spatial disorientation (Choice E), also known as topographical disorientation, refers to the inability to navigate the environment. Spatial
disorientation typically resu lts from damage to the parahippocampal and lingual gyri in the temporal lobe.

Educational Objective: The frontal lobe mediates motor function along with higher-order functions such as motivation, goal-oriented ....
---- - --- --- - ---- - -- - - - - -- - -- -- - - - - - - - - - -- - - -- - -- - - - - - - -- ---- - - - - -- - -- ---
r ~, ~ ~ r,
Previous Next Score Report
https://t.me/USMLENBME2CK Lab Values Calculator Help Pause
Exam Section: Item 33 of 50 National Board of Medical Examiners
■ Mark Clinical Neurology Self-Assessment
- -- • .. -- · - - - . -
man ifestation of th is injury?
.. - - -- • .. - - - -- . . . - - • .. - - . ...

A) Dysd iadochokinesia
B) ldeomotor apraxia
C) Problems with conceptual planning
D) Prosopagnosia
E) Spatial disorientation
Correct Answer: C.

Damage to the frontal lobes typically causes impairment in higher-order functions such as conceptual planning. The frontal lobe is the
largest brain lobe, located anterior to the central sulcus, and is divided into three general areas: the primary motor cortex, the
supplementary and premotor cortices, and the prefrontal cortex. While the former two areas control contralateral motor function, the
prefrontal cortex is an association area unique to humans that mediates motivation, planning, goal-oriented behavior, social behavior,
and personality. Damage to the frontal lobes may occur as a result of trauma, cerebrovascular accidents, tumors, or
neurodegeneration. Patients typically present with amotivation, poor judgment and planning, behavioral disinhibition, and personality
changes. Management involves correcting the underlying cause; however, deficits may not be fully reversible.

Incorrect Answers: A, B, D, and E.

Dysdiadochokinesia (Choice A) refers to the inability to perform rapid alternating movements. Dysdiadochokinesia typically resu lts from
damage to the ipsilateral cerebellar hemisphere.

ldeomotor apraxia (Choice B) refers to the inability to execute a motor sequence despite intact motor abilities. The left parietal lobe (eg,
supramarginal gyrus) is the most common site of damage.

Prosopagnosia (Choice D), the inability to recognize faces, typically results from fusiform gyrus damage. The fusiform gyrus is located
at the border of the temporal and occipital lobes.

Spatial disorientation (Choice E), also known as topographical disorientation, refers to the inability to navigate the environment. Spatial
disorientation typically resu lts from damage to the parahippocampal and lingual gyri in the temporal lobe.

Educational Objective: The frontal lobe mediates motor function along with higher-order functions such as motivation, goal-oriented
behavior, and social behavior. Frontal lobe lesions can lead to problems with conceptual planning, behavioral disinhibition, and
personality changes.

.,.

r ~, ~ ~ r,
Previous Next Score Report
https://t.me/USMLENBME2CK Lab Values Calculator Help Pause
Exam Section: Item 34 of 50 National Board of Medical Examiners
■ Mark Clinical Neurology Self-Assessment

34. A 57-year-old woman is brought to the physician because of a 4-week history of progressive ti ngling in her hands and feet. She says she has also
had decreased sensation in her feet, "like I'm walking on sponges," and a painless tingling sensation down her back and legs when she flexes her
neck. She has not had weakness or changes in bowel and bladder f unction. She was diagnosed with stage Il l ovarian cancer 6 months ago and
has been treated with intravenous cisplatin and etoposide month ly. She has hypothyroidism treated with levothyroxine. Muscle strength is normal.
Deep tendon reflexes are absent. Babinski sign is absent. Sensation to vibration is decreased in the fingertips and absent below the knees, and
sensation to pinprick is decreased in a stocking distribution. Proprioception is decreased in the fingers and toes. Romberg sign is present. Her gait
is slightly wide based. Coord ination is normal. Wh ich of the following is the most likely cause of this patient's neurologic symptoms?

A) Guillain-Barre syndrome
B) Hypothyroidism
C) Meningeal carcinoma
D) Multiple sclerosis
E) Toxic neuropathy
Correct Answer: E.

Toxic neuropathy secondary to cisplatin therapy is the most likely diagnosis. The stocking-glove distribution of sensory abnormalities in
addition to loss of lower extremity reflexes and proprioception, with a normal Babinski and normal motor function is diagnostic of
peripheral neuropathy. While peripheral neuropathy can suggest a wide range of underlying pathologies including uncontrolled diabetes
mellitus, amyloidosis, and chronic hepatitis, among others, patients with new-onset peripheral neuropathy should always be questioned
about exposure to platinum-based chemotherapy agents, which exhibit a dose-dependent toxic effect on peripheral nerves.
Unfortunately, the effect is typically irreversible, although some patients may experience gradual improvement of symptoms over many
years. Paraneoplastic syndromes, which are commonly seen in small cell lung cancer (SCLC), can mimic cisplatin-induced neuropathy,
but this patient does not have a history of SCLC, and does not appear to have any evidence of a metastatic malignancy.

Incorrect Answers: A, B, C, and D.

Guillain-Barre syndrome (Choice A) is typically a self-limited demyelinating polyneuropathy that starts with the distal lower extremities
and progresses proximally. Patients usually have normal sensation but profound muscle weakness. This patient's neurologic findings
are not consistent with this diagnosis.

Hypothyroidism (Choice B) can also cause a symmetric, distal, stocking-g love type of peripheral neuropathy, but other symptoms of
hypothyroidism including fatigue, weight gain, cold intolerance, hair loss, and hypertension would also be expected.

Meningeal carcinoma (Choice C) refers to metastatic disease that has spread to the meninges, which can occasionally occur with ....
- - -- ----- •- -- - - -- - ---- - - -- - -- --- - -- -------- -- -- --- - - -- -
r ~, ~ ~ r,
Previous Next Score Report
https://t.me/USMLENBME2CK Lab Values Calculator Help Pause
Exam Section: Item 34 of 50 National Board of Medical Examiners
■ Mark Clinical Neurology Self-Assessment
- . - - -- - - - ...
D) Multiple sclerosis
E) Toxic neuropathy
Correct Answer: E.

Toxic neuropathy secondary to cisplatin therapy is the most likely diagnosis. The stocking-glove distribution of sensory abnormalities in
addition to loss of lower extremity reflexes and proprioception, with a normal Babinski and normal motor function is diagnostic of
peripheral neuropathy. While peripheral neuropathy can suggest a wide range of underlying pathologies including uncontrolled diabetes
mellitus, amyloidosis, and chronic hepatitis, among others, patients with new-onset peripheral neuropathy should always be questioned
about exposure to platinum-based chemotherapy agents, which exhibit a dose-dependent toxic effect on peripheral nerves.
Unfortunately, the effect is typically irreversible, although some patients may experience gradual improvement of symptoms over many
years. Paraneoplastic syndromes, which are commonly seen in small cell lung cancer (SCLC), can mimic cisplatin-induced neuropathy,
but this patient does not have a history of SCLC, and does not appear to have any evidence of a metastatic malignancy.

Incorrect Answers: A, B, C, and D.

Guillain-Barre syndrome (Choice A) is typically a self-limited demyelinating polyneuropathy that starts with the distal lower extremities
and progresses proximally. Patients usually have normal sensation but profound muscle weakness. This patient's neurologic findings
are not consistent with this diagnosis.

Hypothyroidism (Choice B) can also cause a symmetric, distal, stocking-glove type of peripheral neuropathy, but other symptoms of
hypothyroidism including fatigue, weight gain, cold intolerance, hair loss, and hypertension would also be expected.

Meningeal carcinoma (Choice C) refers to metastatic disease that has spread to the meninges, which can occasionally occur with
ovarian cancer. Patients typically present with cranial neuropathies, headache, seizures, and diplopia.

Multiple sclerosis (Choice D) is an autoimmune demyelinating disease that most commonly presents in white women around the age of
40 to 50 years. It is characterized by central nervous system demyelination of white matter in both the brain and the spinal cord. This
can lead to a variety of neurologic deficits, including weakness, hyperreflexia, and spasticity. Optic neuritis is a classic presenting
diagnosis. This patient's presentation is not consistent with this diagnosis.

Educational Objective: Cisplatin-induced peripheral neuropathy is dose-dependent and presents with symmetric, distal sensory
neuropathy, with absent reflexes and normal motor function. There is no effective treatment, although some patients may recover over
a period of years.

....

r ~, ~ ~ r,
Previous Next Score Report
https://t.me/USMLENBME2CK Lab Values Calculator Help Pause
Exam Section: Item 35 of 50 National Board of Medical Examiners
■ Mark Clinical Neurology Self-Assessment

35. A 6-month-old girl is brought to the physician for a follow-up examination after two audiometry screen ings showed hearing loss. She was born at
term to a 27-year-old prim igravid woman after an uncomplicated pregnancy. Examination of the infant today shows no abnormalities. Audiometry
today shows sign ificant hearing loss bilaterally. Which of the following is the most likely cause of th is infant's condition?

A) Cytomegalovirus infection
B) Group B ~-streptococcal infection
C) Herpes simplex
D) Parvovirus infection
E) Respiratory syncytial virus
Correct Answer: A.

Congenital cytomegalovirus (CMV) infection presents with hearing loss, seizures, petechial rash, and intracranial calcifications in the
neonate. It is the most common cause of nonhereditary sensorineural hearing loss. In about 10% of cases, hearing loss may be an
isolated finding without other symptoms of infection present. Hearing loss is also the most common sequela of an initially asymptomatic
neonate. Pregnant women most often contract CMV through exposure from young chi ldren, often through daycare centers. A primary
maternal infection has a high risk for vertical transmission via the placenta to the fetus. An infected fetus may show findings on
ultrasound (eg, periventricular calcifications or ventriculomegaly) or may be born prematurely. In adults, CMV causes either
asymptomatic infection or mononucleosis-like il lness in healthy individuals, and a variety of opportunistic infections in
immunocompromised patients.

Incorrect Answers: B, C, D, and E.

Group B ~-streptococcal infection (Choice B) is a common cause of neonatal bacteremia, meningitis, and pneumonia. It is acquired as
the neonate passes through the birth canal. It does not cause sensorineural hearing loss.

Herpes simplex virus (HSV) (Choice C) may be transmitted from mother to neonate if active lesions are present during a vaginal
delivery. Neonatal infection may be localized to the skin, involve the nervous system and the skin, or be widely disseminated. Clinical
manifestations of HSV meningoencephalitis include seizures, irritability, lethargy, poor feeding, and temperature instability, but hearing
loss is not typically seen.

Parvovirus infection (Choice D) most frequently causes erythema infectiosum in children. This presents with pink patches on the
bi lateral cheeks of children giving an appearance of slapped cheeks. However, when contracted by a pregnant woman, hydrops fetalis
and fetal loss is a potential complication. Congenital parvovirus infection does not cause hearing loss.

r
.
~,
--- - - - -- - - -- -- - --- - - - -- - - - - -- -- - - - - - - - - - - - - - - - - - - - - -
~ ~
-- - - - - - - - - -
r,
- --- --- ---
....

Previous Next Score Report


https://t.me/USMLENBME2CK Lab Values Calculator Help Pause
Exam Section: Item 35 of 50 National Board of Medical Examiners
■ Mark Clinical Neurology Self-Assessment Please Walt
...
D) Parvoviru s infection
E) Respiratory syncytial virus
Correct Answer: A.

Congenital cytomegalovirus (CMV) infection presents with hearing loss, seizures, petechial rash, and intracranial calcifications in the
neonate. It is the most common cause of nonhereditary sensorineural hearing loss. In about 10% of cases, hearing loss may be an
isolated finding without other symptoms of infection present. Hearing loss is also the most common sequela of an initially asymptomatic
neonate. Pregnant women most often contract CMV through exposure from young chi ldren, often through daycare centers. A primary
maternal infection has a high risk for vertical transmission via the placenta to the fetus. An infected fetus may show findings on
ultrasound (eg, periventricular calcifications or ventriculomegaly) or may be born prematurely. In adults, CMV causes either
asymptomatic infection or mononucleosis-like il lness in healthy individuals, and a variety of opportunistic infections in
immunocompromised patients.

Incorrect Answers: B, C, D, and E.

Group B ~-streptococcal infection (Choice B) is a common cause of neonatal bacteremia, meningitis, and pneumonia. It is acquired as
the neonate passes through the birth canal. It does not cause sensorineural hearing loss.

Herpes simplex virus (HSV) (Choice C) may be transmitted from mother to neonate if active lesions are present during a vaginal
delivery. Neonatal infection may be localized to the skin, involve the nervous system and the skin, or be widely disseminated. Clinical
manifestations of HSV meningoencephalitis include seizures, irritability, lethargy, poor feeding, and temperature instability, but hearing
loss is not typically seen.

Parvovirus infection (Choice D) most frequently causes erythema infectiosum in children. This presents with pink patches on the
bi lateral cheeks of children giving an appearance of slapped cheeks. However, when contracted by a pregnant woman, hydrops fetalis
and fetal loss is a potential complication. Congenital parvovirus infection does not cause hearing loss.

Respiratory syncytial virus (Choice E) is one of the most common causes of bronchiolitis and pneumonia in children younger than age 2
years. It does not cause sensorineural hearing loss.

Educational Objective: Congenital cytomegalovirus (CMV) infection classically presents with sensorineural hearing loss, seizures,
petechial rash, and intracranial calcifications in the neonate. A potential alternative presentation is isolated hearing loss. It is the most
common cause of nonhereditary sensorineural hearing loss.

.,.

r ~, ~ ~ r,
Previous Next Score Report
https://t.me/USMLENBME2CK Lab Values Calculator Help Pause
Exam Section: Item 36 of 50 National Board of Medical Examiners
■ Mark Clinical Neurology Self-Assessment
...

36. A 57-year-old man is broug ht to the physician by his wife because of constant fatigue for 2 weeks. He has consumed 1 pint of vodka daily for
30 years. He appears drowsy. His tem peratu re is 37°C (98.6°F), pulse is 100/min, respirations are 20/min, and blood pressu re is 120/76 mm Hg.
Cardiopulmonary exam ination shows no abnormalities. Abdominal exami nation shows severe distention with a fluid wave. Rectal exam ination
shows hemorrh oids. Test of the stool for occu lt blood is positive. On mental status exam ination, he responds to questions slowly. W hen asked to
write his name, it is an un intel ligible scribble compared with a clear sig nature on his med ical chart from 1 year ago. Laboratory stud ies show:
Hemoglobin 12.5 g/dL
Hematocrit 39%
Leukocyte count 7000/mm 3 with a normal differential
Platelet count 200,000/mm 3
Prothrombin time 18 sec (INR=2)
Partial thromboplastin time 40 sec
Serum
Urea nitrogen 6 mg/dL
Creatinine 0.6 mg/dL
AST 8 U/L
ALT 8 U/L

W hich of the followi ng is most likely to improve this patient's mental status?

A) High-protein, low-carbohyd rate diet


B) Administration of lactu lose
C) Infusion of fresh frozen plasma
D) Transfusion of packed red blood cells
E) Hemodialysis
F) Paracentesis
Correct Answer: B.

Administration of oral lactulose is the intervention that is most likely to improve this patient's altered mental status (AMS). AMS occurs
frequently in patients with decompensated cirrhosis. While the differential diagnosis is broad and includes systemic infection (eg,
urinary tract infection, pneumonia, bacterial peritonitis), intoxication, hypoxemia or hypercapnia, among others, AMS is frequently the
resu lt of hepatic encephalopathy (HE). HE is postulated to resu lt from increased gut generation of ammonia by Gram-negative
organisms coupled with inadequate clearance by the cirrhotic liver. Other clinical findings that suggest hepatic encephalopathy include
the presence of asterixis. Lactu lose is an oral medication given for the treatment and prevention of HE. It is degraded by colonic ....

r ~, ~ ~ r,
Previous Next Score Report
https://t.me/USMLENBME2CK Lab Values Calculator Help Pause
Exam Section: Item 36 of 50 National Board of Medical Examiners
■ Mark Clinical Neurology Self-Assessment

Administration of oral lactulose is the intervention that is most likely to improve this patient's altered mental status (AMS). AMS occurs ...
frequently in patients with decompensated cirrhosis. While the differential diagnosis is broad and includes systemic infection (eg,
urinary tract infection, pneumonia, bacterial peritonitis), intoxication, hypoxemia or hypercapnia, among others, AMS is frequently the
result of hepatic encephalopathy (HE). HE is postulated to result from increased gut generation of ammonia by Gram-negative
organisms coupled with inadequate clearance by the cirrhotic liver. Other clinical findings that suggest hepatic encephalopathy include
the presence of asterixis. Lactu lose is an oral medication given for the treatment and prevention of HE. It is degraded by colonic
bacteria to generate hydrogen ions that create an acidic environment within the gut. The abundance of extra hydrogen ions converts
ammonia (NH 3) to ammonium (NH/), which cannot freely diffuse into the bloodstream and is read ily excreted in the feces. Most
patients demonstrate rapid improvement of their mental status after initiating lactulose therapy. Failure to do so should prompt
evaluation for alternative causes of AMS.

Incorrect Answers: A, C, D, E, and F.

High-protein, low-carbohydrate diet (Choice A) would not resolve this patient's HE. While diets for patients with cirrhosis often include
high protein intake and low salt intake, diet alone would not treat HE.

Infusion of fresh frozen plasma (FFP) (Choice C) would be appropriate for patients with coagulopathy and profound bleeding, as it
includes all the clotting factors in addition to von Willebrand factor and fibrinogen. This patient's guaiac positive stool likely represents
gastritis or ulcers, but his clinical presentation is not consistent with severe bleeding. Additionally, FFP is high volume and can worsen
edema and ascites in cirrhotic patients.

Transfusion of packed red blood cells (Choice D) is appropriate for patients with cirrhosis and massive bleeding from esophageal
varices, or in cases of chronic bleeding when the hemoglobin drops below 7 g/dl. This patient does not meet criteria for transfusion.

Hemodialysis (Choice E) is indicated for refractory hypervolemia, acidosis, electrolyte disturbances including severe hyperkalemia or
hypercalcemia, after ingestion of certain alcohols, or for symptomatic uremia. Dialysis would not improve this patient's encephalopathy.

Paracentesis (Choice F) should be performed on all patients with ascites and associated fever or abdominal pain who are admitted to
the hospital to rule out spontaneous bacterial peritonitis (SBP), but it does not have a role in treating underlying hepatic
encephalopathy. If SBP is discovered, then antibiotics are administered. Additionally, large volume paracentesis can be useful in
patients with tense ascites or volume overload.

Educational Objective: Patients with decompensated cirrhosis who present with altered mental status frequently have hepatic
encephalopathy, which occurs as the result of accumulation of ammonia produced by colonic bacteria. While other causes of
encephalopathy such as infection should be simultaneously evaluated, initiating treatment with oral lactulose is warranted. Lactulose is
degraded by colonic bacteria to create an acidic environment, thereby converting ammonia to ammonium, which cannot be readily
absorbed across the gut wall. One consequence of lactulose therapy is diarrhea, and dosing is often titrated to three or four bowel
movements per day. ....

r ~, ~ ~ r,
Previous Next Score Report
https://t.me/USMLENBME2CK Lab Values Calculator Help Pause
Exam Section : Item 37 of 50 National Board of Medical Examiners
■ Mark Clinical Neurology Self-Assessment

37. A 32-year-old woman comes to the physician because of weakness of her right arm and leg that began 1 week ago when she awoke in the
morn ing. Six days ago, when she took a bath, she noticed that she was unable to feel the heat of the water on the left side of her body. She had a
mild upper respiratory tract infection 3 weeks ago. Cran ial nerves are intact. There is mild weakness of the right upper and lower extremities.
Sensation to pinprick and tem perature is decreased over the left trunk and left upper and lower extremities; sensation to vibration is decreased
over the right foot. Reflexes in the upper and lower extremities are more brisk on the right than the left. Babinski sign is present on the rig ht. Which
of the following is the most likely location of this patient's lesion?

A) Brai n stem
B) Cauda equina
C) Cerebral hemisphere
D) Cervical nerve root
E) Lum bar nerve root
F) Peripheral nerve
G) Spinal cord
Correct Answer: G.

Brown-Sequard synd rome refers to hemisection of the spinal cord, which damages the anterior and dorsal horns along with the three
major spinal cord tracts: the corticospinal tract (carrying motor fibers), dorsal column-medial lemniscus tract (carrying sensory fibers
that mediate fine touch, pressure, vibration, and proprioception), and spinothalamic tract (carrying sensory fibers that mediate pain and
temperature). The corticospinal tract and dorsal-column medial meniscus tract decussate in the medulla, while the spinothalamic tract
decussates near the spinal cord level of the peripheral sensory nerve. Consequently, hemisection of the spinal cord leads to an
ipsilateral lower motoneuron pattern of dysfunction at the level of the lesion and an ipsilateral upper motoneuron pattern of dysfunction
inferior to the lesion. Further, hemisection of the spinal cord results in an ipsilateral impairment of sensation of fine touch, pressure,
vibration, and proprioception, and a contralateral impairment of sensation of pain and temperature. This patient's lesion is likely at the
cervical spinal cord level given the dysfunction of both lower and upper extremities. Brown-Sequard syndrome is most common ly
caused by penetrating trauma but may also be caused by disc herniation, hematoma, or tumor. Treatment depends on the underlying
cause, though it is typically focused on physical rehabilitation.

Incorrect Answers: A, B, C, D, E, and F.

Brain stem (Choice A) and cerebral hemisphere (Choice C) lesions affecting the three major sensory and motor tracts present with
contralateral hemisensory deficits and hemiparesis caused by the decussation of all three tracts inferior to the lesion. Brainstem lesions
also typically cause ipsilateral cranial nerve dysfunction. ....

r ~, ~ ~ r,
Previous Next Score Report
https://t.me/USMLENBME2CK Lab Values Calculator Help Pause
Exam Section : Item 37 of 50 National Board of Medical Examiners
■ Mark Clinical Neurology Self-Assessment

Correct Answer: G. ...

Brown-Sequard syndrome refers to hemisection of the spinal cord, which damages the anterior and dorsal horns along with the three
major spinal cord tracts: the corticospinal tract (carrying motor fibers), dorsal column-medial lemniscus tract (carrying sensory fibers
that mediate fine touch, pressure, vibration, and proprioception), and spinothalamic tract (carrying sensory fibers that mediate pain and
temperature). The corticospinal tract and dorsal-column medial meniscus tract decussate in the medulla, while the spinothalamic tract
decussates near the spinal cord level of the peripheral sensory nerve. Consequently, hemisection of the spinal cord leads to an
ipsilateral lower motoneuron pattern of dysfunction at the level of the lesion and an ipsilateral upper motoneuron pattern of dysfunction
inferior to the lesion. Further, hemisection of the spinal cord results in an ipsilateral impairment of sensation of fine touch, pressure,
vibration, and proprioception, and a contralateral impairment of sensation of pain and temperature. This patient's lesion is likely at the
cervical spinal cord level given the dysfunction of both lower and upper extremities. Brown-Sequard syndrome is most common ly
caused by penetrating trauma but may also be caused by disc herniation, hematoma, or tumor. Treatment depends on the underlying
cause, though it is typical ly focused on physical rehabilitation.

Incorrect Answers: A, B, C, D, E, and F.

Brain stem (Choice A) and cerebral hemisphere (Choice C) lesions affecting the three major sensory and motor tracts present with
contralateral hemisensory deficits and hemiparesis caused by the decussation of al l three tracts inferior to the lesion. Brainstem lesions
also typically cause ipsilateral cranial nerve dysfunction.

Cauda equina (Choice B) syndrome refers to the compression of multiple nerve roots of the cauda equina by disc herniation, lumbar
spondylosis, epidural abscess, or epidural tumor. Patients typically present with unilateral or bi lateral lower extremity radicular pain and
weakness, bladder and rectal sphincter paralysis, and sensory loss of the lower extremities. This patient instead demonstrates
dysfunction of the upper and lower extremities.

Lesions of a cervical nerve root (Choice D), lumbar nerve root (Choice E), or peripheral nerve (Choice F) typically lead to ipsilateral
motor and/or sensory deficits, not contralateral deficits in pain and temperature sensation. Nerve roots carry motor and sensory fibers.
Lesions of cervical nerve roots typically cause motor and sensory symptoms in an upper body myotome/dermatome, while lesions of
lumbar nerve roots typically cause motor and sensory symptoms in a lower body myotome/dermatome. Peripheral nerves may carry
sensory information only, motor information only, or both motor and sensory information. Thus, lesions may cause combined or isolated
sensory or motor dysfunction.

Educational Objective: Brown-Sequard syndrome refers to hemisection of the spinal cord, which damages the anterior and dorsal horns
along with the three major spinal cord tracts: the corticospinal tract, dorsal column-medial lemniscus tract, and spinothalamic tract. This
leads to an ipsilateral lower motoneuron pattern of dysfunction at the level of the lesion and an ipsilateral upper motoneuron pattern of
dysfunction inferior to the lesion. Further, hemisection of the spinal cord results in an ipsilateral impairment of sensation of fine touch,
pressure, vibration, and proprioception, and a contralateral impairment of sensation of pain and temperature.
....

r ~, ~ ~ r,
Previous Next Score Report
https://t.me/USMLENBME2CK Lab Values Calculator Help Pause
Exam Section: Item 38 of 50 National Board of Medical Examiners
■ Mark Clinical Neurology Self-Assessment

38. A 72-year-old man with a 3-year history of Parkinson disease is brought to the physician by his wife for a follow-up examination. Three weeks ago,
his dosage of carbidopa-levodopa was increased . Since then, he has reported seeing people spying on him from across the street. He has no
other history of serious medical or psychiatric illness and takes no other medications. Physical examination shows a resting tremor. There is
cogwheel rig idity and increased muscle tone. On mental status examination, he reports seeing people spying on him and says his wife is "one of
them ." Addition of which of the following medications is the most appropriate next step in pharmacotherapy?

A) Haloperidol
B) Lorazepam
C) Paroxetine
D) Quetiapine
E) Valproic acid
Correct Answer: D.

Carbidopa-levodopa increases dopamine concentrations in several brain areas. In the basal ganglia, an increased dopamine
concentration leads to improvement in Parkinson symptoms. In the limbic system, an increased dopamine concentration leads to
psychotic symptoms such as paranoid delusions or hallucinations. Distressing psychotic symptoms in Parkinson disease are typically
managed with antipsychotic medication or a decreased dose of carbidopa-levodopa. This patient demonstrates residual Parkinson
symptoms that would likely worsen with a decreased dose of carbidopa-levodopa, so antipsychotic medication should be given. Typical
antipsychotics (eg, haloperidol) potently antagonize the dopamine receptor non-selectively across several brain areas, which could lead
to worsened Parkinson symptoms. Atypical antipsychotics, especially quetiapine and clozapine, less potently antagonize dopamine
receptors and modulate other neurotransmitters such as serotonin (which assist in regulating dopamine signaling in the limbic system
and thus indirectly address psychotic symptoms). Consequently, atypical antipsychotics improve psychotic symptoms without
significantly exacerbating Parkinson symptoms.

Incorrect Answers: A, B, C, and E.

Haloperidol (Choice A) is a typical antipsychotic that potently antagonizes dopamine receptors in both the basal ganglia and limbic
system. Haloperidol would improve psychotic symptoms but would potentially exacerbate the patient's Parkinson symptoms.

Lorazepam (Choice B) is a benzodiazepine utilized for acute anxiety. Paroxetine (Choice C) is a selective serotonin reuptake inhibitor
that treats major depressive disorder and anxiety disorders. Valproic acid (Choice E) is a mood stabilizer utilized for bipolar disorder.
These medications do not address psychotic symptoms.

r ~,
- -- - - - -- - - - .
Educational Objective: Carbidopa-levodopa non-selectively increases dopamine concentrations in the basal ganglia and limbic system,
--- -- - - - - - - -- - - -- - - - - - - -- - - - - - - - - - -- - - -- -
~
-----
~
- .
- - - - - - -- - - -- - -
r,
....

Previous Next Score Report


https://t.me/USMLENBME2CK Lab Values Calculator Help Pause
Exam Section: Item 38 of 50 National Board of Medical Examiners
■ Mark Clinical Neurology Self-Assessment
...
A) Haloperidol
B) Lorazepam
C) Paroxetine
D) Quetiapine
E) Valproic acid
Correct Answer: D.

Carbidopa-levodopa increases dopamine concentrations in several brain areas. In the basal ganglia, an increased dopamine
concentration leads to improvement in Parkinson symptoms. In the limbic system, an increased dopamine concentration leads to
psychotic symptoms such as paranoid delusions or hal lucinations. Distressing psychotic symptoms in Parkinson disease are typically
managed with antipsychotic medication or a decreased dose of carbidopa-levodopa. This patient demonstrates residual Parkinson
symptoms that would likely worsen with a decreased dose of carbidopa-levodopa, so antipsychotic medication should be given. Typical
antipsychotics (eg, haloperidol) potently antagonize the dopamine receptor non-selectively across several brain areas, which could lead
to worsened Parkinson symptoms. Atypical antipsychotics, especially quetiapine and clozapine, less potently antagonize dopamine
receptors and modulate other neurotransmitters such as serotonin (which assist in regu lating dopamine signaling in the limbic system
and thus indirectly address psychotic symptoms). Consequently, atypical antipsychotics improve psychotic symptoms without
significantly exacerbating Parkinson symptoms.

Incorrect Answers: A, B, C, and E.

Haloperidol (Choice A) is a typical antipsychotic that potently antagonizes dopamine receptors in both the basal gang lia and limbic
system. Haloperidol would improve psychotic symptoms but would potentially exacerbate the patient's Parkinson symptoms.

Lorazepam (Choice B) is a benzodiazepine utilized for acute anxiety. Paroxetine (Choice C) is a selective serotonin reuptake inhibitor
that treats major depressive disorder and anxiety disorders. Valproic acid (Choice E) is a mood stabilizer utilized for bipolar disorder.
These medications do not address psychotic symptoms.

Educational Objective: Carbidopa-levodopa non-selectively increases dopamine concentrations in the basal ganglia and limbic system,
leading to improvement of Parkinson symptoms along with the risk for psychotic symptoms, respectively. Psychotic symptoms in
patients with Parkinson disease should be treated with atypical antipsychotics, which are low potency dopamine antagonists, to avoid
the exacerbation of Parkinson symptoms.

.,.

r ~, ~ ~ r,
Previous Next Score Report
https://t.me/USMLENBME2CK Lab Values Calculator Help Pause
Exam Section: Item 39 of 50 National Board of Medical Examiners
■ Mark Clinical Neurology Self-Assessment

39. A 72-year-old woman is brought to the physician by her son because of a 1-month history of progressive forgetfulness. During this period, she has
misplaced her keys and checkbook, and she forgot that she was supposed to meet him for lunch last week. The patient says that she has had
progressive fatigue because of recent difficulty staying asleep at night. She often awakens at 4 or 5 am and is unable to fall back asleep. She
takes zolpidem once or twice weekly to help her sleep, but she has taken it nightly for the past 2 weeks. She also has had a 4.5-kg (10-lb) weight
loss and says that nothing tastes good anymore. She has a 10-year history of esophageal reflux disease treated with lansoprazole and a 2-year
history of arthritis treated with acetaminophen. She was an Eng lish professor at a prominent university until 1 year ago when she was requ ired to
retire by university regulations. She maintained an active schedule of lectu ring and writing until 2 months ago. Her hair is unwashed, and she has a
mild body odor. She is 168 cm (5 ft 6 in) tall and weighs 50 kg (110 lib); BMI is 18 kg/m 2. Her pulse is 72/min, and blood pressure is 125/70 mm Hg.
On mental status examination, she appears distracted and worried. She says that her mood is "okay." Her speech is slow and monotone, and she
answers with single words. She is alert and oriented to person, place, and time. Her digit span is 6 forward and 2 backward. She can recall zero of
three objects after 5 minutes. She refuses to perform simple change calcu lations or name past presidents. Which of the following is the most likely
diagnosis?

A) Adjustment disorder with depressed mood


B) Delirium
C) Dementia, Alzheimer type
D) Dysthymic disorder
E) Major depressive disorden
F) Parkinson disease
G) Substance-induced persisting amnestic disorder
Correct Answer: E.

Major depressive disorder (MOD) explains this patient's fatigue, poor sleep, poor appetite, anhedonia (evidenced by food not tasting
good), poor concentration (evidenced by a decreased digit span and distracted appearance), and psychomotor retardation (evidenced
by slow speech). Symptoms of MOD include 2 or more weeks of depressed mood, anhedonia, feelings of guilt or worthlessness,
difficulty concentrating , suicidal thoughts, and/or neurovegetative symptoms (decreased energy, sleep disturbance, appetite
disturbance). These symptoms disrupt everyday functioning, as evidenced by this patient's poor personal hygiene. Elderly patients may
view their neurovegetative symptoms as a manifestation of normal aging or a comorbid medical condition rather than depression.
Consequently, depressed elderly patients may struggle to express their depressed mood, though MOD should be suspected in this
patient with many other depressive symptoms. MOD in elderly patients can cause pseudodementia, which refers to cognitive
impairment secondary to depression, manifesting as this patient's short-term memory difficulties and decreased effort on a brief
cognitive screening. Physicians should always inquire about suicidal ideation in patients with MOD. Mood and cognition typically
improve with antidepressant medication. ....

r ~, ~ ~ r,
Previous Next Score Report
https://t.me/USMLENBME2CK Lab Values Calculator Help Pause
Exam Section: Item 39 of 50 National Board of Medical Examiners
■ Mark Clinical Neurology Self-Assessment

by slow speech). Symptoms of MOD include 2 or more weeks of depressed mood, anhedonia, feelings of guilt or worthlessness, ...
difficulty concentrating, suicidal thoughts, and/or neurovegetative symptoms (decreased energy, sleep disturbance, appetite
disturbance). These symptoms disrupt everyday functioning, as evidenced by this patient's poor personal hygiene. Elderly patients may
view their neurovegetative symptoms as a manifestation of normal aging or a comorbid medical condition rather than depression.
Consequently, depressed elderly patients may struggle to express their depressed mood, though MOD should be suspected in this
patient with many other depressive symptoms. MOD in elderly patients can cause pseudodementia, which refers to cognitive
impairment secondary to depression, manifesting as this patient's short-term memory difficulties and decreased effort on a brief
cognitive screening. Physicians shou ld always inquire about suicidal ideation in patients with MOD. Mood and cognition typically
improve with antidepressant medication.

Incorrect Answers: A, B, C, D, F, and G.

Adjustment disorder with depressed mood (Choice A) refers to depressive symptoms beginning within three months of a stressor and
associated with a significant impairment in functioning. Dysthymic disorder (Choice D), commonly known as persistent depressive
disorder, refers to depressive symptoms lasting longer than two years. Both disorders may include energy, sleep, and appetite changes
without meeting the criteria of MOD.

Delirium (Choice B) is an acute confusional state typical ly associated with medical illness in older patients. Delirium presents with acute
disturbances in awareness, attention, and baseline cognition that fluctuate in severity over the course of the day. This patient does not
show evidence of an acute medical condition or fluctuating cognitive symptoms, rather the patient demonstrates 1 month of progressive
cognitive decline.

Dementia, Alzheimer type (Choice C) is the most prevalent dementia, presenting with progressive cognitive decline that begins with
short-term memory impairment, progresses to apraxia and language abnormalities, and culminates in behavioral and personality
changes preventing the patient from performing basic activities of daily living. Dementia, Alzheimer type, would not present with this
patient's constellation of neurovegetative symptoms or this degree of functional impairment after on ly one month of symptoms.

Parkinson disease (Choice F) presents with dysfunction of voluntary movements in the form of bradykinesia, resting tremor, rigidity, and
postural instability. Dementia may occur late in the disease course and wou ld not be expected after only 1 month of symptoms.

Substance-induced persisting amnestic disorder (Choice G) refers to anterograde or retrograde memory impairment that persists
beyond the expected period of intoxication or withdrawal. This patient illustrates deficits in cognitive domains (eg, attention) along with
symptoms of depression, making MOD the most likely diagnosis.

Educational Objective: Major depressive disorder typical ly features depressed mood, anhedonia, difficulty concentrating, suicidal
thoughts, and/or neurovegetative symptoms (decreased energy, sleep disturbance, appetite disturbance). Elderly patients may
demonstrate pseudodementia, which refers to cognitive impairment arising from depression. Elderly patients are less likely to endorse a
depressed mood than younger adu lts. ....

r ~, ~ ~ r,
Previous Next Score Report
https://t.me/USMLENBME2CK Lab Values Calculator Help Pause
Exam Section: Item 40 of 50 National Board of Medical Examiners
■ Mark Clinical Neurology Self-Assessment

40. A 77-year-old man with Parkinson disease is brought to the physician by his wife because of a 3-month history of unusual behavior at night. She
says that he yel ls in his sleep and waves his arms as if defending himself. Two nights ago, he bit her and tried to push her out of the bed. Current
med ications include carbidopa-levodopa. Examination shows little facial expression, a pill-rolling tremor of both hands, and muscle rigidity. Which
of the following is the most appropriate next step in diagnosis?

A) Multiple sleep latency testing


B) Overnight pulse oximetry testing
C) PolysomnographYi
D) Sleeping and waking EEG
E) Wrist actigraphy
Correct Answer: C.

Polysomnography assists in diagnosing rapid-eye movement (REM) sleep behavior disorder. In typical REM sleep, several neural
circuits terminate on spinal cord motoneurons to cause sleep atonia. When this muscle atonia is lost, patients can violently act out their
dreams. They typically remember the dreams. REM sleep behavior disorder most commonly occurs in adult males and can be
idiopathic or related to underlying a-synuclein deposition (eg, Parkinson disease, multiple system atrophy, or Lewy body dementia).
Polysomnography tracks several sleep-related parameters (eg, brain waves, eye movements, muscle tone, and breathing patterns).
Polysomnography showing absent atonia during REM sleep confirms the diagnosis. Treatment includes creating a safe sleep
environment and, if the behavior is severe, initiating melatonin or clonazepam. Patients with idiopathic REM sleep behavior disorder
should be closely monitored because of the high rate of developing a neurodegenerative disease.

Incorrect Answers: A, B, D, and E.

Multiple sleep latency testing (Choice A) tracks the amount of time it takes a patient to enter the REM stage. A decreased REM sleep
latency indicates narcolepsy. The timing of REM sleep is relatively unaffected in patients with REM sleep behavior disorder.

Overnight pulse oximetry testing (Choice B) can assist in the diagnosis of sleep apnea. In sleep apnea, patients become hypoxemic
several times during the night because of pharyngeal obstruction or disordered breathing. Oxygenation is typically normal in REM sleep
behavior disorder.

Sleeping and waking EEG (Choice D) may disclose slight abnormalities of slow-wave sleep in patients with REM sleep behavior
disorder. However, these abnormalities have not yet been characterized sufficiently to assist in diagnosis.

Wrist actigraphy (Choice E) involves recording movements during sleep with a wrist device. Wrist actigraphy is being investigated as a ....

r ~, r,
- - - - - - - - - - • I. ' - - - - - - - - - - - - - - - - - - - - - -- - - - - - - - - - - - - - - - - - - - - - - - - - - - - - • ' - - - -

~ ~
Previous Next Score Report
https://t.me/USMLENBME2CK Lab Values Calculator Help Pause
Exam Section: Item 40 of 50 National Board of Medical Examiners
■ Mark Clinical Neurology Self-Assessment
...
D) Sleeping and waking EEG
E) Wrist actigraphy
Correct Answer: C.

Polysomnography assists in diagnosing rapid-eye movement (REM) sleep behavior disorder. In typical REM sleep, several neural
circuits terminate on spinal cord motoneurons to cause sleep atonia. When this muscle atonia is lost, patients can violently act out their
dreams. They typically remember the dreams. REM sleep behavior disorder most commonly occurs in adult males and can be
idiopathic or related to underlying a-synuclein deposition (eg, Parkinson disease, multiple system atrophy, or Lewy body dementia).
Polysomnography tracks several sleep-related parameters (eg, brain waves, eye movements, muscle tone, and breathing patterns).
Polysomnography showing absent atonia during REM sleep confirms the diagnosis. Treatment includes creating a safe sleep
environment and, if the behavior is severe, initiating melatonin or clonazepam. Patients with idiopathic REM sleep behavior disorder
should be closely monitored because of the high rate of developing a neurodegenerative disease.

Incorrect Answers: A, B, D, and E.

Multiple sleep latency testing (Choice A) tracks the amount of time it takes a patient to enter the REM stage. A decreased REM sleep
latency indicates narcolepsy. The timing of REM sleep is relatively unaffected in patients with REM sleep behavior disorder.

Overnight pulse oximetry testing (Choice B) can assist in the diagnosis of sleep apnea. In sleep apnea, patients become hypoxemic
several times during the night because of pharyngeal obstruction or disordered breathing. Oxygenation is typically normal in REM sleep
behavior disorder.

Sleeping and waking EEG (Choice D) may disclose slight abnormalities of slow-wave sleep in patients with REM sleep behavior
disorder. However, these abnormalities have not yet been characterized sufficiently to assist in diagnosis.

Wrist actigraphy (Choice E) involves recording movements during sleep with a wrist device. Wrist actigraphy is being investigated as a
screening tool for REM sleep behavior disorder. However, polysomnography is the gold-standard diagnostic tool for REM sleep
behavior disorder.

Educational Objective: REM sleep behavior disorder features absent muscle atonia during REM sleep. Patients typically act out their
dreams in a coordinated, though sometimes violent, manner. REM sleep behavior disorder typically occurs in adult males and can be
idiopathic or related to underlying a-synuclein deposition (eg, Parkinson disease, multiple system atrophy, or Lewy body dementia).

.,.

r ~, ~ ~ r,
Previous Next Score Report
https://t.me/USMLENBME2CK Lab Values Calculator Help Pause
Exam Section : Item 41 of 50 National Board of Medical Examiners
■ Mark Clinical Neurology Self-Assessment

41. A 42-year-old woman is brought to the emergency department by her girlfriend 15 minutes after several episodes of staring and blinking while
eating dinner. The patient has a history of bipolar disorder well controlled with daily bupropion and lithium carbonate for the past 5 years. She
started taking terbinafine 2 weeks ago for chronic toenail fungus. Duri ng the examination , she is conversant. Her pulse is 64/min, respirations are
16/min, and blood pressure is 140/90 mm Hg. A normal rhythm is heard on cardiac examination. Neurologic examination shows no focal fi ndings.
Which of the following is the most likely explanation for this patient's symptoms?

A) Catatonic episode
B) Glioblastoma multiforme
C) Rapid cycling bipolar disorder
D) Seizure activity;
E) Transient ischem ic attack
Correct Answer: D.

Bupropion is an antidepressant medication, sometimes utilized in bipolar disorder for incomplete response to mood stabi lizer therapy,
that decreases the reuptake of norepinephrine and dopamine. By an unknown mechanism, bupropion decreases the seizure threshold.
Bupropion is a cytochrome P450 (CYP)-2D6 substrate, while terbinafine (an oral antifungal medication utilized for onychomycosis) is a
CYP-2D6 inhibitor. Consequently, this patient's bupropion serum concentrations were likely increased by the initiation of terbinafine.
Most medication-induced seizures are generalized tonic-clonic seizures, but some are absence seizures. Absence seizures are
generalized seizures that cause transient staring episodes with impaired consciousness during the seizure. Management involves
switching to a topical onychomycosis agent or a different antidepressant medication that does not decrease the seizure threshold.

Incorrect Answers: A, B, C, and E.

A catatonic episode (Choice A) is a neurobehavioral syndrome marked by the inability to move or speak, or the inability to stop moving
or speaking repetitively, that typically arises from an exacerbation of a psychiatric disorder. Catatonic episodes typically last days rather
than seconds to minutes.

Glioblastoma multiforme (Choice B) is an aggressive primary brain astrocytoma, which carries a poor prognosis. Seizures are a
common presenting symptom, as are headaches and focal neurologic deficits. However, seizures related to brain tumors are typically
focal rather than generalized, and this patient's recent history of medication changes makes a medication-induced seizure more likely.

Rapid cycling bipolar disorder (Choice C) is defined as at least four mood episodes (manic or depressive episodes) in a 12-month
period. Staring episodes are not a typical feature of mood episodes.
....
- -- - - - - - - -- - - - - - - -- - - - -- -- -- ---- - - -- - - - -- - - - - - - - -- --- - -- - - -- - - - - - -- - - - - -- -
r ~, ~ ~ r,
Previous Next Score Report
https://t.me/USMLENBME2CK Lab Values Calculator Help Pause
Exam Section : Item 41 of 50 National Board of Medical Examiners
■ Mark Clinical Neurology Self-Assessment
...
C) Rapid cycling bipolar disorder
D) Seizure activity;
E) Transient ischem ic attack
Correct Answer: D.

Bupropion is an antidepressant medication, sometimes utilized in bipolar disorder for incomplete response to mood stabilizer therapy,
that decreases the reuptake of norepinephrine and dopamine. By an unknown mechanism, bupropion decreases the seizure threshold.
Bupropion is a cytochrome P450 (CYP)-2D6 substrate, while terbinafine (an oral antifungal medication utilized for onychomycosis) is a
CYP-2D6 inhibitor. Consequently, this patient's bupropion serum concentrations were likely increased by the initiation of terbinafine.
Most medication-induced seizures are generalized tonic-clonic seizures, but some are absence seizures. Absence seizures are
generalized seizures that cause transient staring episodes with impaired consciousness during the seizure. Management involves
switching to a topical onychomycosis agent or a different antidepressant medication that does not decrease the seizure threshold.

Incorrect Answers: A, B, C, and E.

A catatonic episode (Choice A) is a neurobehavioral syndrome marked by the inability to move or speak, or the inability to stop moving
or speaking repetitively, that typically arises from an exacerbation of a psychiatric disorder. Catatonic episodes typically last days rather
than seconds to minutes.

Glioblastoma multiforme (Choice B) is an aggressive primary brain astrocytoma, which carries a poor prognosis. Seizures are a
common presenting symptom, as are headaches and focal neurologic deficits. However, seizures related to brain tumors are typically
focal rather than generalized, and this patient's recent history of medication changes makes a medication-induced seizure more likely.

Rapid cycling bipolar disorder (Choice C) is defined as at least four mood episodes (manic or depressive episodes) in a 12-month
period. Staring episodes are not a typical feature of mood episodes.

Transient ischemic attack (Choice E) is a transient episode of focal neurologic dysfunction caused by central nervous system ischemia
without infarction. The focal symptoms last less than 24 hours by definition and may last for as little as a few minutes. This patient
instead demonstrates generalized (bilateral) staring and blinking episodes, which are more likely related to a generalized seizure.

Educational Objective: Bupropion is an antidepressant medication that decreases the seizure threshold. Terbinafine is a CYP-2D6
inhibitor, which may increase the serum concentration of bupropion and increase the risk for developing generalized seizures.

.,.

r ~, ~ ~ r,
Previous Next Score Report
https://t.me/USMLENBME2CK Lab Values Calculator Help Pause
Exam Section: Item 42 of 50 National Board of Medical Examiners
■ Mark Clinical Neurology Self-Assessment

42. A 27-year-old man who is HIV positive comes to the physician because of left arm clumsiness for 1 week. His CD4+ T-lymphocyte count was
78/mm 3 (Normal~500) 8 weeks ago. Medications include zidovudine (AZT), didanosine, ritonavir, and azithromyci n. He appears well. His
temperature is 37.2°C (99°F), pulse is 96/min, and respirations are 16/min. Exam ination shows weakness of the left upper extremity. A CT scan of
the head with contrast shows a ring-enhancing lesion in the right parietal cortex. Wh ich of the following is the most appropriate next step in
management?

A) Intravenous methylprednisolone therapy


B) Oral dapsone therapy
C) Oral sulfadiazine-pyrimethamine therapy;
D) Oral trimethoprim-sulfamethoxazole therapy
E) Lumbar puncture
F) Brai n biopsy
Correct Answer: C.

Oral sulfadiazine-pyrimethamine therapy for the treatment of cerebral toxoplasmosis is the most appropriate next step in management.
Toxop/asma gondii is an intracellular protozoan parasite. Transmission can occur through the ingestion of oocysts from cat feces or
infected meat, direct transmission between a mother and fetus, or from organ transplantation, which leads to invasion fol lowed by
dissemination of the parasite where it typically lies dormant. It is a ubiquitous organism and nearly 10% of individuals wil l be
seropositive for T. gondii, but the parasite does not usually cause severe disease in immunocompetent individuals. In patients with HIV
infection and CD4+ T-lymphocyte counts less than 100/mm 3, however, reactivation can occur with several distinct clinical syndromes of
which cerebral toxoplasmosis is the most severe. Cerebral toxoplasmosis typically presents with multiple ring-enhancing lesions
scattered throughout the brain, although single cortical lesions are also seen. Seizures, focal neurologic deficits, and systemic
symptoms are common. The differential diagnosis includes primary central nervous system lymphoma and cerebral abscess. While
definitive diagnosis is with a brain biopsy that shows a dense infiltrate of lymphocytes, macrophages, and plasma cells, along with the
presence of the T. gondii organism, this is an invasive and morbid procedure. Empiric treatment with su lfadiazine-pyrimethamine is
favored with close clinical observation. Fai lure to improve should prompt a more extensive and potentially invasive diagnostic
approach. Cerebral toxoplasmosis is an AIDS-defining disease.

Incorrect Answers: A, B, D, E, and F.

Intravenous methylprednisolone therapy (Choice A) is often given for severe flares of multiple sclerosis (MS) requiring hospitalization.

. . ..
While patients with MS may have enhancing lesions on MRI, these are usually multiple, periventricu lar, and develop over years.

. . - - .. Choice B can be used to treat dermatitis her etiformis, . . . . . - ..


. . -
Previous Score Report
https://t.me/USMLENBME2CK Lab Values Calculator Help Pause
Exam Section: Item 42 of 50 National Board of Medical Examiners
■ Mark Clinical Neurology Self-Assessment

Correct Answer: C.
...

Oral sulfadiazine-pyrimethamine therapy for the treatment of cerebral toxoplasmosis is the most appropriate next step in management.
Toxop/asma gondii is an intracellular protozoan parasite. Transmission can occur through the ingestion of oocysts from cat feces or
infected meat, direct transmission between a mother and fetus, or from organ transplantation, which leads to invasion fol lowed by
dissemination of the parasite where it typically lies dormant. It is a ubiquitous organism and nearly 10% of individuals wil l be
seropositive for T. gondii, but the parasite does not usually cause severe disease in immunocompetent individuals. In patients with HIV
infection and CD4+ T-lymphocyte counts less than 100/mm 3, however, reactivation can occur with several distinct clinical syndromes of
which cerebral toxoplasmosis is the most severe. Cerebral toxoplasmosis typically presents with multiple ring-enhancing lesions
scattered throughout the brain, although single cortical lesions are also seen. Seizures, focal neurologic deficits, and systemic
symptoms are common. The differential diagnosis includes primary central nervous system lymphoma and cerebral abscess. While
definitive diagnosis is with a brain biopsy that shows a dense infiltrate of lymphocytes, macrophages, and plasma cells, along with the
presence of the T. gondii organism, this is an invasive and morbid procedure. Empiric treatment with su lfadiazine-pyrimethamine is
favored with close clinical observation. Fai lure to improve should prompt a more extensive and potentially invasive diagnostic
approach. Cerebral toxoplasmosis is an AIDS-defining disease.

Incorrect Answers: A, B, D, E, and F.

Intravenous methylprednisolone therapy (Choice A) is often given for severe flares of multiple sclerosis (MS) requiring hospitalization.
While patients with MS may have enhancing lesions on MRI, these are usually multiple, periventricu lar, and develop over years.

Oral dapsone therapy (Choice B) can be used to treat dermatitis herpetiformis, pyoderma gangrenosum, and relapsing polychondritis,
among others. Dapsone can also be used in conjunction with pyrimethamine as prophylaxis for toxoplasmosis but is not considered
adequate for therapeutic management.

Oral trimethoprim-sulfamethoxazole (TMP-SMX) therapy (Choice D) is typical ly given as prophylaxis against toxoplasmosis in patients
with HIV and CD4+ T-lymphocyte counts less than 100/mm 3. High-dose TMP-SMX can be used as an alternative treatment for
toxoplasmosis but is given intravenously.

Lumbar puncture (Choice E) shou ld be avoided in this patient as he has focal neurologic deficits and a mass on neuroimaging, which
raises suspicion for increased intracranial pressure. Lumbar puncture is the preferred diagnostic test for patients with suspected
meningitis or meningoencephalitis, not intraparenchymal cerebral disease.

Brain biopsy (Choice F) may be required if this patient does not respond to empiric therapy. The patient should first be given
sulfadiazine-pyrimethamine and observed for improvement. If there is no improvement, brain biopsy should be considered to ru le out a
cerebral abscess or primary central nervous system lymphoma.

Educational Objective: Cerebral toxoplasmosis is an AIDS-defining illness that occurs in patients with CD4+ T-lymphocyte counts ....

r ~, ~ ~ r,
Previous Next Score Report
https://t.me/USMLENBME2CK Lab Values Calculator Help Pause
Exam Section: Item 42 of 50 National Board of Medical Examiners
■ Mark Clinical Neurology Self-Assessment

Toxop/asma gondii is an intracellular protozoan parasite. Transmission can occur through the ingestion of oocysts from cat feces or ...
infected meat, direct transmission between a mother and fetus, or from organ transplantation, which leads to invasion followed by
dissemination of the parasite where it typically lies dormant. It is a ubiquitous organism and nearly 10% of individuals wil l be
seropositive for T. gondii, but the parasite does not usually cause severe disease in immunocompetent individuals. In patients with HIV
infection and CD4+ T-lymphocyte counts less than 100/mm 3, however, reactivation can occur with several distinct clinical syndromes of
which cerebral toxoplasmosis is the most severe. Cerebral toxoplasmosis typically presents with multiple ring-enhancing lesions
scattered throughout the brain, although single cortical lesions are also seen. Seizures, focal neurologic deficits, and systemic
symptoms are common. The differential diagnosis includes primary central nervous system lymphoma and cerebral abscess. While
definitive diagnosis is with a brain biopsy that shows a dense infiltrate of lymphocytes, macrophages, and plasma cells, along with the
presence of the T. gondii organism, this is an invasive and morbid procedure. Empiric treatment with su lfadiazine-pyrimethamine is
favored with close clinical observation. Fai lure to improve should prompt a more extensive and potentially invasive diagnostic
approach. Cerebral toxoplasmosis is an AIDS-defining disease.

Incorrect Answers: A, B, D, E, and F.

Intravenous methylprednisolone therapy (Choice A) is often given for severe flares of multiple sclerosis (MS) requiring hospitalization.
While patients with MS may have enhancing lesions on MRI, these are usually multiple, periventricu lar, and develop over years.

Oral dapsone therapy (Choice B) can be used to treat dermatitis herpetiformis, pyoderma gangrenosum, and relapsing polychondritis,
among others. Dapsone can also be used in conjunction with pyrimethamine as prophylaxis for toxoplasmosis but is not considered
adequate for therapeutic management.

Oral trimethoprim-sulfamethoxazole (TMP-SMX) therapy (Choice D) is typical ly given as prophylaxis against toxoplasmosis in patients
with HIV and CD4+ T-lymphocyte counts less than 100/mm 3. High-dose TMP-SMX can be used as an alternative treatment for
toxoplasmosis but is given intravenously.

Lumbar puncture (Choice E) shou ld be avoided in this patient as he has focal neurologic deficits and a mass on neuroimaging, which
raises suspicion for increased intracranial pressure. Lumbar puncture is the preferred diagnostic test for patients with suspected
meningitis or meningoencephalitis, not intraparenchymal cerebral disease.

Brain biopsy (Choice F) may be required if this patient does not respond to empiric therapy. The patient should first be given
sulfadiazine-pyrimethamine and observed for improvement. If there is no improvement, brain biopsy should be considered to ru le out a
cerebral abscess or primary central nervous system lymphoma.

Educational Objective: Cerebral toxoplasmosis is an AIDS-defining illness that occurs in patients with CD4+ T-lymphocyte counts
<100/mm 3 and presents with one or more ring-enhancing lesions on MRI, often with focal neurologic deficits. Empiric treatment with
sulfadiazine-pyrimethamine is warranted with close clinical observation. Failure to improve should prompt further investigation with a
brain biopsy to ru le out a cerebral abscess or primary central nervous system lymphoma. ....

r ~, ~ ~ r,
Previous Next Score Report
https://t.me/USMLENBME2CK Lab Values Calculator Help Pause
Exam Section: Item 43 of 50 National Board of Medical Examiners
■ Mark Clinical Neurology Self-Assessment

43. A previously healthy 32-year-old woman had the acute onset of severe headache followed by somnolence 2 hou rs ago. Her temperatu re is
38.3°C (101°F), pulse is 96/min, and blood pressure is 140/92 mm Hg. She is lethargic but oriented and answers questions appropriately. A
CT scan of the head shows no abnormalities except for small ventricles. Laboratory studies show:
Serum glucose 90 mg/dL
Cerebrospinal fluid
Color pi nk
Openi ng pressure 25 mm H20
Glucose 53 mg/dL
Protein 85 mg/dL
RBC 4000/mm 3
WBC 10/mm 3
Segmented neutrophils 65%
Lymphocytes 35%

W hich of the followi ng is the most likely diag nosis?

A) Cryptococcal meningitis
B) Enterovirus infection
C) Idiopathic intracran ial hypertension
D) Subarachnoid hemorrhage
E) West Nile encephalitis
Correct Answer: D.

Subarachnoid hemorrhage (SAH) refers to bleeding into the subarachnoid space and can be the result of a variety of causes, most
commonly trauma or a ruptured intracranial aneurysm in a nontraumatic setting. Patients typically present with an acute, severe
headache, often described as the worst headache of their life, along with neck stiffness, photophobia, and possible stupor or coma in
severe cases. Fever can be associated with SAH and is associated with worse outcomes. CT scan of the head typically shows
hyperdense blood products in the cerebral sulci and basal cisterns. However, sensitivity of CT for SAH can be reduced with small
volume bleeding or delayed presentation. If there is continued strong suspicion for SAH following a normal CT scan of the head, a
lumbar puncture can be performed, which can show red blood cells or xanthochromia (hemoglobin degradation products). As the blood
products circulate within the subarachnoid space, complications include the development of communicating and/or obstructive
hydrocephalus in the acute stage, along with the potential for re-bleeding and vasospasm in the days following the initial bleed. It is a
neurosurgical emergency that may be fatal if progressive or untreated.
....

r ~, ~ ~ r,
Previous Next Score Report
https://t.me/USMLENBME2CK Lab Values Calculator Help Pause
Exam Section: Item 43 of 50 National Board of Medical Examiners
■ Mark Clinical Neurology Self-Assessment Please Wait
...
Correct Answer: D.

Subarachnoid hemorrhage (SAH) refers to bleeding into the subarachnoid space and can be the result of a variety of causes, most
commonly trauma or a ruptured intracranial aneurysm in a nontraumatic setting. Patients typically present with an acute, severe
headache, often described as the worst headache of their life, along with neck stiffness, photophobia, and possible stupor or coma in
severe cases. Fever can be associated with SAH and is associated with worse outcomes. CT scan of the head typically shows
hyperdense blood products in the cerebral sulci and basal cisterns. However, sensitivity of CT for SAH can be reduced with small
volume bleeding or delayed presentation. If there is continued strong suspicion for SAH following a normal CT scan of the head, a
lumbar puncture can be performed, which can show red blood cells or xanthochromia (hemoglobin degradation products). As the blood
products circulate within the subarachnoid space, complications include the development of communicating and/or obstructive
hydrocephalus in the acute stage, along with the potential for re-bleeding and vasospasm in the days following the initial bleed. It is a
neurosurgical emergency that may be fatal if progressive or untreated.

Incorrect Answers: A, B, C, and E.

Cryptococcal meningitis (Choice A) is caused by meningeal infection with Cryptococcus neoformans, which is an encapsulated,
nondimorphic yeast and common opportunistic pathogen in immunocompromised patients. In cryptococcal meningitis, lumbar puncture
classically detects an increased opening pressure, increased cerebrospinal fluid (CSF) protein, and a mononuclear cell pleocytosis.

Enterovirus infection (Choice B) can cause a viral meningitis, which can present with fever, headache, and meningismus. Evaluation of
CSF would disclose an increased leukocyte count with a lymphocytic predominance and normal glucose concentration.

Idiopathic intracranial hypertension (Choice C) or pseudotumor cerebri refers to an idiopathic increase in intracranial pressure (ICP)
that typically occurs in obese women of childbearing age. It typically presents with headaches (that may be postural), transient visual
deficits caused by pressure on the optic nerve, along with nausea and vomiting (symptoms of increased ICP).

West Nile virus (Choice E) is a flavivirus transmitted via the bite of an infected mosquito and may cause West Nile fever with
progression to meningoencephalitis. West Nile fever presents with headache, myalgias, nausea, vomiting, and a nonspecific
maculopapular rash distributed symmetrically on the extremities and trunk. Meningoencephalitis is associated with acute flaccid
paralysis, myoclonus, ataxia, and seizures.

Educational Objective: Patients with SAH can present with an acute, severe headache, often described as the worst headache of their
life, along with fever, neck stiffness, photophobia, and possible stupor or coma in severe cases. If there is a continued strong suspicion
for SAH after a normal CT scan, a lumbar puncture can be performed, which may show red blood cells or xanthochromia (hemoglobin
degradation products).

....

r ~, ~ ~ r,
Previous Next Score Report
https://t.me/USMLENBME2CK Lab Values Calculator Help Pause
Exam Section: Item 44 of 50 National Board of Medical Examiners
■ Mark Clinical Neurology Self-Assessment

44. A 62-year-old woman with metastatic breast cancer is brought to the emergency department because of urinary incontinence for 24 hours. She
has metastases to the sku ll, multiple vertebral bodies, ribs, liver, and para-aortic lymph nodes. Medications include tamoxifen, sustained-release
morphine, stool softeners, and fluoxetine. Her temperature is 36.7°C (98.1 °F), pulse is 84/min, respirations are 14/min, and blood pressure is
128/76 mm Hg. Examination shows a large suprapubic mass. There is significant pain over the entire lumbar spine on fist percussion. Rectal
exam ination shows decreased sph incter tone. Which of the following is the most likely cause of the incontinence?

A) Anticholinerg ic effects of fluoxetine


B) Hypercalcemia
C) Metastases to the bladder
D) Metastases to the cauda equina
E) Urethral spasm
Correct Answer: D.

Cauda equina syndrome occurs because of compression of the spinal nerve roots comprising the cauda equina. Often, such
compression occurs as a result of metastases to the cauda equina, adjacent malignancy, trauma (eg, compression fractures with
fragment retropulsion), disc herniation, spinal stenosis, spinal epidural abscess, or spinal epidural/subdural hematoma. Symptoms
include lower extremity numbness, weakness, paralysis, hyporeflexia, perinea! anesthesia, fecal incontinence with poor rectal tone, and
overflow incontinence marked by the inability to appropriately void and associated urinary retention. Emergent MRI of the lumbosacral
spine is indicated to investigate the cause of cauda equina syndrome, and to determine the necessity of surgical decompression. Initial
treatment for patients with epidural spinal cord compression and neurologic deficits include symptomatic treatment, pain control, and
glucocorticoid therapy prior to definitive medical or surgical therapy. High-dose glucocorticoid therapy is helpful as a bridge to definitive
therapy, as it may transiently improve neurologic function in patients by decreasing edema at the site of cord compression.

Incorrect Answers: A, B, C, and E.

Anticholinergic effects of fluoxetine (Choice A) could also cause urinary retention but would not explain pain at the lumbar spine or
decreased sphincter tone. Anticholinergic adverse effects can antagonize muscarinic receptors in the bladder wall, resulting in impaired
urinary bladder contractility leading to urinary retention. Other anticholinergic effects can include dry mouth, sedation, constipation,
hallucinations, delirium, ataxia, flushed skin, and visual disturbances (cycloplegia). Such effects are uncommon with fluoxetine, and
malignant compression is much more likely in this case.

Hypercalcemia (Choice B) can occur with bony metastases, but patients with severe hypercalcemia would present with nephrolithiasis,
abdominal discomfort, confusion, constipation, and psychiatric disturbances. It would be unlikely to cause urinary retention and overflow
incontinence. ....

r ~, ~ ~ r,
Previous Next Score Report
https://t.me/USMLENBME2CK Lab Values Calculator Help Pause
Exam Section: Item 44 of 50 National Board of Medical Examiners
■ Mark Clinical Neurology Self-Assessment

E) Urethral spasm
...
Correct Answer: D.

Cauda equina syndrome occurs because of compression of the spinal nerve roots comprising the cauda equina. Often, such
compression occurs as a result of metastases to the cauda equina, adjacent malignancy, trauma (eg, compression fractures with
fragment retropulsion), disc herniation, spinal stenosis, spinal epidural abscess, or spinal epidural/subdural hematoma. Symptoms
include lower extremity numbness, weakness, paralysis, hyporeflexia, perinea! anesthesia, fecal incontinence with poor rectal tone, and
overflow incontinence marked by the inability to appropriately void and associated urinary retention. Emergent MRI of the lumbosacral
spine is indicated to investigate the cause of cauda equina syndrome, and to determine the necessity of surgical decompression. Initial
treatment for patients with epidural spinal cord compression and neurologic deficits include symptomatic treatment, pain control, and
glucocorticoid therapy prior to definitive medical or surgical therapy. High-dose glucocorticoid therapy is helpful as a bridge to definitive
therapy, as it may transiently improve neurologic function in patients by decreasing edema at the site of cord compression.

Incorrect Answers: A, B, C, and E.

Anticholinergic effects of fluoxetine (Choice A) could also cause urinary retention but would not explain pain at the lumbar spine or
decreased sphincter tone. Anticholinergic adverse effects can antagonize muscarinic receptors in the bladder wall, resu lting in impaired
urinary bladder contractility leading to urinary retention. Other anticholinergic effects can include dry mouth, sedation, constipation,
hallucinations, delirium, ataxia, flushed skin, and visual disturbances (cycloplegia). Such effects are uncommon with fluoxetine, and
malignant compression is much more likely in this case.

Hypercalcemia (Choice B) can occur with bony metastases, but patients with severe hypercalcemia would present with nephrolithiasis,
abdominal discomfort, confusion, constipation, and psychiatric disturbances. It would be unlikely to cause urinary retention and overflow
incontinence.

Metastases to the bladder (Choice C) from breast cancer are rare and may present with hematuria and other urinary symptoms. It
would not explain the patient's decreased sphincter tone or lumbar spinal tenderness.

Urethral spasm (Choice E) has been associated with urethral pain syndrome, which is characterized by urethral pain, dysuria, nocturia,
urinary urgency, and frequency in the absence of a urinary tract infection.

Educational Objective: Cauda equina syndrome occurs as a resu lt of compression of the spinal nerve roots comprising the cauda
equina, which may present following trauma, metastases directly to the cauda equina nerve roots and/or adjacent malignancy involving
the spine. Symptoms include lower extremity numbness, weakness, paralysis, hyporeflexia, perinea! anesthesia, fecal incontinence
with poor rectal tone, and overflow incontinence marked by the inability to void and associated urinary retention.

.,.

r ~, ~ ~ r,
Previous Next Score Report
https://t.me/USMLENBME2CK Lab Values Calculator Help Pause
Exam Section: Item 45 of 50 National Board of Medical Examiners
■ Mark Clinical Neurology Self-Assessment

45. A 52-year-old woman is brought to the emergency department by her husband 30 minutes after a generalized tonic-clonic seizure. Her husband
reports that she has had a headache and has seemed confused over the past 3 days. She was scheduled to be evaluated by a pulmonolog ist later
this week for the recent onset of mild hemoptysis. She smoked two packs of cigarettes daily for 30 years but quit 5 years ago. She has a 5-year
history of type 2 diabetes mellitus controlled by diet. On arrival, she is postictal. Her tem perature is 37°C (98.6°F), and blood pressure is 110/60
mm Hg. She is minimal ly responsive to painful stimu li. Cardiopulmonary examination shows no abnormalities. There is no peripheral edema.
There are no focal neurologic deficits. Laboratory studies show:
Serum
Na + 112 mEq/L
Glucose 150 mg/dl
Creatinine 0.7 mg/dl
Cholesterol 230 mg/dl
Osmolality 238 mOsmol/kg
Urine
Glucose trace
Protein none
RBC 1-3/hpf
WBC 1-3/hpf
Osmolality 240 mOsmol/kg
Na + 25 mEq/L

Which of the following is the most likely diagnosis?

A) Bacterial pneumon ia H) lntracran ial neoplasm


B) Cirrhosis of the liver I) Mening itis
C) Decreased solute intake J) Nephrotic syndrome
D) Dehydration K) Pseudohyponatrem ia
E) Glucocorticoid deficiency L) Psychogenic polyd ipsia
F) Hyperglycemia M) Renal failure
G) Inappropriate secretion of ADH (vasopressin )
Correct Answer: G.

Smal l cell carcinoma of the lung is a neoplasm associated with numerous paraneoplastic syndromes, including Cushing syndrome caused by
- .- ....

r ~, ~ ~ r,
Previous Next Score Report
https://t.me/USMLENBME2CK Lab Values Calculator Help Pause
Exam Section: Item 45 of 50 National Board of Medical Examiners
■ Mark Clinical Neurology Self-Assessment

Correct Answer: G. ...

Small cell carcinoma of the lung is a neoplasm associated with numerous paraneoplastic syndromes, including Cushing syndrome caused by
adrenocorticotropic hormone production, syndrome of inappropriate antidiuretic hormone (SIADH), myasthenic (Lambert-Eaton) syndrome,
paraneoplastic myelitis, encephalitis, and subacute cerebellar degeneration. This patient's altered mental status, seizures, and confusion are likely to
the result of severe hyponatrem ia secondary to paraneoplastic SIADH in undiagnosed lung cancer. ADH resu lts in increased aquaporin expression on
the luminal surface of collecting duct cells, which increases the membrane permeability to water, leading to inappropriate free water reabsorption,
hyponatremia, decreased serum osmolality, and inappropriately increased urine concentration. The management of SIADH-associated hyponatremia
involves treatment of the underlying disease, restriction of water intake, and gradual sodium correction. If there are associated severe neurologic
symptoms, administration of hypertonic saline may be considered to quickly correct the sodium concentration, although this must be performed with
caution.

Incorrect Answers: Choices A, B, C, D, E, F, H, I, J, K, L, and M.

Bacterial pneumonia (Choice A) can present with hemoptysis, but typically presents with sputum production, fever, and cough.

Cirrhosis of the liver (Choice B) is associated with hypervolemic hyponatremia. However, patients with cirrhosis also present with jaundice and signs of
portal hypertension in contrast to the euvolemia in SIADH.

Decreased solute intake (Choice C) would not itself cause hyponatremia as a resu lt of the kidney's ability to reabsorb sodium and water to maintain
homeostatic osmolality. SIADH impairs this ability by causing unregulated free water reabsorption.

Dehydration (Choice D) first leads to hypernatremia, not hyponatremia. Hypernatremia can develop secondary to inadequate free water intake or
administration.

Glucocorticoid deficiency (Choice E), such as in adrenal insufficiency, causes low serum cortisol concentrations and presents with fatigue,
hypotension, and hypoglycemia.

Factitious hyponatremia caused by hyperglycemia (Choice F) can occur in patients with severe hyperglycemia. This patient's glucose is 150 mg/dl,
which is not enough to cause hyponatremia on laboratory evaluation. Severe hyperglycemia can lead to the osmotic shift of water from the intracellular
to extracellular space, leading to dilutional hyponatremia.

An intracranial neoplasm (Choice H) and meningitis (Choice I) can cause SIADH, but would present with focal neurologic deficits, and in the case of
meningitis, would present with fever and nuchal rigidity.

Nephrotic syndrome (Choice J) is characterized by edema secondary to excessive proteinuria resulting in hypoalbuminemia and diminished
intravascular oncotic pressure.

r - ~, ~ ~ r,
....

Previous Next Score Report


https://t.me/USMLENBME2CK Lab Values Calculator Help Pause
Exam Section: Item 45 of 50 National Board of Medical Examiners
■ Mark Clinical Neurology Self-Assessment
...
Incorrect Answers: Choices A, B, C, D, E, F, H, I, J, K, L, and M.

Bacterial pneumonia (Choice A) can present with hemoptysis, but typically presents with sputum production, fever, and cough.

Cirrhosis of the liver (Choice B) is associated with hypervolemic hyponatremia. However, patients with cirrhosis also present with jaundice and signs of
portal hypertension in contrast to the euvolemia in SIADH.

Decreased solute intake (Choice C) would not itself cause hyponatremia as a result of the kidney's ability to reabsorb sodium and water to maintain
homeostatic osmolality. SIADH impairs this ability by causing unregulated free water reabsorption.

Dehydration (Choice D) first leads to hypernatremia, not hyponatremia. Hypernatremia can develop secondary to inadequate free water intake or
administration.

Glucocorticoid deficiency (Choice E), such as in adrenal insufficiency, causes low serum cortisol concentrations and presents with fatigue,
hypotension, and hypoglycemia.

Factitious hyponatremia caused by hyperglycemia (Choice F) can occur in patients with severe hyperglycemia. This patient's glucose is 150 mg/dl,
which is not enough to cause hyponatremia on laboratory evaluation. Severe hyperglycemia can lead to the osmotic shift of water from the intracellular
to extracellular space, leading to dilutional hyponatremia.

An intracranial neoplasm (Choice H) and meningitis (Choice I) can cause SIADH, but would present with focal neurologic deficits, and in the case of
meningitis, would present with fever and nuchal rigidity.

Nephrotic syndrome (Choice J) is characterized by edema secondary to excessive proteinuria resulting in hypoalbuminemia and diminished
intravascular oncotic pressure.

Pseudohyponatremia (Choice K) refers to a laboratory error of decreased sodium concentration in the setting of normal serum osmolality, which can
be caused by lipemic serum, obstructive jaundice, or plasma cell dyscrasia. This patient has decreased serum osmolality, consistent with SIADH.

Excessive intake of free water in psychogenic polydipsia (Choice L) can result in dilute urine and hyponatremia. This patient has inappropriately
concentrated urine and low serum osmolality, consistent with SIADH.

Renal failure (Choice M) can be associated with hyponatremia in volume overload. The patient's hyponatremia is not caused by abnormalities of
sodium reabsorption or excretion, but rather, increased free water reabsorption as a result of SIADH.

Educational Objective: Small cell carcinoma of the lung is associated with paraneoplastic SIADH. ADH results in increased collecting duct permeability
to water, leading to inappropriate free water reabsorption, hyponatremia, decreased serum osmolality, and inappropriately concentrated urine.
....

r ~, ~ ~ r,
Previous Next Score Report
https://t.me/USMLENBME2CK Lab Values Calculator Help Pause
Exam Section: Item 46 of 50 National Board of Medical Examiners
■ Mark Clinical Neurology Self-Assessment

46. A 3-week-old male newborn is brought to the emergency department by his parents because of a 1-day history of increased fussiness and
drowsiness. En route to the emergency department, he had rhythmic jerking movements of all extrem ities. His mother received minimal prenatal
care and was treated for gonorrheal cervicitis during pregnancy; her Streptococcus agalactiae (group B) status was unknown. He was born at
38 weeks' gestation. Examination at birth showed no abnormalities. Today, he is irritable and responds only to painful stimulation. His temperature
is 39°C (102.2°F), pulse is 160/min, and respirations are 32/min. Examination shows no conjunctivitis or cutaneous lesions. The lungs are clear to
auscultation. A lumbar puncture is performed, and cerebrospinal fluid analysis shows:
Glucose 60 mg/dl
Total protein 100 mg/dl
WBC 300/mm 3
RBC 5000/mm 3
Gram stain
WBC 2+
Organisms none

In addition to ceftriaxone and vancomycin, which of the following is the most appropriate pharmacotherapy?

A ) Acyclovifi
B) Dexamethasone
C) Ganciclovir
D) Ribavirin
E) Zidovudine
Correct Answer: A.

Neonatal sepsis is a potential ly life-threatening syndrome characterized by severe systemic manifestations of infection in infants less
than 28 days of age. The culprit pathogen is most commonly acquired by vertical transmission during transit of the birth canal or
horizontal transmission from contact with the healthcare environment. It is considered early-onset neonatal sepsis if the condition
develops within the first week of life and late-onset if it occurs after. Early-onset neonatal sepsis is typical ly caused by
Streptococcus agalactiae (group B) or Escherichia coli. Common organisms implicated in late-onset neonatal sepsis include S.
agalactiae (group B), E. coli, Staphylococcus species (especially S. aureus), Pseudomonas aeruginosa, and Candida albicans. Initial
empiric antimicrobial therapy shou ld cover these bacterial organisms. Lumbar puncture is indicated in the evaluation of neonatal sepsis
as infants do not display typical symptoms of meningitis as associated with adult patients. Empiric coverage for herpes simplex virus
(HSV) meningoencephalitis with acyclovir should be added to the regimen if the cerebrospinal fluid displays an increased erythrocyte
count or if the patient presents with characteristic herpetic lesions.
....

r ~, ~ ~ r,
Previous Next Score Report
https://t.me/USMLENBME2CK Lab Values Calculator Help Pause
Exam Section: Item 46 of 50 National Board of Medical Examiners
■ Mark Clinical Neurology Self-Assessment
...
Correct Answer: A.

Neonatal sepsis is a potential ly life-threatening syndrome characterized by severe systemic manifestations of infection in infants less
than 28 days of age. The culprit pathogen is most commonly acquired by vertical transmission during transit of the birth canal or
horizontal transmission from contact with the healthcare environment. It is considered early-onset neonatal sepsis if the condition
develops within the first week of life and late-onset if it occurs after. Early-onset neonatal sepsis is typical ly caused by
Streptococcus agalactiae (group B) or Escherichia coli. Common organisms implicated in late-onset neonatal sepsis include S.
agalactiae (group B), E. coli, Staphylococcus species (especially S. aureus), Pseudomonas aeruginosa, and Candida albicans. Initial
empiric antimicrobial therapy shou ld cover these bacterial organisms. Lumbar puncture is indicated in the evaluation of neonatal sepsis
as infants do not display typical symptoms of meningitis as associated with adult patients. Empiric coverage for herpes simplex virus
(HSV) meningoencephalitis with acyclovir should be added to the regimen if the cerebrospinal fluid displays an increased erythrocyte
count or if the patient presents with characteristic herpetic lesions.

Incorrect Answers: B, C, D, and E.

Dexamethasone (Choice B) may be used as an adjuvant therapy in the treatment of bacterial meningitis to decrease the risk for
neurologic complications and mortality, especially in adults with meningitis caused by S. pneumoniae. The benefit is less clear in
neonates and is not routinely recommended.

Ganciclovir (Choice C) is an antiviral agent with activity against cytomegalovirus (CMV). Congenital CMV infection can present with
developmental delay, cognitive defects, hearing loss, seizures, and a petechial rash. CMV does not typically cause neonatal sepsis.

Ribavirin (Choice D) is an antiviral agent used in the treatment of hepatitis C virus (HCV) and respiratory syncytial virus (RSV). It
inhibits guanine nucleotide synthesis. Complications include hemolytic anemia. Ribavirin is a severe teratogen and should be avoided
during pregnancy.

Zidovudine (Choice E) is a nucleoside reverse transcriptase inhibitor (NRTI) used in the treatment of HIV infection. It is commonly
utilized for known exposure of an infant to HIV in utero to reduce the risk for vertical transmission. Infants are started on a course
immediately after birth. If exposure risk is unknown, the mother and infant may be tested for the presence of HIV antibodies or
identifiable HIV nucleic acid sequences prior to initiating therapy.

Educational Objective: Neonatal sepsis is a severe condition warranting broad antimicrobial coverage. Thorough evaluation for a
source is indicated, including lumbar puncture. Empiric coverage with acyclovir for HSV meningoencephalitis should be initiated if the
CSF displays an increased erythrocyte count or if the patient presents with characteristic herpetic lesions.

.,.

r ~, ~ ~ r,
Previous Next Score Report
https://t.me/USMLENBME2CK Lab Values Calculator Help Pause
Exam Section : Item 47 of 50 National Board of Medical Examiners
■ Mark Clinical Neurology Self-Assessment

47. An otherwise healthy 62-year-old man is brought to the emergency department 45 minutes after an episode of loss of consciousness wh ile
shaving. Prior to the episode, he had light-headedness, nausea, and dimming of vision. His wife states that he was unconscious for about 1 minute
and lost control of bladder function. She did not notice any movement during the episode. He had a sim ilar episode 1 month ago while shaving. At
that time, echocard iography, 24-hour ambulatory ECG monitoring, and a tilt test showed no abnormalities; cardiac enzyme activities were within
the reference range. His current blood pressure is 150/96 mm Hg. A grade 2/6, systolic ejection murmur is heard best at the upper left sternal
border. The remainder of the exam ination shows no abnormalities. Wh ich of the following is the most likely diagnosis?

A) Absence seizure
B) Aortic stenos is
C) Carotid sinus hypersensitivity;
D) Generalized tonic-clonic seizure
E) Neurocardiogenic (vasovagal) syncope
F) Transient ischem ic attack
Correct Answer: C.

Carotid sinus hypersensitivity syndrome is characterized by an exaggerated response to carotid baroreceptor stimulation, with resultant
sinus bradycardia, hypotension, and syncope. Baroreceptors located at the carotid bifurcations and the aortic arch are important in
regulating cardiac output and maintaining adequate perfusion pressures to the brain and organs of the upper body. Baroreceptors are
sensitive to mechanical pressure; normally, an increase in blood pressure stimulates baroreceptor activity, which activates a reflex arc.
This results in parasympathetic innervation of the sinoatrial and atrioventricular nodes and inhibition of sympathetic vascular tone. In
turn, this causes decreased pulse and peripheral vascular resistance. Mechanical compression of the baroreceptors from external
palpation, such as during shaving of the neck, can potentially stimulate this reflex.

Incorrect Answers: A, B, D, E, and F.

Absence seizure (Choice A) is characterized by a transient blank stare, and typically occurs in brief episodes multiple times throughout
the day. It is more common in child ren. EEG monitoring characteristically shows 3-Hz spike-and-wave complexes during the episodes.
Often, patients disclose no postictal confusion. The patient's symptoms are not consistent with absence seizure.

Aortic stenosis (Choice B) presents with a crescendo-decrescendo systolic murmur best heard at the upper right sternal border with
radiation to the carotid arteries. Syncope associated with aortic stenosis typically occurs during exertion because of the inability to
increase stroke volume to match perfusion demands, resulting in consequent cerebral hypoperfusion. Syncope during shaving or other
mild activities is less consistent with aortic stenosis.
....

r ~, ~ ~ r,
Previous Next Score Report
https://t.me/USMLENBME2CK Lab Values Calculator Help Pause
Exam Section : Item 47 of 50 National Board of Medical Examiners
■ Mark Clinical Neurology Self-Assessment
...
Correct Answer: C.

Carotid sinus hypersensitivity syndrome is characterized by an exaggerated response to carotid baroreceptor stimulation, with resultant
sinus bradycardia, hypotension, and syncope. Baroreceptors located at the carotid bifurcations and the aortic arch are important in
regulating cardiac output and maintaining adequate perfusion pressures to the brain and organs of the upper body. Baroreceptors are
sensitive to mechanical pressure; normally, an increase in blood pressure stimulates baroreceptor activity, which activates a reflex arc.
This results in parasympathetic innervation of the sinoatrial and atrioventricular nodes and inhibition of sympathetic vascular tone. In
turn, this causes decreased pulse and peripheral vascular resistance. Mechanical compression of the baroreceptors from external
palpation, such as during shaving of the neck, can potentially stimulate this reflex.

Incorrect Answers: A, B, D, E, and F.

Absence seizure (Choice A) is characterized by a transient blank stare, and typically occurs in brief episodes multiple times throughout
the day. It is more common in children. EEG monitoring characteristically shows 3-Hz spike-and-wave complexes during the episodes.
Often, patients disclose no postictal confusion. The patient's symptoms are not consistent with absence seizure.

Aortic stenosis (Choice B) presents with a crescendo-decrescendo systolic murmur best heard at the upper right sternal border with
radiation to the carotid arteries. Syncope associated with aortic stenosis typically occurs during exertion because of the inability to
increase stroke volume to match perfusion demands, resulting in consequent cerebral hypoperfusion. Syncope during shaving or other
mild activities is less consistent with aortic stenosis.

Generalized tonic-clonic seizure (Choice D) is the most common type of generalized seizure and is characterized by the sudden onset
of muscle contractions followed by rhythmic jerking of the extremities. Bowel and/or bladder incontinence commonly occurs. Episodes
are accompanied by a postictal state marked by confusion and fatigue.

Neurocardiogenic (vasovagal) syncope (Choice E) is a common cause of syncope, often associated with nausea, lightheadedness,
visual disturbances, pallor, and/or diaphoresis. Symptoms usually occur after standing for a prolonged period. A normal tilt test in this
patient makes this diagnosis less likely than carotid sinus hypersensitivity syndrome.

Transient ischemic attack (Choice F) is defined as a transient episode of neurologic dysfunction caused by focal brain, spinal cord, or
retinal ischemia without infarction. A transient ischemic attack resulting in syncope does not typically present with a postictal state.

Educational Objective: Syncope is a common presenting complaint with numerous causes. A careful history that evaluates for
provoking factors and physical examination can guide further diagnostic studies. Syncope that occurs exclusively with neck palpation is
suggestive of carotid sinus hypersensitivity syndrome.

....

r ~, ~ ~ r,
Previous Next Score Report
https://t.me/USMLENBME2CK Lab Values Calculator Help Pause
Exam Section: Item 48 of 50 National Board of Medical Examiners
■ Mark Clinical Neurology Self-Assessment

48. A 32-year-old man is brought to the physician because of a 10-month history of difficulty walking and decreased grip strength and stiffness of his
hands that are interfering with his job as a plumber. He has no history of serious illness. There is frontal balding. Cranial nerve examination shows
bilateral ptosis and atrophy of the sternocleidomastoid and temporalis muscles. There is distal intrinsic musculature weakness of the hand and
bilateral weakness on ankle dorsiflexion. Percussion of the hypothenar muscles resu lts in slow relaxation. Sensation is intact. Which of the
following is the most likely diagnosis?

A) Amyotrophic lateral sclerosis


B) Facioscapulohumeral muscular dystrophy
C) Myasthenia gravis
D) Myotonic dystrophy;
E) Spinal muscular atrophy
Correct Answer: D.

Myotonic dystrophy is a genetic disorder caused by expansion of a trinucleotide repeat, which occurs when a series of three
nucleotides in repetition are replicated incorrectly and too many repeats are included in the replicated product. Other conditions caused
by trinucleotide repeat expansions include fragile X syndrome, Huntington disease, and Friedrich ataxia. Myotonic dystrophy has
several phenotypes, but all are characterized by muscle weakness and myotonia, defined as delayed relaxation following a normal
contraction of the muscle. Classic myotonic dystrophy, as seen in this case, presents during the second, third , or fourth decades of life
with muscle weakness, myotonia, cataracts, frontal balding, cardiac arrhythmias, and hypogonadism. The most frequently affected
muscles include the facial muscles, sternocleidomastoid, distal forearm muscles, intrinsic hand muscles, and ankle dorsiflexors. Life
expectancy is slightly reduced for patients with classic myotonic dystrophy and premature death is most commonly caused by
respiratory or cardiac dysfunction.

Incorrect Answers: A, B, C, and E.

Amyotrophic lateral sclerosis (Choice A) is a progressive and incurable neurodegenerative disorder. Its clinical hallmark is the presence
of both upper motoneuron signs, such as hyperreflexia and spasticity, along with lower motoneuron signs, such as atrophy and
fasciculations, accompanied by muscle weakness.

Facioscapulohumeral muscular dystrophy (Choice B) is a type of muscular dystrophy in which progressive muscle weakness of the
facial muscles, scapulae, upper arms, lower legs, and hip girdles occur. The muscles are common ly involved in an asymmetric pattern
and it typically presents by the age of 20 years.

Myasthenia gravis (Choice C) is an autoimmune disorder that commonly presents with ptosis, diplopia, dysphagia, and muscle ....
-- - --- - -- - - - -- - --- - -- --- - - - -- -- -- - - - - - -- - -- - - - - - -- - - - - - -- - - --- ---
r ~, ~ ~ r,
Previous Next Score Report
https://t.me/USMLENBME2CK Lab Values Calculator Help Pause
Exam Section: Item 48 of 50 National Board of Medical Examiners
■ Mark Clinical Neurology Self-Assessment

E) Spi nal muscular atrophy


...
Correct Answer: D.

Myotonic dystrophy is a genetic disorder caused by expansion of a trinucleotide repeat, which occurs when a series of three
nucleotides in repetition are replicated incorrectly and too many repeats are included in the replicated product. Other conditions caused
by trinucleotide repeat expansions include fragile X syndrome, Huntington disease, and Friedrich ataxia. Myotonic dystrophy has
several phenotypes, but all are characterized by muscle weakness and myotonia, defined as delayed relaxation following a normal
contraction of the muscle. Classic myotonic dystrophy, as seen in this case, presents during the second, third , or fourth decades of life
with muscle weakness, myotonia, cataracts, frontal balding, cardiac arrhythmias, and hypogonadism. The most frequently affected
muscles include the facial muscles, sternocleidomastoid, distal forearm muscles, intrinsic hand muscles, and ankle dorsiflexors. Life
expectancy is slightly reduced for patients with classic myotonic dystrophy and premature death is most commonly caused by
respiratory or cardiac dysfunction.

Incorrect Answers: A, B, C, and E.

Amyotrophic lateral sclerosis (Choice A) is a progressive and incurable neurodegenerative disorder. Its clinical hallmark is the presence
of both upper motoneuron signs, such as hyperreflexia and spasticity, along with lower motoneuron signs, such as atrophy and
fasciculations, accompanied by muscle weakness.

Facioscapulohumeral muscular dystrophy (Choice B) is a type of muscular dystrophy in which progressive muscle weakness of the
facial muscles, scapulae, upper arms, lower legs, and hip girdles occur. The muscles are common ly involved in an asymmetric pattern
and it typically presents by the age of 20 years.

Myasthenia gravis (Choice C) is an autoimmune disorder that commonly presents with ptosis, diplopia, dysphagia, and muscle
weakness, which are worse with repetitive activity and later in the day. The slow relaxation of muscles, or myotonia, is not present, and
neither is the frontal balding characteristic of classic myotonic dystrophy.

Spinal muscular atrophy (Choice E) is a group of autosomal recessive disorders that lead to the degeneration of motoneurons and
result in progressive hypotonia and areflexia. Patients may exhibit difficulty with mobility and frequently experience respiratory distress
and difficulty feeding.

Educational Objective: Myotonic dystrophy is a genetic disorder caused by expansion of a trinucleotide repeat characterized by muscle
weakness, myotonia, cataracts, frontal balding, cardiac arrhythmias, and hypogonadism. The most frequently affected muscles include
the facial muscles, sternocleidomastoids, distal forearm muscles, intrinsic hand muscles, and ankle dorsiflexors.

.,.

r ~, ~ ~ r,
Previous Next Score Report
https://t.me/USMLENBME2CK Lab Values Calculator Help Pause
Exam Section: Item 49 of 50 National Board of Medical Examiners
■ Mark Clinical Neurology Self-Assessment
...

49. A 57-year-old woman comes to the physician because of persistent numbness of the back of her right hand for 2 weeks. Her symptoms began
after operative treatment of a distal rad ial fracture. Sensation to pinprick is decreased over the dorsal first web, thumb, and index finger. There is
no motor deficit. Which of the following is the most likely site of nerve injury?

A) Axillary nerve I) Radial nerve above the elbow


B) Long thoracic nerve J) Radial nerve at the elbow
C) Median nerve above the elbow K) Radial nerve at the distal forearm
D) Median nerve at the elbow L) Suprascapular nerve
E) Median nerve at the wrist M) Thoracodorsal nerve
F) Musculocutaneous nerve above the elbow N) Ulnar nerve above the elbow
G) Musculocutaneous nerve at the elbow 0) Ulnar nerve at the elbow
H) Musculocutaneous nerve at the wrist P) Ulnar nerve at the wrist
Correct Answer: K.

Distal radius fractures are a common orthopedic injury, often caused by a fall onto an outstretched hand. Treatment of a distal radius fracture can be
nonoperative with a splint or cast immobilization or can involve an operation with percutaneous pinning or fixation with plating. Distal radius fractures
may be complicated by median nerve injury (eg, acute carpal tunnel syndrome), while operative management can be complicated by injury to the
superficial radial nerve in the wrist. The median nerve supplies sensation to the volar and dorsal distal aspects of the lateral three and a half fingers
and lateral palm of the hand (thenar eminence), whereas the radial nerve supplies sensation to the dorsal bases of the lateral three and a half fingers
and the lateral dorsum of the hand. This patient has impaired sensation along the dorsum of the hand, consistent with a radial nerve injury at the wrist
or distal forearm.

Incorrect Answers: A, B, C, D, E, F, G, H, I, J, L, M, N, 0, and P.

Axil lary nerve (Choice A) palsy can occur after a humeral neck fracture or shoulder dislocation. The axillary nerve innervates the deltoid muscle,
leading to impaired arm abduction and loss of sensation over the deltoid muscle.

The long thoracic nerve (Choice B) innervates the serratus anterior muscle. Injury to this nerve can lead to a winged scapu la.

The median nerve (Choices C, D, and E) can be injured in a supracondylar humerus fracture, a fracture dislocation of the wrist, or a laceration along
its course. A median nerve injury can cause symptoms depending on the location of the lesion, causing palsy distal to the injury. Median nerve palsy
can result in impaired wrist flexion, thumb opposition, or flexion of the lateral fingers; it can also cause impaired sensation to the thenar eminence and
. .

Previous Score Report


https://t.me/USMLENBME2CK Lab Values Calculator Help Pause
Exam Section: Item 49 of 50 National Board of Medical Examiners
■ Mark Clinical Neurology Self-Assessment

Correct Answer: K. ...

Distal radius fractures are a common orthopedic injury, often caused by a fall onto an outstretched hand. Treatment of a distal radius fracture can be
nonoperative with a splint or cast immobilization or can involve an operation with percutaneous pinning or fixation with plating. Distal radius fractures
may be complicated by median nerve injury (eg, acute carpal tunnel syndrome), while operative management can be complicated by injury to the
superficial radial nerve in the wrist. The median nerve supplies sensation to the volar and dorsal distal aspects of the lateral three and a half fingers
and lateral palm of the hand (thenar eminence), whereas the radial nerve supplies sensation to the dorsal bases of the lateral three and a half fingers
and the lateral dorsum of the hand. This patient has impaired sensation along the dorsum of the hand, consistent with a radial nerve injury at the wrist
or distal forearm.

Incorrect Answers: A, B, C, D, E, F, G, H, I, J, L, M, N, 0, and P.

Axil lary nerve (Choice A) palsy can occur after a humeral neck fracture or shoulder dislocation. The axillary nerve innervates the deltoid muscle,
leading to impaired arm abduction and loss of sensation over the deltoid muscle.

The long thoracic nerve (Choice B) innervates the serratus anterior muscle. Injury to this nerve can lead to a winged scapu la.

The median nerve (Choices C, D, and E) can be injured in a supracondylar humerus fracture, a fracture dislocation of the wrist, or a laceration along
its course. A median nerve injury can cause symptoms depending on the location of the lesion, causing palsy distal to the injury. Median nerve palsy
can result in impaired wrist flexion, thumb opposition, or flexion of the lateral fingers; it can also cause impaired sensation to the thenar eminence and
volar and dorsal distal aspects of the lateral three and a half fingers.

The musculocutaneous nerve (Choices F, G, and H) courses proximal in the arm, and injury can cause impaired forearm flexion and supination. It
supplies sensation over the lateral forearm and would not cause sensory deficits in the hand.

The radial nerve can be injured above the elbow (Choice I) as a result of compression at the axi lla from using crutches or sleeping with the arm over a
chair. Injury to the radial nerve above or at the elbow (Choice J) would result in loss of elbow and wrist extension, as well as loss of sensation over the
posterior forearm and dorsal hand. This patient only has symptoms in the hand, resulting from a distal radial nerve injury.

The suprascapu lar nerve (Choice L) innervates the supraspinatus musclle that is responsible for abduction of the arm and the infraspinatus muscle that
is responsible for lateral rotation of the arm.

The thoracodorsal nerve (Choice M) innervates the latissimus dorsi muscle, which is responsible for adduction, extension, and internal rotation of the
arm.

The ulnar nerve (Choice N, 0, and P) can be injured in a medial epicondyle fracture of the humerus or a fracture of the hook of the hamate in the wrist.
A proximal injury at the elbow can cause impaired wrist flexion, and a distal injury in the wrist can cause impaired flexion of the medial fingers, and
abduction and adduction of the fingers. The ulnar nerve supplies sensation to the volar and dorsal aspects of the medial fingers and the hypothenar ....

r ~, ~ ~ r,
Previous Next Score Report
https://t.me/USMLENBME2CK Lab Values Calculator Help Pause
Exam Section: Item 49 of 50 National Board of Medical Examiners
■ Mark Clinical Neurology Self-Assessment

superficial radial nerve in the wrist. The median nerve supplies sensation to the volar and dorsal distal aspects of the lateral three and a half fingers ...
and lateral palm of the hand (thenar eminence), whereas the radial nerve supplies sensation to the dorsal bases of the lateral three and a half fingers
and the lateral dorsum of the hand. This patient has impaired sensation along the dorsum of the hand, consistent with a radial nerve injury at the wrist
or distal forearm.

Incorrect Answers: A, B, C, D, E, F, G, H, I, J, L, M, N, 0, and P.

Axil lary nerve (Choice A) palsy can occur after a humeral neck fracture or shoulder dislocation. The axillary nerve innervates the deltoid muscle,
leading to impaired arm abduction and loss of sensation over the deltoid muscle.

The long thoracic nerve (Choice B) innervates the serratus anterior muscle. Injury to this nerve can lead to a winged scapu la.

The median nerve (Choices C, D, and E) can be injured in a supracondylar humerus fracture, a fracture dislocation of the wrist, or a laceration along
its course. A median nerve injury can cause symptoms depending on the location of the lesion, causing palsy distal to the injury. Median nerve palsy
can result in impaired wrist flexion, thumb opposition, or flexion of the lateral fingers; it can also cause impaired sensation to the thenar eminence and
volar and dorsal distal aspects of the lateral three and a half fingers.

The musculocutaneous nerve (Choices F, G, and H) courses proximal in the arm, and injury can cause impaired forearm flexion and supination. It
supplies sensation over the lateral forearm and would not cause sensory deficits in the hand.

The radial nerve can be injured above the elbow (Choice I) as a result of compression at the axi lla from using crutches or sleeping with the arm over a
chair. Injury to the radial nerve above or at the elbow (Choice J) would result in loss of elbow and wrist extension, as well as loss of sensation over the
posterior forearm and dorsal hand. This patient only has symptoms in the hand, resulting from a distal radial nerve injury.

The suprascapu lar nerve (Choice L) innervates the supraspinatus musclle that is responsible for abduction of the arm and the infraspinatus muscle that
is responsible for lateral rotation of the arm.

The thoracodorsal nerve (Choice M) innervates the latissimus dorsi muscle, which is responsible for adduction, extension, and internal rotation of the
arm.

The ulnar nerve (Choice N, 0, and P) can be injured in a medial epicondyle fracture of the humerus or a fracture of the hook of the hamate in the wrist.
A proximal injury at the elbow can cause impaired wrist flexion, and a distal injury in the wrist can cause impaired flexion of the medial fingers, and
abduction and adduction of the fingers. The ulnar nerve supplies sensation to the volar and dorsal aspects of the medial fingers and the hypothenar
eminence.

Educational Objective: Operative management of a distal radius fracture can be complicated by radial or median nerve injury. The median nerve
supplies sensation to the volar and dorsal distal aspects of the lateral three and a half fingers and lateral palm of the hand (thenar eminence), whereas
the radial nerve supplies sensation to the dorsal bases of the lateral three and a half fingers and the lateral dorsum of the hand. ....

r ~, ~ ~ r,
Previous Next Score Report
https://t.me/USMLENBME2CK Lab Values Calculator Help Pause
Exam Section: Item 50 of 50 National Board of Medical Examiners
■ Mark Clinical Neurology Self-Assessment

50. A 62-year-old woman is brought to the emergency department because of a 3-day history of progressive weakness and numbness. These
symptoms first occurred in her feet and have spread to include her chest. She also has had difficulty emptying her bladder completely. She has
moderate midback pain that began 1 month ago and now radiates to her chest; it is exacerbated when she coughs. One year ago, she had breast
cancer and underwent lumpectomy followed by radiation and chemotherapy. Her only medication is tamoxifen. Vital signs are within normal limits.
Muscle strength is 5/5 in the upper extremities and 4/5 in the lower extrem ities with increased tone. Deep tendon reflexes are 2+ in the upper
extremities and 4+ in the lower extremities. Babinski sign is present bilaterally. Sensation to pinprick is decreased below the T4 level. Which of the
following is the most likely diagnosis?

A) Epidural spinal cord compression


B) Guillain-Barre syndrome
C) Leptomeningeal carcinomatosis
D) Paraneoplastic myelopathy
E) Transverse myelitis
Correct Answer: A.

Spinal cord compression can occur because of malignancy, trauma (eg, compression fractures with fragment retropu lsion), disc
herniation, spinal stenosis, spinal epidural abscess, or spinal epidural hematoma. Neurologic symptoms depend on the site of spinal
cord compression, with sensation and motor deficits below the level of the lesion. Patients may manifest upper motoneuron
examination findings such as spasticity, hyperreflexia, and extensor plantar reflex (Babinski sign). Initial treatment for patients with
epidural spinal cord compression and neurologic deficits include symptomatic treatment, pain control, and glucocorticoid therapy prior
to definitive therapy. High-dose glucocorticoid therapy is helpful as a bridge to definitive therapy, can help control pain, and may
transiently improve neurologic function in patients secondary to decreasing edema at the site of cord compression.

Incorrect Answers: B, C, D, and E.

Guillain-Barre syndrome (Choice B) is an inflammatory, demyelinating polyneuropathy that occurs idiopathically or following infection
with a virus or bacteria, classical ly a gastrointestinal illness from a pathogen such as Campy/obacter Jejuni. It presents as ascending
paralytic weakness starting in the lower extremities and may eventually include respiratory muscles. It generally self-resolves but can
persist for months. It wou ld resu lt in hyporeflexia, not hyperreflexia as in this patient.

Leptomeningeal carcinomatosis (Choice C) refers to metastatic neoplastic involvement of the leptomeninges, or the arachnoid and pia
mater, of the central nervous system. Involvement of the leptomeninges can lead to hydrocephalus, increased intracranial pressure,
cranial nerve or spinal nerve dysfunction, and cerebral edema. Leptomeningeal carcinomatosis is characterized by multifocal
neurologic deficits, and patients may present with headache, altered mental status, cranial neuropathies, cerebellar dysfunction, ....
-- -- -- - - -- - - -- - - - - - - - - - - -- -
r ~, ~ ~ r,
Previous Next Score Report
https://t.me/USMLENBME2CK Lab Values Calculator Help Pause
Exam Section: Item 50 of 50 National Board of Medical Examiners
■ Mark Clinical Neurology Self-Assessment

E) Transverse myelitis ...


Correct Answer: A.

Spinal cord compression can occur because of malignancy, trauma (eg, compression fractures with fragment retropu lsion), disc
herniation, spinal stenosis, spinal epidural abscess, or spinal epidural hematoma. Neurologic symptoms depend on the site of spinal
cord compression, with sensation and motor deficits below the level of the lesion. Patients may manifest upper motoneuron
examination findings such as spasticity, hyperreflexia, and extensor plantar reflex (Babinski sign). Initial treatment for patients with
epidural spinal cord compression and neurologic deficits include symptomatic treatment, pain control, and glucocorticoid therapy prior
to definitive therapy. High-dose glucocorticoid therapy is helpful as a bridge to definitive therapy, can help control pain, and may
transiently improve neurologic function in patients secondary to decreasing edema at the site of cord compression.

Incorrect Answers: B, C, D, and E.

Guillain-Barre syndrome (Choice B) is an inflammatory, demyelinating polyneuropathy that occurs idiopathically or following infection
with a virus or bacteria, classical ly a gastrointestinal illness from a pathogen such as Campy/obacter Jejuni. It presents as ascending
paralytic weakness starting in the lower extremities and may eventually include respiratory muscles. It generally self-resolves but can
persist for months. It wou ld resu lt in hyporeflexia, not hyperreflexia as in this patient.

Leptomeningeal carcinomatosis (Choice C) refers to metastatic neoplastic involvement of the leptomeninges, or the arachnoid and pia
mater, of the central nervous system. Involvement of the leptomeninges can lead to hydrocephalus, increased intracranial pressure,
cranial nerve or spinal nerve dysfunction, and cerebral edema. Leptomeningeal carcinomatosis is characterized by multifocal
neurologic deficits, and patients may present with headache, altered mental status, cranial neuropathies, cerebellar dysfunction,
seizures, radiculopathy, or cauda equina syndrome.

Paraneoplastic myelopathy (Choice D) can occur in malignancy and presents with progressive spastic paresis, and can be associated
with encephalitis, sensory neuronopathy, optic neuropathy, or chorea.

Transverse myelitis (Choice E) is caused by inflammation across both sides of one segment of the spinal cord. Al l sensory and motor
pathways are either partially or completely interrupted. Like spinal cord compression, there is often motor loss and diminished
sensation in all dermatomes below the level of the lesion. Transverse myelitis can be idiopathic and some cases fol low an antecedent
respiratory or gastrointestinal illness. It has also been associated with central or systemic autoimmune disorders such as multiple
sclerosis or systemic lupus erythematosus.

Educational Objective: Spinal cord compression can occur because of malignancy and metastases to the spine. Neurologic symptoms
depend on the site of spinal cord compression, with sensation and motor deficits below the level of the lesion. Patients may manifest
upper motoneuron examination findings such as spasticity, hyperreflexia, and extensor plantar reflex (Babinski sign).

.,.

r ~, ~ ~ r,
Previous Next Score Report
https://t.me/USMLENBME2CK Lab Values Calculator Help Pause

You might also like